Download Question 1 - finals2009

Document related concepts

Infection control wikipedia , lookup

Epidemiology of metabolic syndrome wikipedia , lookup

Prenatal testing wikipedia , lookup

Dental emergency wikipedia , lookup

Multiple sclerosis research wikipedia , lookup

List of medical mnemonics wikipedia , lookup

Transcript
1.2
Question 1
As an obstetric SHO, you are presented with a primigravida who is 41 weeks + 2 days
pregnant.
She has had an uneventful pregnancy but has been admitted to hospital for
induction of labour.
you for advice.
She is unsure how to prepare a feed of formula milk.
She asks
List four pieces of practical advice you would give her on how to
prepare a feed.
Any four of the following;
•Hands must be washed and absolutely clean before handling the bottles and teats
• Suitable bottles and teats must be thoroughly washed and sterilised before use
•Water should be boiled before it is used to make a feed
•Information on how much water and the number of scoops of formula powder
needed for a feed for a baby of various ages can be found on the back of the
manufacture’s formula milk container.
•Water should be put into the bottle FIRST and accurately measured in the bottle by
using the fluid ounces/mls guide at the side of bottle.
•The correct number of scoops of formula powder should be put into the bottle AFTER
the water has been added
•A clean knife should be used to scrape off any excess powder in each scoop to
ensure the correct amount of formula powder is in each scoop
•The bottle filled with water and formula powder should be GENTLY shaken to ensure
adequate mixing
•The milk should be allowed to cool and tested on the back of the hand before giving it
to the baby
•The milk, once prepared can be stored in the fridge for up to 24 hours.
•The best way to warm a refrigerated bottle is by placing it in a jug of warm water.
Take great care if you heat the feed in a microwave oven because of the risk of hot
spot burning.
Question 2
A mother presents to the GP clinic and is concerned about when and how to wean her
20 week old baby.
a. What is the current recommended age for weaning a baby?
6 months (recommended by department of health)
1
b. List three signs are suggestive that a child is ready for weaning?
The baby is hungrier and cries more for food
The baby wakes up for a feed more time during the night than previously
The baby shows more interest in food
The baby starts to make chewing motions with their mouth
c. What types of foods can be initially introduced to the baby? List two.
Baby Rice cereal mixed in with breast or formula milk (recommended by the
department of health as a starter)
Pureed fruit (e.g. Banana)
Pureed cooked vegetables ( e.g potatoes, carrots broccoli, peas, parsnips)
d. List two types of food you would advice the mother to avoid in the first stages of
weaning
Gluten containing products (if weaned before 6 months)
Don't add sugar or salt to any foods
Don't feed hot spices in the first 12 months
Avoid (until 12 months) raw or soft eggs, patè, soft cheeses such as brie, honey( in
rare cases it can lead to infant botulism
Avoid peanut butter until 12 months or 3 years if allergies run in the family,
Avoid whole nuts until 6 years (due to the dangers of choking).
e. The mother asks you ‘What about milk, do I have to discontinue that now?’
What
will you reply? Give a reason for your answer.
You should reply ‘No’
The reason for this is that babies still rely on milk for most of their energy (calorie)
intake.
1.2
Feeding
By the end of the course students should be able to:
•
make and give a feed
•
give advice to parents about infant feeding and weaning
2
1.5
Question 1
A two-day-old term infant on a postnatal ward has had bilious vomits after every feed
since birth.
The baby is hungry and feeds well but is yet to pass meconium. Clinical
examination reveals a dysmorphic child with a flat occiput, large tongue, low set ears
and single palmar crease. A plain abdominal film was taken and showed a double bubble
appearance in the stomach.
a. What is the ‘double bubble appearance’ indicative of?
Duodenal atresia
b. What genetic condition does this child have that accounts for his dysmorphic
features?
Down’s Syndrome
c. List three further investigations you would perform and give a reason for each
investigation stated
Karyotype: To confirm the diagnosis of chromosomal abnormality
Upper gastrointestinal (GI) contrast study: There has been no passage of
meconium
for 48 hours
Echocardiogram: Congenital heart disease should be actively looked for in babies with
Down’s syndrome
d. Give three possible short term or long term complications of this condition that may
necessitate intervention.
Any three of the following:
Congenital heart disease
Duodenal atresia
Severe learning disability
Small stature
Recurrent respiratory infections
Hearing impairment from acute otitis media
Visual impairment from cataracts, squints
Increased risk of leukaemia
Atlanto-axial subluxation (rare)
Hypothyroidsm
3
Alzheimer’s disease
e. What information, support or advice can be given to the parents at the time of
diagnosis and thereafter?
Any one of the following:
Written explanation of condition and its cause
Information about the short term and long term implications of diagnosis
Information about national charities and self help groups e.g Down Syndrome
Association
Professional counselling to help deal with feelings of guilt, disappointment or
anger
Antenatal diagnosis for future pregnancies
Question 2
Ben aged two months is admitted with tachypnoea following an upper respiratory tract
infection.
On examination there is a thrill, a loud (grade4/6) pansystolic murmur at the
left sternal edge and the liver is palpable 4 cm below the costal margin.
a. What is the most likely cause of his clinical condition?
Heart failure, probably precipitated by an upper respiratory tract infection
b. What is the most likely underlying diagnosis?
Ventricular septal defect (VSD)
c. A chest x-ray was performed. Name 3 signs on the x-ray that may be indicative of
your chosen diagnosis
Cardiomegaly,
Enlarged pulmonary arteries,
Increased pulmonary vascular markings
Pulmonary oedema
d. What single diagnostic investigation would you perform now?
Echocardiography
e. Outline 2 important aspects of management that may be considered in this
condition and give a reason for performing each intervention
1) Drug therapy – furosemide, thiazide, spironolactone, ACE inhibitors
Reason – To treat symptomatic heart failure
2) Surgery
Reason: Severe symptoms with failure to thrive OR
Reason: Pulmonary hypertension with possible progression to pulmonary vascular
4
disease and subsequent Eisenmenger’s syndrome.
3.) Monitoring and observation (symptoms, signs, CXR, echocardiography)
Reason: This is acceptable in asymptomatic patients.
By the age of 2 years at
least 50% of small and moderate sized VSDs undergo sufficient partial or
complete spontaneous closure to make intervention unnecessary.
1.5
Congenital Abnormalities
By the end of Phase II , students should be able to:
•
recognise and explain to parents the principles of management of the commoner
congenital abnormalities
5
1.6
1. Nick and Judith have been blessed with a bouncing baby boy, Gavin.
What three measurements are plotted on his centile growth chart?
Head circumference
Weight
Height
Being distinctly average, he follows the 50th centiles. What would be noted on his chart
to constitute mild failure to thrive?
Mild failure to thrive is a fall across two centile lines.
(Severe failure to thrive is a fall over three centile lines)
Name two Organic and two Non-Organic causes of failure to thrive.
Organic:
Inability to feed
Poor retention of food
Illness-induced anorexia
Impaired nutrient absorption
Increased energy requirements
Metabolic causes
Chromosomal disorders/syndromes
Congenital infection
Non-Organic:
Feeding problems
Maternal stress
Lack of stimulation and undernutrition
Munchausen’s syndrome by proxy
Three years later, Gavin’s parents tell you he has reached all his developmental
milestones at the correct age.
Name three of the five developmental areas that are measured.
6
Gross motor
Fine motor
Vision
Hearing and Speech
Social behaviour
At what median age are the following milestones achieved?
Crawls –
8-9 months
Has a mature pincer grip –
10 months
Smiles responsively 6 weeks
What screening tool is used for testing hearing in infants?
The Distraction test
At what age would this tool be used?
6-9 months
2. Child X has cerebral palsy. Name two ways in which cerebral palsy might present.
Abnormal tone and posturing
Feeding difficulties
Delayed motor milestones
Abnormal gait when walking is achieved
Language and social developmental delay
The management of Child X’s cerebral palsy will involve a multidisciplinary team
approach.
Name four members who might be involved.
7
Paediatrician
General Practitioner
Eric’s Family
Physiotherapist
Occupational therapist
Speech and language therapist
Social worker
Health visitor
Surgeon (e.g. fundoplicaiton for recurrent oesophageal reflux)
1.6
Developmental child health
By the end of Phase II , students should be able to:
•
recognise failure to thrive, its common causes and initiate management
•
make a developmental assessment of the infant and toddler
•
recognise delay in speech and in walking
•
examine a child for hearing loss, including distraction testing
•
examine a child for reduced visual acuity and squint
•
outline to parents the facilities available for children with learning difficulties
•
outline to parents the facilities available for children with mobility difficulties
•
recognise short stature and refer appropriately
•
recognise delayed puberty and discuss the causes and investigation with patients and
parents
8
1.7
JB is a child who has been diagnosed with cystic fibrosis.
This requires daily
medication, frequent visits to hospital and several hospital admissions for respiratory
illness.
When he is 3 he is admitted to hospital for 2 weeks.
During this time his
parents are unable to visit.
1) What are the stages of the acute separation response in young children?
Protest
Despair
Detachment
2) What factors in family life can have a negative impact on the psychological wellbeing
of a child?
Give 6 examples.
Parental mental illness
Inconsistent, unpredictable discipline
Divorce
Inappropriate expectations/responsibilities for age.
Abuse
Intrusive overprotection.
Bullying.
3) What are the common disorders of psychological development that could present in
any child at age <5 yrs, 5-10 years old and as an adolescent? Give 3 examples for each.
-
Age <5 yrs
Meal refusal
Sleeeping problems (night terrors, nightmares, waking at night)
Breath-holding attacks
(Aggressive behaviour, Autism, Tantrums).
-
5-10 years old
Nocturnal enuresis
Hyperactivity
School refusal
(Anxiety, antisocial behaviour, Tics, Faecal soiling)
-
Adolescence
Anorexia nervosa
9
Chronic fatigue syndrome
Deliberate self-harm
(Drug misuse, depression)
JB starts at nursery, and the nursery nurse expresses concerns that he may have
features of an autistic spectrum disorder.
4) What are the three main features of autism?
Poor social interaction
Lack of imagination
Poor communication skills.
5) What other behaviour patterns can be seen in autism?
Give 2 examples.
Obsessive behaviours
Repetitive behaviours
When JB is 9 he starts refusing to go to school.
6) What are the two main causes of school refusal?
Separation anxiety persisting beyond the toddler years.
School phobia – anxiety provoked by an aspect of school.
7) Name 3 professional groups who could be involved when an older child refuses to go
to school.
Teachers
Educational psychologist
Educational welfare officer
8) Name 6 behavioural features that might suggest attention deficit disorder.
Fidgeting
Continually interrupts
Instructions never obeyed
Blurts out answers
Behind with schoolwork
Short attention span.
10
9) What are the social and emotional issues that JB might face when he becomes a
teenager?
Low self esteem
Problems gaining independence from parents
Rebelling against medication regime.
Problems conforming with peer group because of limitations of illness.
Depression
Anxiety
Alcohol and drug misuse
1.7
Child Psychiatry
By the end of Phase II , students should be able to:
• recognise the common presentations of psychiatric disorder in children and distinguish the
common causes
• recognise the interaction between child and family upon the psychological disorders of
childhood
• discuss attachment theory
• recognise the common disorders of childhood psychological development
• recognise the impact on children of physical/mental illnesses in a parent
• recognise the psychological aspects of chronic physical illness in children
• discuss school refusal with a child and its parents
• recognise the social and emotional issues in adolescents with chronic illness
11
1.10
1) You are a house officer in A+E and you see an 8 month old child who has had
vomiting and diarrhoea for 2 days.
What signs would you look for to assess hydration? (6 marks)
Any 6 of:
Sunken fontanelle
Prolonged capillary refill time
Dry mucous membranes
Reduced skin turgor
Sunken eyes/tearless
Oliguria
Tachypnoea
Tachycardia
Reduced level of consciousness
Hypotension
How would you clinically assess severity of dehydration in terms of body weight loss?
(3 marks)
Mild dehydration
< 5%
Moderate dehydration 5-10%
Severe dehydration >10%
Name some routes through which rehydration therapy be administered? (2 marks)
Oral
NG tube
Intravenous
Intra-osseous
Name a micro-organism that commonly causes gastroenteritis in children in developed
countries? (1 mark)
Any of:
12
Viruses –Rotavirus, Adenovirus, Coronavirus
Bacteria – Campylobactor, Shigella, E.Coli, Staphylococcus, Salmonella
2) A 2 year old boy is given the diagnosis of coeliac disease in outpatient clinic.
What symptoms and signs would suggest this diagnosis? (6 marks)
Any of:
Failure to thrive
Abnormal stools/diarrhoea – foul smelling, pale, frequent and bulky
Vomiting
Weight loss
Constipation
Short stature
Abdominal distension
Buttock wasting
Irritability
Delayed motor milestones
Glossitis, angular stomatitis
Dermatitis Herpetiformis
What serological investigations would you consider? (2 marks)
Any of:
Anti-gliadin antibodies
Anti-endomysial antibodies
Anti-tissue transglutaminase antibodies
Anti-reticulin antibodies
What advice would you give to parents about the long term management and
implications of coeliac disease? (2 marks)
13
Any of:
Removal of all products containing wheat, rye and barley from the diet for life
Referral to a dietician
Possibility of a gluten challenge later in childhood to reassess the susceptibility of the
jejunum to damage by gluten
Life long follow-up
Increased risk of complications in later life
Extended questions:
What other diseases are patients with coeliac disease more prone to in later life? (1
mark)
Any of:
Small bowel malignancy
Enteropathy associated T cell lymphoma
Increased risk of other autoimmune disorders – thyroid, adrenal, pancreas, SLE, RA
Osteoporosis
Ulcerative jejunoileitis
The gold standard investigation for diagnosis of coeliac disease is jejunal biopsy.
What histological features would you expect to see in the jejunum of a patient with
coeliac disease? (2 marks)
Any of:
Lymphocyte infiltration
Subtotal villous atrophy
Crypt atrophy
Increased intra-epithelial cells
3) A 4 year old child presents with constipation.
What are the symptoms which would suggest a diagnosis of constipation? (2 marks)
Any of:
Child passes <1 motion every 3 days
14
Passage of stools that are hard, small or pellet like
Stools that are difficult or painful to pass
What lifestyle changes would you suggest to parents to help manage the constipation?
(2 marks)
Any of:
Increase fluid intake
Diet high in fibre, fruit and vegetables
Regular toileting
What is encopresis, and how can affect a child and their family? (3 marks)
Encopresis is the passage of stools in unacceptable places in a child over the age of
4 years old.
Any sort of bio-psycho-social answer eg. Increased washing of clothes,
teasing/bullying at school, increased risk of abuse,
Name two different types of laxatives available and give an example of each? (4 marks)
Any of:
Bulk forming laxatives – Ispaghula Husk (Fybogel, Fibrelief)
Stimulant laxatives – Senna (or Bisacodyl, Dantron, Sodium Picosulfate, Glycerol)
Faecal softeners – Liquid paraffin (or arachis oil)
Osmotic laxatives – Lactulose (or macrogols, magnesium salts, phosphates, sodium
citrate)
Bowel cleansing preparations – Picolax (or Klean-prep, Citramag)
Extended question:
Name some organic causes of constipation (2 marks)
Any of
Hypothyroidism
Hypercalcaemia
Hirschprungs disease
15
Urinary concentrating defect
Anorectal stenotic lesions
Anal Fissures
Spinal/Neuromuscular disease
1.10
Gastrointestinal Disease
By the end of Phase II , students should be able to:
• recognise the common causes of feeding difficulty and initiate management
• discuss feeding difficulties with parents
• recognise dehydration and the associated biochemical changes
• recognise the common causes of vomiting in infancy, childhood and adolescence and initiate
management
• recognise the common causes of acute and chronic diarrhoea in infancy, childhood and
adolescence and initiate management
• recognise the common causes of constipation in infancy, childhood and adolescence and
initiate management
• give advice to parents on gastroenteritis
• recognise and assess obesity
• make an initial assessment of a child with acute abdominal pain and refer appropriately
• recognise the common causes of chronic abdominal pain
• recognise the common causes of an abdominal mass in infancy, childhood and
adolescence
16
1.11
A 4 year old Asian boy presents with a history of swelling around his eyes, most
noticeable in the morning for the preceding 5 days. His parents have noted that his
urine appears frothy and is reduced in volume.
Q1. What is the most likely diagnosis?
Nephrotic syndrome
Q2. Apart from peri-oribital and ankle swelling what other areas would you look for
swelling particularly if this was a severe case?
Swelling of scrotum
On examination there is dullness to percussion and reduced breath sounds at the right
base.
Q3. What are these clinical findings suggestive of and why has it happened?
Pleural effusion which is secondary to hypoalbunimaemia
Q4. What initial investigations would be implemented to confirm this diagnosis?
Urine dipstick
U+ E
albumin levels
microscopy and culture
Q5. What is the treatment for this patient and how long should the patient be treated
for?
17
Oral steroids – prednisolone 60mg/m2/day for a period of 10days
Q6. What are the side-effects of these drugs in children? Name 2.
Growth stunting and immunosuppresion
Q7. What are the serious complications which may occur with this condition? (2)
Hypovolaemia (intravascular compartment becomes depleted)
Thrombosis ( hypercoaguable state due to urinary loss of antithrombin and increased
blood viscocity from raised haematocrit predisposes to thrombosis).
Q8. What prophylaxis should be given in the acute phase of this condition and why?
Pencillin prophylaxis is given in acute phase to prevent pneumococcus infection due
to urinary loss of immunoglobulins increasing susceptibility to infection
1.11
Renal/urinary
By the end of Phase II , students should be able to:
•
recognise oedema in infancy, childhood and adolescence, discuss its causes and
management with parents
•
recognise a history of haematuria, and initiate investigations
•
recognise polyuria and initiate investigations
•
identify the commoner causes of bedwetting and give advice to parents
•
fit a urine bag, obtain a specimen for testing
•
explain the need for suprapubic aspiration to parents
•
recognise and interpret the urine abnormalities in urinary infections
18
1.12
Qu. 1. Lucy is a 2 year old girl who presents to A & E. The parents are very worried, they
say that Lucy has been ill with a temperature for a few days and this morning just after
breakfast she had a ‘fit’ which lasted about quarter of an hour.
a) List three causes of Lucy’s fit in your differential diagnosis:
Febrile convulsion
Meningitis with a febrile convulsion.
Infantile spasm (3-12 months of age).
b) To help you distinguish which of your differentials is most likely list 6 things you
would specifically want to rule out on examination?
Photosensitivity,
Neck stiffness.
Fever,
Purpuric rash,
Signs of shock
Positive Brudzinski’s/Kernig signs
c) Your SHO performs a lumbar puncture on Lucy. What changes would you expect in
the CSF for both bacterial and viral meningitis?
1) Bacterial: Turbid CSF, increased (!!) polymorphs, increased
proteins,
increased (!!) glucose.
2) Viral: Clear CSF, Lymphocytes increased, Normal or increased (!) protein,
normal or increased (!) glucose.
d) What are the main causative agents of meningitis?
1 month-6 years:
Neisseria meningitides
Streptococcus pneumoniae.
Haemophilus influenzae
e) What is the immediate management for meningitis? Step 1 is done for you.
19
1) ABC
Start antibiotics - Third generation Cephalosporin (Cefotaxime)
plus either
Rifampicin or Vancomycin whilst waiting for CSF culture results..
CSF lumbar puncture (ideally CT first )
Check antibiotic regime against CSF results and change if
necessary.
Establish if there are any contacts.
Reassess
f) List 4 complications of meningitis:
Hearing loss.
Local vasculitis - cranial nerve pathology.
Local cerebral infarction.- epilepsy.
Hydrocephalus.
Other: Cerebral abcess and Subdural effusion.
Extended: What antibiotic would you give to anyone Lucy may have had contact with
who is at risk?
Rifampicin.
Qu.2. Dave is a 15 year old boy who presents to his GP with his mum. Whilst shopping
his mum witnessed Dave having a ‘fit’. She explains that he suddenly dropped to the
floor and at first was still but then started to shake. When he came round he was sleepy
and spent the rest of the day in bed. His mum is very worried.
a) What extra information would be useful to know?
Was he incontinent?
Has it happened before?
Did he have any preceding aura or warning sign.
Is he on any medication?
Does he use recreational drugs?
Does he drink alcohol?
b) Name 5 different types of generalised epilepsies.
Absence seizures.
20
Myoclonic seizures.
Tonic seizures.
Tonic-clonic seizures.
Atonic seizures.
c) Dave admits that this is his fourth fit. His friends had seen two of the previous three
fits and had described the same fit in the same way as his mum. What investigations are
required?
Blood tests checking electrolytes.
Serum glucose.
EEG.
CT or MRI.
d) Name three anticonvulsant drugs that you could prescribe for generalised
tonic-clonic seizures?
Sodium valporate.
Carbamazepine.
Lamotrigine.
e) Dave explains that once has finished school he wants to join the army. Mention 5
things you would want to cover with Dave and his mum?
Provide written information (Leaflets) on epilepsy.
Give details of support groups.
3) Educate both Dave and his mum about the recovery position and
how rectal
diazepam is useful and how it should be administered.
4) Explain that there are driving restrictions imposed by the DVLA regarding
how
long he must be free of fits before he is allowed to drive. He should look
these
up when he wants to learn to drive.
He will need to contact the army to see if he will be eligible to join.
1.12
Central Nervous System
By the end of Phase II , students should be able to:
•
take a history of febrile fits
•
take a history of epilepsy in infancy, childhood and adolescence
21
•
outline to parents the nature and treatment of febrile fits and epilepsy
•
examine a floppy baby and discuss the nature of the problem with parents
•
recognise the possibility of meningitis and initiate management
•
recognise and interpret the CSF abnormalities of meningitis
•
recognise the common causes of abnormalities in gait in infancy, childhood and
adolescence
22
1.13
A 7 year old boy presents to A&E with after concerns from school about the extent of
continuous bruising on the buttocks and legs.
Q1. What would concern you about this pattern of bruising illustrated in Figure 1?
Unexplained injuries to protected parts of the body such as the buttock may indicate
the presence of non-accidental injury.
The presence of bruises that have various
ages may signify multiple episodes of injury caused by ongoing physical abuse
Q2.What other areas of the body if bruised would raise your suspicions of a diagnosis
of bruising caused by non-accidental injury?
Thighs, torso, frenulum, ears and neck
Q3. How can you make a rough estimate the age of the bruising in this picture?
By the colour of the bruise. The picture shows yellow/brown colour indicating that
these bruises are older than a day. A bruise of within a day will be erythematous
Q4. What is the difficulty with determining the bruise age by colour?
23
Determining the bruise's age by its color must take into account the individual
patient's skin tone, the bruise's location (whether overlying soft tissue or bone), the
patient's relative nutritional state or other indeterminate factors.
Q5. What investigation may be indicated in the presence of suspicious bruising?
Platelet count and coagulation screen to exclude thrombocytopaenia and other
bleeding diatheses which often presents with multiple and excessive bruising.
Q6.
How would you proceed with this case and whom should you inform?
If child abuse is suspected in the UK the Social Services Child Family department
must be informed. A decision must be made whether the child requires immediate
protection either hospital admission or placement in foster home. In most cases
further management will involve evaluation of the family by social workers and Child
Protection Conference.
Q7. Who should be present at the Child Protection Conference?
Paediatrician, GP, health visitor, social worker, police, nursing staff, representatives
from school and parents.
1.13
Haematology
By the end of Phase II , students should be able to:
•
assess anaemia in infancy and childhood
•
assess a child with bruising
24
1.14
SW is a 5-year-old male who presented to A&E with severe vomiting, abdominal pain,
and drowsiness. This had lasted for approximately 4 hours and was preceded by 1 week
history of flu like illness. On examination he had signs of weight loss and dehydration.
A) What would be your main differential in SW? Give two other examples of differential
diagnoses.
Diabetic Ketoacidosis
Gastroenteritis
Appendicitis
B) After initial ABC management, give three other management steps.
Rehydration with fluids (normal saline over 48-72hrs)
Insulin administration (sliding scale starting at 0.1units/kg/hr)
Potassium infusion.
(metabolic acidosis should correct itself following fluid administration).
C) SW was newly diagnosed with insulin dependent diabetes mellitus following this
incident. Give an example of how you would explain to his parents what this condition
is and how it needs to be managed.
“Your son has been diagnosed with diabetes which means that he is not producing
insulin within his pancreas, which we all need to regulate our blood sugar. This is due
to an autoimmune process, where there is destruction of the islet cells within the
pancreas which produce the insulin. Unfortunately he will need to be put on insulin,
and will need daily injections throughout his life. If his condition is managed effectively
then he will almost certainly lead a normal life.”
D) SW is responding well to treatment, give two regular investigations that his parents
25
could perform at home to monitor his diabetes.
Blood glucose.
Urine dipsticks (to check for protein and ketones).
E) List 3 key members of the diabetes team which will be essential in providing
continuing care throughout SW’s childhood.
-Consultant Paediatrician (with an interest in diabetes)
-Diabetic specialist nurse
-Paediatric dietician
F) SW needs to be placed upon an insulin regime. Give 2 examples of possible regimes:
Twice-daily regimen – Twice daily short acting and intermediate acting insulin
Basal-bolus regimen – Three times daily short acting boluses and one or two medium
acting basal boluses of insulin.
1.14
Diabetes
By the end of Phase II , students should be able to:
•
diagnose and assess diabetes
•
outline to 'parents' in a role play situation the management of childhood diabetes
•
perform ward-based blood and urine tests and interpret the results
26
1.15
A 1 year old boy, DS, is brought to the GP by his mother with an itchy rash, which is
diagnosed as atopic eczema.
1) What are the other possible diagnoses.
Give 4.
Contact dermatitis
Pityriasis rosea
Fungal infection
Psoriasis
Urticaria
Chickenpox
2) What features would you expect to find on examination of this child’s rash.
Give 4
points
present mainly on face and scalp.
(Occurs mainly on flexure surfaces in older
children)
excoriation
weeping rash
crusted rash
erythematous rash
Dry skin.
3) What type of illnesses is this child at increased risk of developing?
Atopic disease – hayfever, asthma.
4) What is the underlying pathological reaction.
Type I Hypersensitivity
Increased IgE production in response to common environmental allergens.
27
5) What has caused this serious exacerbation of this child’s eczema?
Infection with Herpes Simplex Virus
6) What else could cause an exacerbation of this child’s eczema?
Give 5 points
Staph aureus infection
Contact with an irritant/allergen
Psychological stress
Idiopathic
Environment (heat, humidity)
7) What simple advice could you give to DS’s mother for managing this child’s eczema?
Give 4 points.
28
Avoid soap and detergents
Avoid nylon and wool clothing next to the skin
Cut nails short (to reduce damage from scratching)
Avoid any precipitating allergens.
8) What medical treatments are available for the treatment of eczema?
Give 5.
Emollients
Topical corticosteroids
Occlusive bandages
Antibiotics
H1 histamine antagonists
1.15
Skin disease
By the end of Phase II , students should be able to:
• recognise the common skin conditions in infancy, childhood and adolescence
• outline to parents the management strategies available
29
1.16
PW is a 2 year old girl who presents to A&E with a painful swollen right arm.
Give 3 salient points that you would ask within the history.
-What was she doing at the time of injury?
-Any previous history of injury to the arm?
-Any other injuries?
What investigations would you order?
X-ray of site of injury.
Blood results (coagulation disorders, calcium and phosphate levels, copper deficiency)
What type of fracture is shown in the radiograph above?
Greenstick fracture of the right radius
30
A CxR taken at the same time is shown below:
What abnormality is shown by the arrows?
healed rib fractures
What other points in the history may aid a diagnosis of NAI?
Previous injuries in unlikely areas e.g. skull fractures, facial bruising, cigarette burns.
Inconsistent stories
Social circumstances e.g. underlying drug / alcohol abuse within the family.
If you do suspect non-accidental injury, who else would you involve in the
investigation and management? List three.
Social services
Health worker
GP
Community Paediatrician
Police in severe cases
Give one other important diagnosis which may produce the picture seen in
radiograph B, and what other examination findings may substantiate this
diagnosis.
31
Osteogenesis imperfecta
Blue sclera
Skeletal deformities
Short stature
Joint laxity
(Excellent question)
Give the pathophysiology of this condition.
Osteogenesis imperfecta is a condition resulting from abnormality in the type I
collagen. Type I collagen fibers are found in bones, organ capsules, fascia, cornea,
sclera, tendons, meninges, and dermis. Structurally, this protein is composed of a
left-handed helix formed by intertwining of pro-a1 and pro-a2 chains. Mutations in the
loci coding for these chains result in osteogenesis.
1.16
Fractures in Children
By the end of their teaching the students should be able to:
•
recognise those fracture patterns that occur in children and their possible aetiological
factors
•
order appropriate investigations
•
be aware of non-accidental injury and know the outline strategy for the handling of non
accidental injuries
•
act as part of the team in the immediate and definitive care
32
1.17
Question 1
Harry, a 7 year-old boy has had pain in the right leg since he woke up this morning. He
finds it difficult to walk on the leg due to pain, which has been getting worse throughout
the day. His teachers at school said that he was unable to join in his P.E lesson because
he was in too much discomfort and his mother was asked to pick him up early from
school since the pain was getting significantly worse. The pain is in the groin, front of
the thigh and radiates down to the knee; is exacerbated by movements at the hip and
weight bearing. Harry does not have any pain in any other joints, and he otherwise feels
fit and well. There is no history of trauma; Harry was playing happily the previous day.
He had a moderately severe viral upper respiratory tract infection 2 weeks ago; but has
now fully recovered. All other past medical history was unremarkable. Harry is afebrile.
◦
List 3 differential diagnoses on the basis of this history:
transient monoarticular synovitis (TMS)
septic arthritis (SA)
juvenile rheumatoid arthritis
(others
to
consider;
osteomyelitis,
leukaemia
and
other
malignancies)
◦
What findings on examination would you look for to aid your diagnosis?
Assess gait and ability to weight bear (TMS – able to walk despite discomfort;
SA – severe pain, tenderness to palpation, spasm, and refusal to walk)
Examination of the hip: TMS – no signs of trauma; mild, or no effusion unlikely to be obviously swollen;
mild restriction of hip motion, especially abduction and internal rotation. (cf.
SA or bacterial infection – hip is held in a flexed, abducted and externally
rotated position and severely painful to move)
Vital signs: TMS – afebrile; SA – usually febrile (>38.50C)
◦
Are there any particular features of the history that give you a clue to the diagnosis?
**Past history of non-specific viral upper respiratory infection – Approximately 70% of
involved children have a history of prodromal symptoms indicative of a viral URTI
7-14 days before onset of TMS of the hip
Other features consistent with this diagnosis:
Sex: male-to-female ratio is 2:1; Age: mean age of onset is 6 years, most patients are
3-8 years of age; distribution and character of symptoms: monoarticular, acute onset,
33
progressive pain exacerbated by active movement at the joint but still able to weight
bear; otherwise in good health, no systemic symptoms associated; no history of
trauma.
Harry’s mother persuaded him to rest as much as possible and gave him some pain
relief, which eased his discomfort. However, after two days, Harry felt able to mobilise
again and was keen to get back to school and play football in the playground with his
friends. After one day back at school, running around, Harry’s hip became extremely
painful once again and he was unable to weight bear on it by the evening. His mother
took him to their GP the following day for further review.
◦
Given the progression of symptoms, what now seems to be the most likely
diagnosis?
Transient monoarticular synovitis (hip)
◦
What investigations would you instigate?
1st line: Blood tests (FBC, U&E’s, LFT’s)
AP and lateral pelvic x-rays
Others to consider: Aspiration of fluid if moderate effusion present – may improve
symptoms and improve local blood flow
Bone scans may help to differentiate a septic process
◦
What advice would you give Harry and his mother meanwhile to help ease the
discomfort?
Bedrest and non-weight bearing until the pain resolves, followed by limited activities
(for 1-2 weeks)
Important to communicate that rest is essential even following resolution of pain – if
the child returns to normal activities too early, exacerbation of the symptoms can
occur
NSAID’s can help for symptomatic relief
Advised to return to the GP for further evaluation if there is lack of
improvement/worsening of symptoms despite adequate rest to eliminate other, more
serious disorders.
Question 2
John is 5 years old. His mother is concerned because she has noticed him walking with
an increasingly obvious limp for the last 3 weeks. He does not complain of discomfort
when running or walking but has reported intermittent “soreness” of the leg from time
34
to time. When asked where it hurts, he points to the front of his right thigh. John is
otherwise in good health, has no recent history of illness and all other joints are non
painful and normally active/non-restrictive. There is no history of trauma and review of
all systems was unremarkable – he is afebrile.
On examination John appeared short for his age, and walked with a limp – minimal
weight bearing on the right leg. There was no discrepancy of leg length, swelling, signs
of trauma or localised tenderness on palpation. There was decreased range of
movement at the hip – poor abduction and internal rotation and muscle spasms were
seen in the quadriceps of the right leg. Examination of the left leg and hip was normal.
Investigations showed John to have delayed bone age.
◦
What is the most likely diagnosis?
Perthes disease – idiopathic avascular necrosis (osteonecrosis) of the
capital femoral epiphysis (CFE) and the associated complications in a
growing child. Caused by interruption of the CFE blood supply.
◦
What issues should be discussed with John and his mother about problems that
could occur in later life given the suspected diagnosis?
The short-term prognosis concerns femoral head deformity at the completion of the
healing stage. Development of late degenerative arthritis occurs; the incidence is
essentially 100% in children who are 10 years of age or older at onset. This rate is in
contrast to a negligible risk in children 5 years or younger and a 38% risk when onset
occurs between 6 and 9 years of age. Therefore, discussion should focus on the
potential that there is a moderate risk but the age of onset is of good prognostic value.
Long-term prognosis involves the potential for osteoarthritis of the hip in adulthood
◦
What features of the history suggest that there is a good prognosis?
Age – 5 years old
(The age related risk is associated with premature CFE closure. Early intervention to
prevent deformity of the femoral head prior to closure reduces the likelihood of
degenerative changes occurring, reducing the incidence of later complications/joint
disease and the associated morbidity in early adulthood.)
◦
Which specific investigation would help determine the severity of the condition?
AP and lateral pelvic x-ray – technique of choice for determining the extent of CFE
involvement, useful for following disease progression and identifying CFE collapse
and extrusion and assess response to treatment.
35
Occasionally additional procedures such as arthrography, bone scans, and MRI may
be useful. Bone scans and MRI are helpful in recognising early perthes disease but
are of limited value in assessing the extent of CFE involvement or following the
disease progression.
◦
What interventions could be considered (possibly not immediately) to improve
John’s prognosis and avoid complications in later life?
Surgical and non-surgical “containment” - the femoral head is contained within the
acetabulum, which acts as a mold for the reossifying CFE.
Perthes disease is a local, self-healing disorder – treatment focuses of prevention of
femoral head deformity and secondary osteoarthritis. Containment is indicated for
children 6 years of age or older in whom more than half the CFE is involved. This is
accomplished by non-surgical containment using abduction casts and orthoses or by
surgical containment with proximal femoral varus osteotomy – long term results are
85-90% satisfactory.
Objective 1.17
The Limping Child
By the end of their teaching the student should be able to:
•
examine the child with the painful hip or knee
•
consider the conditions of irritable hips, synovitis, systemic rheumatoid arthritis, slipped
upper femoral epipyhsis, Perthes disease, septic arthritis, osteomylitis and tumour
•
initiate appropriate investigations
•
outline treatment to the child and family
36
2.2
Miss AB is a 26 year old woman who has been to her GP with concerns that her period
is 3 weeks late.
1. Give two other symptoms which could be obtained form the history, and may
suggest that she is pregnant. (2 points)
breast tenderness
vomiting
2. There are several clinical signs which can be seen on examination of a pregnant
woman (later gestation), name 4. (4 points)
Striae gravidarum
linea nigra
areolae pigmentation
chloasma
montgomery’s tubercles (on the breast)
3. A urine pregnancy test is suggested. What substance does this investigation
detect, what is it produced by and what role does it play in pregnancy. (3 points)
ßhCG (human chorionic gonadotrophin)
produced by the trophoblast
it maintains the corpus luteum which secretes progesterone to maintain the
pregnancy
4. Routine ultrasound scan is performed at 11-14 weeks; suggest one use that this
has. (1 point)
confirm gestational age
identify multiple pregnancies
nuchal thickness in Down’s Syndrome
37
2.2
Diagnosis of pregnancy
By the end of Phase II , students should be able to:
•
diagnose pregnancy on clinical grounds
•
use tests of pregnancy appropriately
•
recognise the role of ultrasonography in the diagnosis of pregnancy
38
2.3
Mrs J is a 30 year old lady who has come to ask about starting a family. She has no
medical problems but smokes 20 cigarettes a day.
1. State 2 factors which you would advise her about and for each give a reason why. (4
points)
Smoking cessation: Any of; associated with IUGR, premature labour, miscarriages
Folic acid supplements: reduce the risk of neural tube defects
2. What important advice would you give if Mrs J had a pre-existing condition such as
diabetes or epilepsy? (1 point)
Ensure optimum management of pre-existing maternal disease prior to pregnancy if
possible and address potential teratogenic medications
(question not quite fit in: debate role of screening for genetic disease and be able to take a
family hx with reference to genetic disease)
2.3
Pre-pregnancy counselling and care
By the end of Phase II , students should be able to:
•
recognise the importance of pre-pregnancy counselling in particular groups
•
debate the role of screening for genetic disease
•
take a simple family history with reference to genetic disease
39
2.4
A 35 year old lady presents to her GP saying that she has carried out a home pregnancy
testing kit and it has revealed a positive result.
She wants to confirm whether or not
she is pregnant and how far she has progressed through her pregnancy.
Name five likely symptoms of pregnancy noticed in the first trimester of pregnancy, that
this lady may mention.
Morning sickness.
Fatigue.
Frequency of micturition.
Heartburn.
Constipation.
Breast changes.
Emotional lability.
This lady tells you that the date of her last menstrual period (LMP) was on the 7 th
September 2005.
Calculate using Naegle’s Rule the estimated date of delivery (EDD).
Estimated date of delivery (EDD) = Last menstrual period (LMP) + 9
mnths 7 dys

= 7th September
+ 9 mnths 7
dys

= 14th September 2005 + 9 mnths

= 14th June 2006
Give one reason directly associated with the menstrual cycle, that could affect the
accuracy of this calculated EDD.
Some women have longer than average length of menstrual cycles.
Variation in the length of the proliferative phase of the menstrual cycle.
List three other confounding factors could affect EDD accuracy.
Assumption of 40 weeks gestation (280 days).
Assumption of conception occurring two weeks after first day of LMP
(14 days into cycle ovulation occurs, assumption of conception
40
occurring on this day, but could have occurred any time between then
and next date of expected menstrual period).
Break through bleeding could confuse the date of LMP.
Bleeding from recent IUCD use could confuse date of LMP.
Break through bleeding and irregularity of cycles after cessation of
oral contraceptive pill could confuse date of LMP.
Describe the basis of a standard pregnancy test.
B human chorionic gonadotrophin (B hCG) secreted from placenta
and trophoblastic tissue is excreted in the maternal urine.
This is
detected by monoclonal antibodies to it, tagged to latex or red cells.
A visible colour change in the detection probe is noticed by this
reaction which confirms the pregnancy.
What visible changes may be noticed on speculum examination in a pregnant woman?
Bluish tinge to the vagina and cervix.
Give three ultrasound parameters that may be noted with respect to the foetus, to verify
pregnancy dates.
Presence and size of gestation sac.
Foetal crown-rump length.
Foetal bipareital diameter.
Foetal head circumference.
Foetal abdominal circumference.
Foetal femur length.
List five potential risk factors for abnormality in pregnancy.
Maternal age.
Previous miscarriage.
Consanguineous marriage between parents.
Alcohol abuse.
Drug abuse.
Teratogen ingestion.
Maternal illness (including infections such as HIV).
41
Poor folic acid intake.
There is concern from an ultrasound scan that this woman’s child may have Down’s
Syndrome.
A “triple test” is carried out on maternal serum.
Name the three factors
tested, and indicate whether these would be high or low if the foetus does have Down’s
Syndrome.
Human chorionic gonadotrophin levels raised.
Decreased alpha fetoprotein levels.
Decreased unconjugated oestradiol levels.
Name and give a brief description of three investigations used to detect foetal
abnormalities (other than the “triple test” and ultrasound scanning).
1 Amniocentesis – fine needle used to aspirate amniotic fluid from the
mother’s uterus, under ultrasound real time guidance.
Cells shed
from foetal gut and skin can then be used for further tests such as
karyotyping.
2 Chorionic villus sampling – sample of actively dividing trophoblast
cells allows rapid karyotyping.
Done in 1 st
trimester by
transabdominal/transcervical route.
3 Placental biopsy – sample of actively dividing trophoblast cells
allows
rapid
karyotyping.
Done
in
2nd/3rd
trimester
by
transabdominal route.
4 Foetal Tissue Sampling – done under real time ultrasound
guidance, involving skin or liver or other organ tissue extraction
depending on indication..
5 Antenatal foetal blood sampling – done under USS guidance
transabdominally (placental cord, foetal intrahepatic vessels, foetal
heart).
Name a foetal abnormality that could be responsible for raised alpha fetoprotein levels
in maternal serum.
42
Neural tube defect (anencephaly, spina bifida).
Give three other reasons for raised alpha fetoprotein in maternal serum.
Exomphalos.
Gastroschisis.
Polycystic kidneys.
Foetal death (tissue autolysis).
Foetal teratoma (AFP secreted from tumour).
Duodenal/oesophageal atresia.
A younger pregnant woman mentions a long term problem with acne vulgaris, for
which she had roaccutane treatment.
What sort of drug is this and why is this of
relevance with respect to her unborn child?
It is an isotretinoin medication derived from vitamin A and is highly
teratogenic.
2.4
Ante natal care
By the end of Phase II , students should be able to:
•
assess pregnancy by a detailed obstetric and general medical history
•
calculate the expected date of delivery and allow for the possible confounding effects of
irregular menstruation and the effects of the oral contraception
•
identify normal pregnancy
•
identify the possibility of abnormality in pregnancy
43
2.5
A PRHO clerks a 16 year old girl who presented to the A&E department with acute
abdominal pain.
The girl gave a history of generalised lower abdominal pain for 3 days, worsening over
the past 24 hours and being more localised over right iliac fossa.
On examination she has rebound abdominal tenderness in right iliac fossa.
1) Give four likely differential diagnoses.
Gastrointestinal – appendicitis, mesenteric adenitis, Meckel’s diverticulum,
inflammatory bowel disease, gastroenteritis, peritonitis.
Reproductive – Ectopic pregnancy, salpigitis/pelvic inflammatory disease,
endometriosis, mittelschmerz.
Renal – UTI, ureteric colic(stones).
Pain from adjacent areas
2) The PRHO then performs a pregnancy test:
a) What hormone is measured to test for pregnancy?
Beta - Human chorionic gonadotrophin.
b) Specifically which cells secrete it?
Trophoblast cells.
c) When is it first measurable in the urine?
2 weeks after implantation.
3) The hormone level is >1000 IU/L and the uterus is empty on ultrasound scan.
Give two possible explanations for this and underline the most serious one.
Early intrauterine pregnancy (<5weeks).
Ectopic pregnancy.
44
4) With regards to this serious problem, what factors may predispose to this? Give
two.
Previous tubal surgery.
Fertility treatment (IVF, GIFT).
Previous Ectopic pregnancy.
Congenital abnormalities (diverticula).
IUCD.
Progesterone only pill.
Emergency contraceptive.
5) Give one medical and one surgical treatment options.
Medical –methotrexate either systemically or direct injection into fallopian tube
under USS guidance.
Surgical – laparoscopy and salpingectomy or salpingostomy.
6) How would you follow this patient up?
Beta-HCG and TV USS every 1-2 days to ensure termination.
Give all rhesus negative women anti-D.
7) In a 16 year old girl specifically, what advise would you give regarding
contraception?
2.5
Major problems of pregnancy
By the end of Phase II , students should be able to:
•
recognise, and initiate management of
•
spontaneous miscarriage
•
ectopic pregnancy
•
trophoblastic disease
•
abruptio placentae
•
placenta praeva
•
unclassified antepartal haemorrhage
•
recognise pre-eclampsia
45
•
initiate investigation of gestational hypertension
•
explain the consequences of pre-eclampsia
•
outline to a patient the management of pre-eclampsia and gestational hypertension
46
2.6
You are a GP at your local practice. An extremely anxious 40-year-old woman comes to
see you after reading a news paper article about the risks of Downs Syndrome in older
women. You know this patient well. You diagnosed her pregnancy last month after she
missed her last two periods. The pregnancy test was positive and her last menstrual
period (LMP) was 10 weeks ago. You and the patient discuss prenatal screening with
particular reference to Downs Syndrome.
What investigation can reveal the gestational age of the fetus more reliably than a
history of her LMP?
Ultrasound
What four main types of prenatal screening are available to detect Downs Syndrome?
Maternal serum screening (blood) or Triple Test.
Ultrasound
Amniocentesis
Chronic Villus Sampling
Which three markers are assessed by the Triple Test.
AFP (alpha fetoprotein)
Oestriol
HCG.
Explain why the cut off for high vs low risk of Downs Syndrome via the Triple Test is set
at 1 in 250?
If the risk is 1 in 250 or less, the risk of having a baby with Down syndrome is greater
than the risk of having a miscarriage from the amniocentesis.
If the risk is more than 1 in 250, the risk of having a miscarriage from amniocentesis is
greater than the risk of having a baby with Down syndrome.
47
What abnormalities may be found on Ultrasound to indicate Downs Syndrome? List
3.
Any three of….
Nuchal translucency
Choroid plexus cyst
Echogenic bowel
Echogenic intracardiac focus
Dilatation of kidneys
The fetus was determined to be high risk of Downs Syndrome by Triple Test. What two
further investigations can give a more reliable assessment and at what gestational age
can they be performed? Note: They both allow fetal chromosomal analysis.
Amniocentesis at 14-18 weeks, and Chronic Villus Sampling at 10-12 weeks.
List 5 side effects/ risks for these investigations?
Cramps
Bleeding
Infection
Leak amniotic fluid
Increased risk of misscariage.
This patient asks you if medication can be harmful during pregnancy. Below are six
common drugs. List one major side effect to the fetus in each.
Warfarin:
Neonatal haemorrhage
Tetracyclines:
Abnormal bone growth or Teeth discoloration
ACE inhibitors:
Renal damage
Benzodiazepines:
NSAID:
Phenytoin:
Amiodarone:
Neonatal withdrawal
Delayed closure of ductus arterious
Cleft palate or cardiac abnormalities
Neonatal goitre.
48
2.6 Fetal growth, development and disease
By the end of Phase II , students should be able to:
• recognise and interpret discrepancy in uterine size for gestational age
• outline to patients the role of blood tests, ultrasound amniocentesis and chorionic villus
sampling in the establishment of fetal health
• describe the role and methods of fetal assessment in pregnancy
• recognise the potential adverse effects of drugs in pregnancy
• debate the benefits and costs of prenatal screening
49
2.7
A 32-year-old multiparous woman presents at 10 weeks’ gestation to the antenatal
clinic. She has a blood pressure of 140/90 mmHg.
1.)
Name 3 causes for this BP
(3 marks)
Essential hypertension
Chronic hypertension
Secondary hypertension
2.)
What examinations should be performed?
(2 marks)
Repeat BP measurements
Fundoscopy – assesses severity of hypertension
3.)
What investigations should be performed?
(4 marks)
FBCs
U+Es
LFTs
24-hour urinary protein concentration
4.)
How should this condition be managed?
(2 marks)
Treatment should be aimed at controlling the BP. Drugs that have been used
include methyldopa, labetolol and nifedipine. These drugs can be used as
single agents or in combination. (Important to remember that labetolol and
nifedipine have been known to cause fetal and neonatal bradycardias.
Methyldopa may cause abnormal LFTs and depression).
50
A 26-year-old woman who is now 37 weeks pregnant has been attending the antenatal
clinic since she was 14 weeks pregnant. A routine FBC performed by the midwife at 36
weeks shows the following abnormal results:
Hb = 8.2 g/dl
MCV = 68.4 fl
WBC = 9.0 x 109
1.)
What is the diagnosis?
(1 mark)
Microcytic anaemia
2.)
What are the reasons for this diagnosis?
(2 marks)
Low Hb, low MCV
3.)
What other symptoms may she present with?
(3 marks)
Breathlessness
easy fatiguability
asymptomatic
palpitations
tinnitus
symptoms from underlying cause. e.g. GI bleed- malaena, rectal bleed
4.)
What further investigation will you perform on her?
(2 marks)
Serum ferritin
5.)
Assume she has iron deficient anaemia what 2 treatment options may you
recommend for her at this stage?
(2 marks)
Parenteral iron, blood transfusion
A glucose tolerance test was performed on a pregnant woman who previously had a
stillbirth at 39 weeks. The results are shown below:
51
Fasting = 5.5 mmol/l
60 minutes = 12.4 mmol/l
120 minutes = 10.5 mmol/l
1.)
What is the diagnosis?
(1 mark)
Impaired glucose tolerance
2.)
What treatment will you recommend for her? (2 marks)
Dietary control in the first instance, if unsuccessful, insulin should be started
3.)
What are the complications of the condition during pregnancy?
(3 marks)
Fetal macrosomia, unexplained intrauterine death, polyhydramnios
4.)
When is the baby most likely to be delivered?
(1 mark)
At 38 weeks gestation
5.)
If the mother is placed on insulin, what must be done after delivery?
(1 mark)
Stop the insulin and return her to her pre-pregnancy status; without any
treatment)
6.)
What advice will you give this woman about the long-term implications of this
condition? (2 marks)
She has a higher risk of developing adult-onset diabetes mellitus and
therefore will require frequent screening
Mrs PB attended for routine antenatal care at 28 weeks gestation when she was found
to have a + protein in her urine. She had no symptoms. An MSU grew E. Coli for which
she was treated successfully. On 2 subsequent visits she was found to have proteinuria
and an MSU showed significant showed significant bacterial growth but she remained
asymptomatic.
52
1.)
What is this condition called?
(2 marks)
Asymptomatic significant bacteruria
2.)
What is the sequelae if this is untreated?
(1 mark)
Symptomatic UTI
3.)
What 2 causes of this condition need to be excluded? (2 marks)
Urinary tract abnormalities
kidney stones
pyelonephritis
4.)
What treatment would this patient benefit from?
(2 mark)
Antibiotics with either trimethoprim, ampicillin or nitrofurantoins
5.)
What further investigations need to be performed on her? (2 marks)
Ultrasound scan of kidneys, intravenous urogram 3-4 months after
delivery
2.7
Medical problems in pregnancy
By the end of Phase II , students should be able to:
• initiate management of the common self-limiting problems in pregnancy
• recognise the potential adverse effects of intercurrent disorders such as anaemia, asthma,
cardiac disease, diabetes, hypertension, renal and thyroid disease
• initiate appropriate investigations for these disorders
• appreciate the risk of thromboembolic disease in pregnancy
• participate in the long-term management of these conditions
53
2.8
Mrs G has conceived after her third IVF attempt. She has just had an USS at 6 weeks
and has been told she has triplets.
a. What three possible first trimester complications may she have?
Miscarriage
Hyperemesis gravidarum
Vanishing triplet syndrome.
b. What prophylactic drugs must she have during pregnancy? Name 2.
Iron tablets
folic acid.
c. What would be the most common complication in the second half of
pregnancy?
Preterm labour
d. What three other maternal complications may arise?
Anaemia
Hypertension
Preeclapsia
pressure symptoms.
e. What precautions might be taken to reduce risk and complications of
prematurity?
Admission into hospital for bed rest.
Antenatal steroids to reduce the severity of respiratory distress syndrome in the
babies.
f.
How may the babies be delivered?
The most common way is by C-section but vaginal delivery is possible.
54
2.8
Multiple pregnancy
By the end of Phase II , students should be able to:
•
recognise the likelihood and make the diagnosis of multiple pregnancy
•
recognise the complications of multiple pregnancy during pregnancy, parturition and the
puerperium
•
outline to patients the possibility of preterm labour and the early symptoms
55
2.9
A woman was admitted to the delivery suite at 38 weeks gestation with absent fetal
movements for 24 hours and fetal heart was not picked up on CTG.
a) How would you confirm the diagnosis of intrauterine fetal death? (1 mark)
Ultrasound scan
b) How would you counsel the mother? (3 marks)
Explain the diagnosis to the mother in an empathic way
explain need to deliver the fetus
explain need to investigate the cause of intrauterine death
c) Name 4 investigations you would perform on the mother. (4 marks)
Coagulation profile
thyroid function test
HbA1c
Kleihauer test
lupus anticoagulant and anticardiolipin antibodies
infection screen
d) Name 2 investigations you would perform on the fetus. (2 marks)
Swabs from nose, throat ear and mouth
Skin biopsy for karyotype
Autopsy
2.9
Intrauterine and neonatal deaths
By the end of Phase II , students should be able to:
• recognise the antecedents and methods of prevention of intrauterine and neonatal death
56
• recognise on clinical grounds the possibility of intrauterine death and initiate appropriate
investigations
• communicate with patients in a role play situation about the diagnosis of intrauterine death
• participate as a team member in the counselling of parents sustaining an intrauterine or
neonatal death
57
2.10
A gravida 0, aged 37 women, comes to see you for advice as she is planning to start a
family.
She has heard that there are risks associated with starting a family later in life.
a) List 2 maternal risks of elderly primigravida. (2 marks)
Any of:
Increased risk of ectopic pregnancy
Increased spontaneous loss
Molar pregnancy
Thromboembolism
Mortality.
b) Give 2 obstetric risks associated with increased age at first delivery. (2 marks)
2 of:
abruptio placenta (HTN, fribroids)
placenta previa
increased rate of c-section
uterine infection (postpartum endometriosis).
c) What are the foetal risks? Give 2. (2 marks)
Increased incidence of twinning
Iatrogenic preterm delivery
Aneuploidy
Non-chromosomal anomalies/ autosomal dominant mutations (due to
increased paternal age).
d) What is the aim of prenatal diagnosis? (2 marks)
Identification of abnormalities to prepare the parents
Offer termination
Identify conditions which may influence the timing of delivery
Enable adequate paediatric support to be available
Identify foetuses that may benefit from in utero treatment.
e) Name 2 prenatal tests and what they are used for. (2 marks)
Maternal serum -feto protein – neural tube defects
Triple test (HCG, MSAFP, oestriol) – aneuploidy
58
Ultrasound scan – soft markers (nuchal translucenecy in Down’s,
echogenic bowel in CF), structural abnormalities (congenital heart
disease, abdominal wall defects).
Amniocentesis: tissue sampling and karyotyping
Chorionic villus sampling: tissue sampling and karyotyping.
A 13 year old girl comes to see you with a 8 week history of amenorrhoea.
f)
What are the possible diagnoses?
Pregnancy
Emotional disturbances (stress, exams)
Weight loss/athleticism
PCOS
Drugs (OCP)
Hyper/hypothyroidism
Hyperadrenalism
g) What first line investigation would you do?
Pregnancy test
h) What are the issues associated with an adolescent pregnancy? (2 marks)
Lack of concordance with prenatal care.
Nutritional issues –
growth demands of adolescence

Vitamin deficiency

Iron deficiency (poor diet, heavy erratic periods)

Alcohol and drugs

Body image and dieting
Infections - UTI

STD
Other - lack of social support (partner, home, school)

Infant disadvantage.
A 34-year-old lady comes to see you as she has a positive pregnancy test.
She has 7
children.
i)
Define grand multiparity. (1 mark)
59
5 or more pregnancies carried to viability.
j)
What risks are associated with this?
Abruptio placenta (due to age/HTN)
Placenta previa (due to previous scarring)
Post partum haemorrhage (due to atony and therefore lack of
contraction)
Precipitate delivery
k) What social issues need to be considered?
Childcare during and after pregnancy
Socio-economic factors
Contraceptive advice.
Mrs J, a 25 year old G2 P0 is referred to you in the antenatal clinic.
She is a known to
have a problem with alcohol abuse and her midwife is concerned that she is continuing
to drink throughout her pregnancy.
l)
List 3 facial features of Foetal Alcohol syndrome.
Absent/hypoplastic philtrum
Broad upper lip
Flattened nasal bridge
Micrognathia
Micropthalmia
Short nose
Short palpebral fissures
m) List 5 other features of Foetal Alcohol Syndrome.
Hyperactivity/irritability
Developmental delay
Growth deficiency
Mental retardation
Poor coordination
Congenital heart and joint defects
n) Give one other effect of drinking during pregnancy.
Increased spontaneous abortion
60
Restricted foetal growth
Preterm birth
Ms H G1 P0 is pregnant.
She works as a nurse and had a needle-stick injury 1 year ago.
Having never been vaccinated, she contracted acute hepatitis B and has recently had a
blood test showing presence of Hepatitis B E antigen.
o) Is her foetus at risk of vertical transmission?
Yes (up to 90% if HBeAg present)
p) By what routes may Hep B be transmitted to Ms H’s baby?
Intrapartum exposure
Haematogenous
Transplacental
Breastfeeding.
q) What management plan would you follow?
Advise to abstain from alcohol/hepatotoxins
Notify staff on labour ward
Ensure baby receives Hep B immunoglobulin and vaccine
LFT’s
?amniocentesis
?delivery route
discuss breastfeeding
Mrs K is HIV positive and pregnant with her first child.
r)
What maternal factors affect vertical transmission?
Advanced maternal disease
Low CD4 count
High viral load
Genital tract HIV viral load
Genital tract infections
s) What obstetric factors effect transmission?
Mode of delivery (reduced in c-section)
61
Duration of membrane rupture
Prematurity
t)
What interventions would you recommend?
Antiretroviral therapy
Elective c-section
Formula feeding
u) What antiretroviral therapy would you prescribe?
Triple therapy – zidovudine, lamivudine and nevirapine
v) How would you manage the neonate?
Treat with antiretroviral therapy (depending on mothers viral load)
Diagnostic testing: HIV PCR at day 1, 1 month, 3 months, 6 months,
12 mnth

HIV antibody at 18 months
BCG not given until after 18 months.
2.10
Special situations
By the end of Phase II , students should be able to:
•
recognise that special obstetric risks are associated with grand multi-parity, elderly
primigravidae, adolescent pregnancies and pregnancies in drug and alcohol abusers
•
recognise the particular risks to patients and staff in circumstances of Hepatitis B
and HIV infection
62
2.11
Ms K is a 25 year old primigravida of 29 weeks gestation who presented with an
abdominal ‘tightening’ sensation which were becoming more painful and more
frequently in nature.
Define premature labour.
Premature labour is defined as labour occurring after 24 weeks and before 37 weeks
gestation.
Describe five characteristics of a woman who is at increased risk of developing
premature labour.
Body mass index <19
Low social class
Extremes of reproductive age (under 20 and over 35)
Domestic violence
Previous preterm labour
Others to mention – unsupported, smoking, previous preterm labour, bacterial
vaginosis and chronic medical conditions.
Name five possible causes of Ms K’s premature labour
Infection eg. Chorioamnionitis, maternal pyelonephritis, appendicits.
Uteroplacental ischaemia eg. Abruption
Uterine overdistension eg. Polyhydramnios, multiple pregnancy.
Cervical incompetence
Foetal abnormaility
Upon initial examination of Ms K in the EAU department describe what you would do.
Assess mother to see if she is stable (? shocked from a large abruption)
Assess for signs of a precipitant of preterm labour e.g. sepsis, polyhydramnios, and
severe pre-eclampsia, obstetric cholestasis.
63
Determine frequency and regularity of contractions
Sterile speculum examination – check for dilatation of the cervix
Begin foetal monitoring – CTG
Ascertain foetal presentation (cephalic/breech)
Inform paediatricians
Ultrasound examination – abnormal lie and presentation common in preterm labour.
What investigations would you carry out?
Blood tests
MSU and urinalysis
High vaginal and endocervical swabs
Assess for liquor using nitrazine sticks (PH – sensitive) – turn black, if it is not obvious.
False positives from infected vaginal discharge, semen, blood and urine
It was discovered that Ms K membranes were not ruptured, cervix was approx 1 cm
dilated. There was no obvious precipitant or cause. What treatment should be initiated
at this stage?
Corticosteriods – betametasone or dexamethasone 12mg/12h IM twice promotes
foetal surfactant production, lowering mortality and complications of Respiratory
Distress syndrome. Use before 34 weeks only.
Antibiotics – to be used if membranes rupture or obvious infection
Consider tocolysis
Monitor maternal pulse, BP, temperature, contractions and foetal heart rate.
Tocolytic therapy was initiated. Name two known drugs and describe its mode of action.
Ritodrine/salbutomol – beta agonist, act on beta – receptors in myometrium to cause
relaxation.
Nifedipine – calcium channel blocker – block calcium channels in the myometrium,
interrupting contraction.
64
Mention a side effect of each named drug.
Ritodrine – pulmonary oedema, tachycardia, hyperglycaemia and hypokalaemia.
Nifedipine – potential to alter uteroplacental bloodflow causing foetal compromise.
Maternal side-effects headache, flushing and tremor.
2.11
Preterm labour
By the end of Phase II , students should be able to:
•
recognise the circumstances when preterm labour may occur
•
outline management to the patient
65
2.12
Mandy, aged 27, is expecting her first baby. No known complications are identified.
Early in her pregnancy, she discusses the delivery options with her midwife. Initially
Mandy is in favour of a home birth. But she later changes her mind, choosing to deliver
the baby in hospital.
A) Suggest two reasons why Mandy may have decided to opt for a hospital based birth.
(any 2 from below)
Primigravid women are more likely to experience failure to progress in the first or
second stages of labour than multigravid women. This would require hospital transfer.
Regional anaesthesia,
Resuscitation of the newborn,
Management of postpartum complication are all more difficult at a home delivery,
compared with a hospital delivery.
In her 39th week of pregnancy Mandy attends the maternity unit. She is experiencing
contractions at a rate of one every 5-10 minutes and believes that she is in labour.
B) List 3 symptoms may Mandy have experienced, which herald the onset of labour?
Regular uterine contractions.
Rupture of membranes.
Show.
C) What is the definition of labour?
The occurrence of regular uterine contractions accompanied by cervical dilatation.
The midwife confirms that Mandy has started the first stage of labour.
D) Define the three stages of labour.
66
First stage: From the onset of labour until the cervix is fully dilated.
Second stage: From full cervical dilation to birth of the baby.
Third stage: From birth of the baby to delivery of the placenta.
The midwife initiates monitoring of the progress of Mandy’s labour using a partogram.
E) What four factors are recorded on a partogram?
Cervical effacement.
Cervical dilatation.
Descent of presenting part.
Uterine contractions
The wellbeing of Mandy and her foetus are also monitored.
F) List two parameters used to assess maternal wellbeing during labour, and two to
assess foetal wellbeing.
Maternal (any 2 of below):
Blood pressure
Pulse
Temperature
Pain relief
Fluid balance
Foetal (any 2 of below):
Foetal heart rate
Colour of liquor
Foetal scalp pH
A short cardiotocograph recording is taken on admission.
G) What parameters should be used to interpret a cardiotocograph recording?
Remember: “BRAVADO”
67
BR
The baseline rate.
A Acceleration.
VA
Variability.
D Decelerations.
Overall impression.
F) Indicate two findings on a cardiotocograph that may indicate foetal distress.
(any 2 of following)
The baseline rate <110 or >160 beats/minute.
Accelerations absent for over 20 minutes.
Variability <5 beats/minute.
Decelerations not associated with contractions.
Overall impression non-reassuring.
F) List 4 indications for continuous cardiotocograph monitoring.
(any 4 of following)
Preterm infant (<37 weeks completed gestation).
Foetuses that are or are suspected to be small for gestational age.
Multiple pregnancies.
Women with epidural analgesia.
Women with Syntocinon augmentation of labour.
Women who have been induced.
Women who are hypertensive.
Women with major medical disorders, including diabetes.
Women who develop meconium staining of the amniotic fluid during labour.
Women who undergo a trial of uterine scar.
If a foetal heart abnormality is recorded with the Pinard stethoscope/sonicaid.
G) Which drug is routinely given during for the third stage of labour?
When should it be administered?
Syndometrine.
Administered with the delivery of the anterior shoulder.
68
2.12
Normal labour
By the end of Phase II , students should be able to:
• describe the environments in which normal labour may be conducted and their relative
benefits and risks
• acknowledge the wishes of the mother in this decision
• recognise the onset of labour
• manage normal labour under supervision
• identify the drugs commonly used in labour and their benefits and risks
• participate in fetal surveillance in labour
• recognise the common signs of fetal distress
69
2.14
Louise, a 25 year old woman, has successfully delivered her first baby, a healthy male,
by normal full term delivery.
Normal third stage has completed, though she needed
repair of a third degree tear.
She is wishing to breast feed but is experiencing some
difficulty.
Define the puerperium
6 week period from the completion of the 3rd stage of labour
What physiological changes take place during the puerperium?
e.g.s
Genital tract:
Uterine involution
Lochia
Endometrium regeneration and return of menstruation
CVS:
Fall in plasma volume and cardiac output
Return of BP to normal (if raised in pregnancy)
Blood:
Increased coagulability due to rise in platelets and clotting factors
U+E return to normal
Rise in HB and haematocrit with haemoconcentration
Urinary tract:
Reduction of physiological dilatation GFR fall to normal
Reduced bladder sensitivity to over-distension
List benefits of breast feeding
To mother
To baby
(from Lifespan;)
Benefits to mother:
70
post-partum recovery- uterine involution, less weight gained, figure
and physical fitness
psychological- bonding.
Measure of success by society
Long-term health- lower risk osteoporosis, ovarian ca
post-menopausal, breast cancer
Benefits to baby:
Species specificity
Nutritional benefits
Emotional- bonding
Enhanced immunity- less infection
Lower risk cot death
Lower risk allergies
Coats gut- lower risk gastroenteritis
Cost
For what maternal related reasons may breast feeding be difficult?
Mastitis
Abscess formation
Postnatal depression/ psychosis
Multiple births
Exhaustion
What advise would you give to a mother such as Louise who is learning for the first
time to establish successful lactation?
Louise asks you about postnatal contraception.
She has used the OCP previously, but
has heard that some drugs should not be given whilst breastfeeding, and wonders if
breastfeeding alone will be enough to protect her.
What contraceptive advice would you give her?
What are her options and when
should any measures be started?
Lactation not adequate alone
Start contraception 4-6 wks post-delivery
OCP contraindicated if breastfeeding- suppresses lactation
Progesterone-only pill or depot is safe when breastfeeding
IUD safe- insert at end of 3rd stage or at 6 weeks (infection screen first).
71
drugs that should be avoided during breastfeeding, and their potential
effect on the baby
With your help, Louise successfully starts breastfeeding.
She is managed on the ward
for a few days.
What routine assessments should be made daily postnatally?
What common
problems in the puerperium should be identified?
Assess:
Uterine involution and lochia
Blood pressure
Temperature
Pulse
Perineal wound
Common problems:
Infection
Thrombo-embolism
Poor wound healing
Urinary incontinence
Urinary retention
Faecal incontinence
Feeding problems
Post-natal depression and psychosis
Louise develops pyrexia.
What are the most likely sources/causes?
How would you assess and
manage it?
Sources/causes:
Genital tract infection- endometritis (± retained products of
conception)
Urinary tract infection
Breast- Mastitis
72
Wound infection
Lung infection
DVT
Examine- abdo, breasts, IV access sites, chest, legs
Cultures- Blood, urine, high vaginal, fetal
Broad spectrum antibiotics
Louise returns home with her new baby.
A few days later, she is visited by her
midwife.
What symptoms might suggest post-natal depression?
What are the risk
factors?
Tiredness
Guilt
Feelings of worthlessness
RF:
Social/emotional isolation
Previous history
Pregnancy complications
What measures might you take in treatment
Social support
Psychotherapy
Antidepressants
2.14
Normal puerperium and lactation
By the end of Phase II , students should be able to:
•
make routine assessments of patients to identify infection, thrombosis and feeding
problems and assess wound healing
•
recognise the factors leading to the establishment of successful lactation
73
•
advise patients on the use of drugs in lactation
•
recognise the possibility of post-natal depression and psychosis
74
2.15
Mrs S is a 35 year old primigravida at 40+10 weeks gestation.
She has been admitted
to the labour ward for induction of labor.
1) What is the rationale for induction at this timing?
Due to increased perinatal mortality rates/Caesarean section after 42 weeks.
1 mark
2) Give 3 circumstances where induction of labor would be contraindicated?
Previous caesarean section,
Multiple Pregnancy,
Maternal Cardiac disease,
Pre-eclampsia.
3 marks
The progress of labor was monitored using the Bishops score.
3) Name 4 of the criteria examined for a bishops score.
Dilation,
Consistency,
Length,
Station,
Position.
4 marks
Mrs S progressed through to the 2nd stage of labor.
Her blood pressure (110/70), pulse
(75bpm), and temperature (37.1ºC) were all monitored.
minutes.
Contractions were 3 in every 10
During her contractions, fetal heart rate was monitored by CTG, and fetal
distress was noted.
4) List 3 signs of fetal distress that can be observed on a CTG.
Unprovoked decelerations,
75
Decreased variability,
Fetal bradycardia(<110bpm)/tachycardia(>175bpm),
Slow recovery of fetal heart rate following a deceleration.
3 marks
Due to the fetal distress and a prolonged 2 nd stage, assisted ventouse delivery was
attempted.
5) Give two complications of a ventouse delivery, and 2 complications of a forceps
delivery.
Ventouse:
Cephalo/skull haematoma, Scalp laceration, Vaginal tears, Neonatal jaundice.
Forceps:
Vaginal tears, Ruptured uterus, Facial nerve palsy.
4 marks
Following successful delivery, Mrs S was noted to have lost 500ml of blood and was
continuing to lose blood.
6) Give 3 causes of post partum haemorrhage
Caesarean section,
Atonic uterus,
Retained placenta,
Trauma during delivery.
3 marks
7) What drug is commonly used at the end of the 2nd stage of labor to aid the progress of
the 3rd stage, and what is it a combination of?
Syntometrine, combination of Ergometrine and Oxytocin.
2 marks
76
2.15
Abnormal labour
By the end of Phase II , students should be able to:
•
describe the role of induction of labour and the possible methods
•
recognise common fetal and maternal problems in the first stage of labour
•
participate in the management of delay in the first stage of labour including the use of
oxytocin and monitoring techniques
•
recognise delay in the second stage of labour and the circumstances when it may occur
•
recognise the common complications of the third stage of labour and describe how they
may be prevented
•
recognise and participate in the immediate management of post-partum haemorrhage
•
discuss with patients the need for operative delivery including Caesarean Section
•
debate critically the role of Caesarean Section
77
3.2
Mr Phillips, a 68 year old retired gardener comes to see you his GP complaining of
difficulty passing water which has been blood stained.
After skilfully eliciting the history, you suspect the prostate to be the cause of Mr
Phillips’ troubles
a) Apart from examining Mr Phillips’ abdomen, which examination is indicated here?
Digital rectal examination
b) List four other indications of when to perform the examination mentioned in a.
Suspected lower GI
prostate
cauda equina/ back injury
Rectal bleeding
Severe constipation
Pain in the anal or rectal area
Suspected enlarged prostate gland
Suspected rectocele.
c)
List four things you would normally inspect for whilst performing this examination
(in any case, not just for Mr Phillips).
Rectal prolapse
fistulo-in-ano
skin tags
anal fissure
anal warts
haemorrhoids
anal carcinoma
pruritus ani
excoriation from diarrhoea
78
d) Give two distinguishing features for each of prostatitis, prostatic carcinoma and BPH
upon palpation of the prostate gland.
Hard and nodular in prostatic carcinoma
boggy and tender in prostatitis
smooth and diffusely enlarged in BPH
e) Assessment of anal tone is an important aspect of digital rectal examination. Which
type of muscle comprise the internal and external sphincters of the anus and what is
their respective innervation?
Internal is involuntary smooth muscle, sympathetic innervation via the hypogastric
and pelvic plexus
External is striated voluntary muscle. The proximal aspect is in intimate relationship
with the puborectalis muscle. The pudendal nerve, specifically the inferior rectal
branch, innervates the external anal sphincter.
Question 2
Mrs Wilson is a 56 year old school teacher. She has two children and is overweight with
a BMI of 28.
She has come to see you the HO in A&E with a 3 month history of
RUQ/epigastric pain, colicky in nature and is occasionally relieved by food. She also
has a previous history of a peptic ulcer.
a) List two likely differential diagnoses that fit with Mrs Wilson presenting complaints.
Chronic cholecystitis
Biliary colic
Duodenal/gastric ulcer
GORD
b) Name two treatments for Mrs Wilsons’ symptoms?
79
Lifestyle and diet modification – avoid spicy food, stop smoking,

weight loss
Simple antacids - gaviscon
Proton pump inhibitor – omeprazole
H2 Receptor antagonists – ranitidine
c) List two further investigations required at this stage to identify the cause for Mrs
Wilsons problems.
H-pylori testing: CLO test or Urea Breath test
Upper GI endoscopy
Ultrasound scan abdomen
d) List 4 indications for upper GI endoscopy apart from weight loss and age.
Symptoms > 4 weeks
Dysphagia/odynophagia
Persisting symptoms despite treatment
Relapsing symptoms
Persisting vomiting
GI bleeding
e) Outline 3 of the potential complications of upper GI endoscopy
Problems related to sedation/medication
Bleeding
Perforation
f) Following the upper GI endoscopy, Mrs Wilson became unwell with abdominal pain
made worse by coughing.
What does her chest x-ray show and what is the
commonest cause of this condition.
80
X-ray shows pneumoperitoneum (gas shown under both sides
diaphragm)
Commonest cause is from perforated gastroduodenal ulcer.
81
Question 3
Mr Hughes, a 34 year old stock broker, attends A&E with a 11 day history of yellowing of
his skin and abdominal pain. He has noticed darkening of his urine and his motions
have become putty coloured in the last three days.
a) Give four likely differential diagnoses for this patient?
Pancreatitis
Alcoholic liver disease
Acute hepatitis
Choledocholithiasis (gallstone in CBD)
Drugs (paracetamol)
Ascending cholangitis
His liver function blood results are as follows
LFT:
Bilrirubin (conjugated) – 68
(3-17mol/L)
Alkaline Phosphatase – 300
(40-120u/L)
ALT – 40
(3-35u/L)
b) What do these blood results imply is the likely pathological process? And what other
biochemical parameters would you like to know?
Diagnosis - Obstructive/cholestatic jaundice
Biochemical parameters - Gamma GT & Serum Amylase
c) State one further investigation you would arrange to determine the cause.
Abdominal ultrasound scan
ERCP
d) Mr Hughes has an ERCP arranged by your registrar.
Give two indications for this
test and state three complications that can occur.
82
Indications (diagnostic)
Cholangitis
Jaundice with intra hepatic duct dilatation
Jaundice with normal ducts and a non diagnostic biopsy
Recurrent pancreatitis
Post cholecystectomy pain
Indications (therapeutic)
Sphincterotomy for CBD stones
Stenting for malignant strictures
Complications
Acute pancreatitis
Bleeding
Cholangitis
Perforation
Extended Question
Briefly outline the normal metabolism of bilirubin.
Bilirubin is breakdown product of haemoglobin.
It is conjugated with glucuronic acid by hepatocytes and becomes water
soluble.
The conjugated bilirubin is secreted into bile and then passes in to the gut.
Some is taken up by the liver (enterohepatic circulation) with the remainder
being converted to urobilinogen by gut bacteria.
Urobilinogen is either reabsorbed or excreted by the kidneys, or converted
stercobilin which colours faeces brown
83
3.2
Investigations
By the end of Phase II
students should be able to:
•
perform and interpret digital rectal examination
•
recognise the radiological features of intra-peritoneal air, obstructed bowel and correct
placement of naso-gastric tubes
•
explain to a patient the reasons for conduct of and possible complications of upper
gastro-intestinal endoscopy, colonoscopy ERCP and contrast radiography
•
request abdominal ultrasound and plain abdominal radiographs and chest radiographs
appropriately seeking advice where necessary
•
interpret liver function tests and routine liver serological and immunological investigations

relate the results to the underlying pathology
84
A 68 year old gentleman presents with a 3 month history of nocturia, poor stream,
increased frequency, urgency and hesitancy.
The SHO suspects Benign Prostatic Hyperplasia and feels she needs to perform a
digital rectal examination (DRE).
How would you perform a digital rectal examination whilst maintaining dignity for the
patient?
Fully explain the procedure to the patient.
Make sure you will not be interrupted
Ensure the patient is kept as comfortable and warm as possible.
Lie patient on their left side with knees pulled to chest.
Use gloves and lubricant.
Part the buttocks.
Ask the patient to take a deep breath and gently insert your finger into the anus.
Feel with finger pad and twist with finger tip.
When finished, wipe anus.
Inspect glove for faeces, blood.
You palpate the prostate gland. Typically, how would you differentiate between a patient
with BPH and a patient with Prostate Cancer?
A normal prostate is the size of a walnut.
In BPH-the gland may still feel smooth, yet is enlarged.
In prostate cancer, the gland may show asymmetrical, nodular enlargement. It may
feel characteristically stony hard and irregular with obliteration of the median sulcus.
Name 5 differential diagnoses which would require a DRE to be performed?
BPH
Prostate Cancer
Cauda Equina Syndrome- assess anal tone by asking patient to squeeze your finger.
Bowel Cancer- feel for masses and inspect glove for blood
Chronic Constipation- feel for impacted faeces and inspect glove for faeces.
.
Objective: Digital rectal examination
85
3.4
1. A 35 year old gentleman presents to his GP complaining of recent onset dyspepsia
a) Name 5 likely differentials to consider
Duodenal ulcer
Gastro-oesophageal reflux
Functional dyspepsia
MI
Gastric ulcer
Gastric malignancy
Gallstones
Gastritis
Oesophagitis
Pneumonia
b) What do you understand by the term dyspepsia?
A non-specific group of symptoms related to the upper GI tract including epigastric
pain related to hunger, eating specific foods or time of day; associated bloating;
heartburn which may be associated with acid reflux.
c) On further history taking he revealed he suffered with epigastric pain which was
worse before meals and at night and relieved by eating and drinking milk; he denied any
‘alarm’ symptoms. What is the most likely diagnosis?
Duodenal ulcer (but don’t rely on history alone)
d) What are the main risk factors involved?
Helicobacter pylori*
Smoking
Increased gastric acid secretion
Drugs (aspirin, NSAIDs, steroids)*
Blood group O
Increased gastric emptying
*Major factors
86
e) Under what circumstances is referral for urgent endoscopy necessary?
Patients of any age with dyspepsia when presenting with any of the following:
Chronic GI bleeding
Progressive unintentional weight loss
Progressive difficulties swallowing
Persistent vomiting
Iron deficiency anaemia
Epigastric mass
Suspicious barium meal
Also patients aged 55 years and over with unexplained and persistent (4-6/52) recent
onset dyspepsia alone.
f) Explain possible initial pharmacological therapies for this patient (mention 3).
Initial strategy proton pump inhibitor (PPI) full dose for a month
Testing for and treating H.pylori. H.pylori can be initially detected using either a
carbon-13 urea breath test or a stool antigen test. If positive start eradication
programme ‘triple therapy’, a 7 day twice daily course of: full dose PPI (e.g.
lansoprazole 30mg bd) + metronidazole 400mg + clarithromycin 250mg; or full dose
PPI + amoxicillin 1g and clarithromycin 500mg (Insufficient evidence to guide which
should be offered first, NICE guidelines 2004)
If no response to above try H2 receptor antagonists (ranitidine or cimetidine) or a
prokinetic;
No response to above treatments refer for 2nd opinion
g) Before beginning pharmacological therapies what initial advice may you offer?
Review medications for possible causes of dyspepsia (steroids, NSAIDs, Ca
antagonists, nitrates, theophyllines, bisphosphonates). Use of simple antacids
Stop smoking
Reduce alcohol intake
Avoid food that worsens symptoms
Weight reduction
87
h) Peptic ulcer surgery (including antrectomy with vagotomy and subtotal gastrectomy
with Roux en Y) is nowadays rarely performed due to the success of medical therapy.
However you may encounter patients who are now experiencing the long-term
complications of ulcer surgery. Briefly describe 3 of these long-term complications.
Dumping syndrome – fainting, vertigo, sweating, which may be due to the osmotic
effect of rapid transit of food from the stomach into the small intestine. Fluid is
reabsorbed into the jejunum causing temporary hypovolaemia.
Anaemia – may be due to iron deficiency because hydrochloric acid is needed for iron
absorption or due to vitamin B12 deficiency because intrinsic factor is required. Both
hydrochloric acid and intrinsic factor are absent after partial gastrectomy.
Steatorrhoea and diarrhoea
Bile reflux and vomiting
Small stomach syndrome
Recurrent ulceration and malignancy in gastric remnant
Osteomalacia
i) What do you understand by the term ‘non-dysfunctional dyspepsia’?
Functional means symptoms occur in the absence of any demonstrable abnormalities.
Functional dyspepsia can present with a range of symptoms including upper
abdominal pain/discomfort, fullness/early satiety, bloating and nausea.
2. A 67 year old gentleman presents to his GP complaining of a 3-4 month history of
epigastric pain/discomfort and unintentional weight loss.
a) What other features in the history may suggest a gastric carcinoma, mention 3?
Anorexia
Dysphagia
Upper GI bleed
Metastatic disease
Vomiting
Anaemia
88
Mass in abdomen
b) What clinical signs may be evident, name 5?
Epigastric mass
Hepatomegaly
Jaundice
Ascites
Troissier’s sign (Virchow’s node)
Acanthosis nigricans
c) What risk factors are associated with gastric carcinoma, name 5?
Diet – nitrosamines, pickling methods
Blood group A
Pernicious anaemia
Smoking
Alcohol
H.pylori infection
Atrophic gastritis
d) Name 3 types of tumour which can be found in the stomach
Adenocarcinoma (commonest)
Lymphoma
Leiomyomas
Leiomyosarcomas
neurofibromas
GI stromal tumours (GISTs)
e) Describe what you may find on macroscopic examination?
Polypoid, ulcerating or infiltrative (Borrmann classification)
If widespread can cause linitis plastica (leatherbottle stomach)
f) How and where may gastric carcinomas spread?
Direct invasion
89
Lymphatic spread – Virchow’s node
Haematogenous spread – liver, lung, brain
Transcoelomic e.g. to ovaries (Krunkenberg tumour)
g) What investigations may this patient undergo, name 5?
Urgent upper GI endoscopy – diagnostic - multiquadrant biopsy of all gastric ulcers
Bloods – FBC (anaemia), LFTs (mets)
Barium meal
CT/MRI – staging
Bone scan – staging
Endoscopic ultrasound – assess depth of gastric invasion and local lymph node
involvement
Diagnostic laparoscopy – used to exclude undiagnosed peritoneal or liver
secondaries prior to consideration of resection.
3) A 45 year old lady presents to A&E with severe right upper quadrant pain and
vomiting?
a) What differential diagnoses would you consider?
Biliary colic
Acute cholecystitis
Pancreatitis
Peptic ulcer disease
Hepatitis
Ruptured aneurysm
b) On further history taking she reveals she has experienced similar episodes of pain
over the past few months but never as severe and as prolonged as the current pain.
The pain radiates to the right tip of the scapula and is precipitated by fatty foods. On
examination the patient is lying still with shallow breathing. What other signs may you
find on examination?
Tenderness
Guarding
Murphy’s sign
90
Tachycardia
Low grade pyrexia
Mass
Jaundice (if stone moves into CBD)
c) Name the most important investigation and likely findings.
Abdominal ultrasound scan - shows gallstones, thickened gallbladder wall, checks
dilatation of biliary tree indicative of obstruction.
d) How would you manage this patient?
NBM
IV fluids
Strong analgesia e.g. pethidine
Antibiotics e.g. cefuroxime and metronidazole
Surgery – controversial issue regarding timing of surgery for patients with acute
cholecystitis. OHCM states in suitable candidates do cholecystectomy within 72h,
Early surgery is associated with fewer complications. If delayed, relapse occurs in
18%. Otherwise cholecystectomy deferred for 6-12 weeks until the inflammation has
settled.
Percutaneous cholecystostomy - elderly or high risk patients unsuitable for surgery
e) Name 5 complications of gallstones.
Biliary Colic
Acute cholecystitis
Chronic cholecystitis
Empyema
Gangrene/perforation
Cholestatic jaundice
Ascending cholangitis
Acute pancreatitis
Gallstone ileus
4) A 58 year old gentleman presents to his GP complaining of retrosternal pain
91
exacerbated by lying down and drinking alcohol and associated with regurgitation of
gastric contents.
a) Name the most likely diagnosis to consider?
Gastro-oesophageal reflux disease (GORD)
b) Name 5 risk factors associated with this disorder.
Hiatus hernia
Obesity
Pregnancy
High caffeine intake
Alcohol
Smoking
Systemic sclerosis
Increased gastric volume (large meals, delayed gastric emptying)
Drugs (TCA, anti-cholinergics, Ca channel blockers, Nitrates)
c) Briefly describe the underlying mechanisms responsible for this disorder.
Normally reflux of acid is prevented by
the lower oesophageal sphincter (LOS)
angle of His
crural fibres of the diaphragm
prominent mucosal folds
positive intra-abdominal pressure acting on the LOS
Therefore failure of one or more of these mechanisms or if hiatus hernia present
allows gastric contents to reflux into the oesophagus causing oesophagitis.
d) Briefly describe 4 complications
Oesophagitis – ulceration of oesophageal mucosa (mild to severe)
Barrett’s oesophagus – a premalignant disorder, metaplastic change from normal
squamous to columnar epithelium
Barrett’s ulcer – may bleed or perforate
Iron deficiency anaemia – blood loss from severe oesophagitis
Stricture – fibrosis may result from healing oesophagitis
92
Oesophageal malignancy – adenocarcinoma
e) Name 3 investigations and possible findings.
Upper GI endoscopy – directly visualise, biopsy and cytological brushings to confirm
presence of oesophagitis and to grade its severity and exclude malignancy.
Barium swallow – can show structural abnormalities
24h oesophageal pH monitoring – significant reflux if pH <4 for >4.7% of time.
f) Briefly describe lifestyle advice?
Weight loss
Raise bed head
Small regular meals
Avoid hot drinks, alcohol and eating <3h before bed
Avoid drugs affecting oesophageal motility (nitrates, anticholinergics, TCA) or that
damage the mucosa (NSAIDs)
g) How would you further manage this patient?
Initially try antacids or alginates.
If symptoms persist for >4wks (or weight loss; dysphagia; excessive vomiting; GI
bleeding) refer for GI endoscopy.
Oesophagitis confirmed – try PPI
Pro-kinetic drugs
Surgery – Nissen fundoplication if symptoms are very severe and there is radiological
or pH-monitoring evidence of severe reflux.
3.4
Dyspepsia
By the end of Phase II students should be able to:
• recognise and distinguish between peptic ulcer disease, gastro-oesophageal reflux
disease and functional dyspepsia clinically and on investigation including
investigation for H. pylori infection
93
• initiate management for these conditions including treatment for H. pylori infection
• communicate to a patient the diagnosis of peptic ulcer disease with an explanation
of the management including life-style changes
• recognise the possibility of gastric cancer in older patients with upper abdominal
symptoms
• recognise the possibility of gall bladder disease in patients with upper abdominal
pain and to initiate appropriate investigations
94
3.6
You are called to see a 53year old man in A & E who looks unwell with a large, distended
abdomen and jaundice. He is unkempt and smells strongly of alcohol.
a) At what plasma bilirubin level is jaundice visible? (1mark)
35mol/L
b) Give a brief description of bilirubin metabolism. (3marks)
Formed from breakdown of Hb
Conjugated with Glucoronic acid by hepatocytes to make it water soluble
Conjugated bilirubin secreted into bile & out into gut
Some passes into enterohepatic circulation
Rest converted to Urobilinogen by gut bacteria
Urobilinogen either:
Reabsorbed and excreted by kidneys
Converted to stercobilin which colours faeces brown
Jaundice can be classified by the site that is causing the problem. State the 3 types
and give an example of each. (3marks) Pick one of the types and fill in the table
below to show what you would expect to find when testing LFT’s – State if the level
would be increased, decreased or within the normal range. (2marks)
Pre-hepatic
Hepatocellular
Post-Hepatic
Bilirubin
↑
↑
↑
AST
N
↑
↑
ALP
N
↑ or N
↑
GGT
N
↑
↑
PRE-HEPATIC
Physiological = neonatal
Haemolysis
Glucuronyl transferase deficiency = Gilbert’s syndrome, Crigler-Najar syndrome

LFT’s:

Bilirubin (micromoles/l) 50-150 (normal range 3-17)

AST I.U. < 35 (normal range <35)
95

ALP I.U. <250 (normal range <250)

Gamma GT I.U. 15-40 (normal range 15-40)
HEPATOCELLULAR
Viruses = Hep. A, B, C, CMV, EBV
Drugs = Paracetamol overdose, Anti-TB (RIP), Sodium Valporate etc.
Alcoholic hepatitis
Cirrhosis
Hepatic Metastases
Haemochromatosis
Wilson’s Disease
Budd-Chiari syndrome
Dubin-Johnson syndrome
1-antitrypsin deficiency
Toxins = carbon tetrachloride

LFT’s:

Bilirubin (micromoles/l) 50-250 (normal range 3-17)

AST I.U. 300-3000 (normal range <35)

ALP I.U. <250-700 (normal range <250)

Gamma GT I.U. 15-200 (normal range 15-40)
CHOLESTATIC (OBSTRUCTIVE)
Gallstones in common bile duct
Pancreatic cancer
Drugs = Antibiotics (flucloxacillin, fusidic acid, co-amoxiclav), Oral contraceptives,
Clorpromazine etc.
Cholangiocarcinoma
Sclerosing cholangitis
Primary biliary cirrhosis
Biliary atresia

LFT’s:

Bilirubin (micromoles/l) 100-500 (normal range 3-17)

AST I.U. 35-400 (normal range <35)

ALP I.U. >500 (normal range <250)

Gamma GT I.U. 30-50 (normal range 15-40)
You want to take a history from this man. List 4 pertinent questions that you would
like to ask. (4marks)
96
Blood transfusions
IV Drug Use
Body piercing/tattoos
Sexual Hx
Travel abroad
Alcohol consumption
FHx of jaundice
Medications
Contact with other jaundiced people
From the history you discover that this man has been drinking heavily for the past
10years. State the most likely diagnosis and two things might you expect to find on
examination of this man. (2marks)
Diagnosis = Alcoholic liver disease causing cirrhosis/hepatic failure.
Palmar Erythema
Spider naevi
Gynaecomastia
Bruising
Scratch marks
Dupytren’s contracture
Clubbing
Leuconychia
Testicular atrophy
Hepatomegally
Ascites
Based upon your most likely diagnosis from (e), what investigations would you like
to carry out for this patient? (3marks)
Bloods

FBC

Folate & B12

LFT’s – Alk. Phos., ALT, AST, Bilirubin

GGT

Albumin

PT/INR
97

WCC

Platelets

Virology – rule out infective cause
Ascitic Tap
Liver USS
You diagnose alcoholic liver cirrhosis. How would you manage this patient’s ascites?
(2marks)
Fluid restriction
Spironolactone (100mg/24h PO,  every 48rs to 400mg/24h)
Add Furosemide (<120mg/24h PO)
Monitor U & E’s, Cr
Chart weight loss = aim for 0.5kg/day
You hear your consultant talking about portal hypertension and porto-systemic
anastomoses.
What is portal hypertension and how does it occur? (1mark)
Damage to liver causes a raised pressure in the venous portal system.
Raised pressure stimulates expansion of collaterals between the portal and
systemic system
Name the four sites of porto-systemic anastomoses. (2marks)
Oesophagus = Haematemesis
Umbilical = Caput medusa
Anus = Haemorrhoids (rectal bleeding)
Retro-peritoneal
3.6 Jaundice & hepatomegally
By the end of Phase II students should be able to:
-
distinguish pre-hepatic, hepatic, post-hepatic jaundice on clinical & biochemical
grounds
-
distinguish between infectious and mechanical causes of biliary obstruction
-
initiate appropriate investigations
-
distinguish the common causes of hepatomegally on clinical grounds
-
initiate investigations for hepatomegally
98
-
recognise the manifestations of chronic liver disease including encephalopathy and
portal hypertension
-
recognise the situations associated with acute hepatic failure, the signs of hepatic
failure and initiate immediate management
99
Mr Best, who admits to an alcohol intake of 20 cans of lager per day, has a 3
month history of jaundice, abdominal swelling, shortness of breath and itching.
Looking through his previous admission notes you see that cirrhosis of the liver
was diagnosed.
What is cirrhosis
Loss of normal liver architecture with diffuse fibrosis and nodular
degeneration which is usually irreversible
In late cirrhosis, a small liver can be found. However, earlier on in the disease a
patient may have hepatomegaly. Apart from cirrhosis name 3 causes of
hepatomegaly.
CCF
Infectious causes e.g. acute hepatitis, weils disease, Epstein barr virus
Carcinoma, neoplasm
Cholangitis
Haematological -Leukaemia, lymphoma, thalassaemia
Vascular – e.g. amyloid
Reidel’s lobe
Reye’s syndrome
Abcess, cyst
Tricuspid incompetence (pulsatile)
As well as alcohol abuse, name three causes of cirrhosis
Infection
Hepatitis – viral, autoimmune, fungal
Autoimmune
Primary sclerosing cholangitis
 Autoimmune liver disease
Genetic causes alpha 1 antitrypsin deficiency
 Haemochromatosis
 Copper deposition (wilson’s disease)
100
drugs – e.g. amiodarone, methyldopa, methotrexate
hepatic venous outflow obstruction: venoocclusive disease, Budd Chiari
"cardiac cirrhosis" from constrictive pericarditis
What abnormalities in blood results (FBC, LFT etc) would you expect in
someone with chronic liver disease associated with alcohol abuse(values not
required)
FBC –mean cell volume increase
Decreased platelets and WCC
Increased gamma GT
Increased AP and AST
Decreased albumin
Increased bilirubin (jaundice)
INR/PT increase
State the test you would perform to confirm your suspicion of ascites on
examination
One of:
Shifting dullness
Fluid thrill
(‘Dipping’ may also be done to look for masses underlying ascites)
What other investigations would you carry out in suspected ascites
Ultrasound
Diagnostic paracentesis
Name three ways in which ascites is managed?
Bed rest
101
Fluid restriction
Low salt diet
Spironolactone (frusemide added if no response)
Observation: daily weights, U&E, creatinine (stop diuretics if Na↓ or
creatinine ↑)
Paracentesis (with albumin infusion)
Mr Best has signs of encephalopathy and portal hypertension. Name two
presentations for each.
Encephalopathy:
Liver flap
Confusion
Restlessness
Slurred speech, incoherent speech
Coordination problems
Drowsiness, stupor
Coma
Foetor hepaticus
hyperreflexia
Portal hypertension:
ascites - with low plasma albumin
porto-systemic shunts - e.g. caput Medusae, oesophageal varices
haematemesis or melaena - due to rupture of gastro - oesophageal varices
venous hum of abdomen
haemorrhoids
peripheral oedema
(increased risk of hepatocellular carcinoma)
Name three hand signs of liver disease
Palmar erythema
Brusing
102
Terry’s nails
Dupytren’s contracture
Gynecomastia
Clubbing
Asterixis
leuconychia
Objective: Chronic liver disease
103
3.7
Whilst working in A&E as a newly qualified F1 doctor you are presented with Ruddiger a
52 year old male.
From the history you ascertain that Ruddiger has had an alcohol
problem for the past 10 years following the loss of a jackpot winning lottery ticket.
He
has recently been drinking even more following the loss of his trusted Labrador, Wotsit.
On examination the most striking finding is his distended abdomen which you suspect
is due to ascites.
List 2 clinical signs you would look for to demonstrate ascites?
Shifting dullness
Fluid thrill
Abdominal distension
List 2 signs associated with ascites that you would look for?
Abdominal wall venous distension
Ankle oedema
Distension of the neck veins
Divarifacation of the recti
Your examination has clearly demonstrated that Ruddiger has ascities.
medical student attached to your firm asks you what ascites is.
An inquisitive
What would be your
response?
Ascites is the presence of free fluid within the peritoneal cavity
What investigation would you order to determine the nature of the fluid within his
abdomen?
Diagnostic paracentesis / ascites tap
List 2 things that this fluid would be analysed for?
104
Protein
Malignant cells
Infection
Blood / white cells
The fluid can either be a transudate or an exudate.
What is the difference and list 2
causes for each?
Transudate = protein <25 g/L
Exudate = protein > 25 g/L
Transudate – cardiac failure, liver failure, cirrhosis, hypoproteinaemia
Exudate – Intra-abdominal malignancy, infection, pancreatitis
Investigations show the fluid to be a transudate.
You suspect the ascites is due to
damage to Ruddiger’s liver from the excessive alcohol.
List 4 things that your
management would include to relieve the ascites?
Bedrest
Fluid restriction (<1.5 L/day)
Low salt diet
Diuretic – spironolactone
Diuretic – furosemide
Therapeutic paracentesis / ascites tap
Alcohol avoidance advice
3.7
Ascites
By the end of Phase II students should be able to:
•
detect ascites clinically
•
initiate appropriate investigation having regard to the likely causes
•
initiate management of hepatic ascites
105
3.9
A 26 year old medical student comes to you complaining of abdominal pain and
explosive diarrhoea that has been getting worse for the past 7 days. She has noticed
some blood mixed in with her stool and is now very concerned. She reports that she
has been travelling around Thailand for the previous 3 weeks.
a) What is the likely cause of her bloody diarrhoea?
Any of :
b)
Pancreatitis,
Campylobacter, Shigella, Salmonella, E.coli (1)
inflammatory
bowel
disease,
antibiotics,
malabsorption
and
thyrotoxicosis are all causes of chronic diarrhoea, but can you name 2 other bacteria,
protazoa and viruses that cause acute diarrhoea (½ for each)?
Bacteria
Clostridium difficile
Mycobacterium tuberculosis
vibro choleae. (1)
Protazoa
Entamoeba Histolytica
Giardia lamblia (1)
Viruses
Rotavirus (Grp A)
Astrovirus
Norwalk-like virus
enteric adenovirus
calicivirus (1)
c) What investigation and treatment would you recommend ?
Stool culture +/- parasite screen (1)
Rehydration using isotonic oral fluids containing electrolytes and glucose (1).
(Intravenous fluid replacement may be indicated in patients with persistent vomiting)
Ciprofloxacin 500mg bd or Erythromycin 500mg qds for 1 week is indicated as her
diarrhoea is persistent.(1)
Anti diarrhoeal drugs should be avoided here to avoid prolonging the infection.
d) She returns 2 months later complaining of severe constipation and she has not had
106
her bowels open for 10 days. Give 3 different drugs used in the treatment of
constipation and briefly mention how they exert their effects: (½ each 3max)
Lactulose/Magnesium sulphate
- Osmotic laxative (1)
Senna/Docusate sodium - Motility agent (1)
Ispaghula Husk/Fybogel - Bulk forming laxative (1)
Extended Question – The defecation reflex occurs in response to distension of the
rectal wall mediated by mechanoreceptors, can you describe the stages involved ?
Contraction of the rectum
Relaxation of the internal and external sphincter
An initial contraction of the external anal sphincter
Increased peristaltic activity in the sigmoid colon
Relaxation of the external anal sphincter
Expulsion of faeces
(27 extra marks if you remember all this)
3.9
Diarrhoea and constipation
By the end of Phase II students should be able to:
• appreciate likely diagnoses in patients with acute and chronic diarrhoea and how they may
be distinguished on clinical grounds
• initiate appropriate investigation
• assess the physiological effects of severe diarrhoea
• explain the importance of oral rehydration solutions
• distinguish the common causes of constipation on clinical grounds
• initiate appropriate investigations for constipation
107
• manage constipation
108
3.10
A 15 year old girl newly diagnosed with Inflammatory bowel disease.
Q1. How might she have presented?
Diarrhoea with blood / mucus
RIF mass
Abdo pain
Weight Loss
Signs of anaemia
Systemic signs E.g Skin and Eyes
Fever
Q2. How would you investigate her (she presented with diarrhoea)?
FBC – Anaemia, white cell count, raised platelets
LFT’s – albumin, alk phos
U/E’s – potassium
Inflammatory markers – CRP, ESR
Anti-endomysial antibodies – (celiac)
Stool sample – infective cause
Endoscopy – histology and macroscopic appearance
Q3. On histology how would you differentiate between UC and Crohns?
UC
Crohns
Confined to Mucosa
Skip lesions with multiple biopsies
Crypt abscess
Transmural
Cell displasia
Granulomas and giant cells
Macrophages, lymphocytes and
plasma
cells
Q4. What are the macroscopic differences?
UC
Crohns
Rectum to iliocaecal valve
Mouth to Anus
(occasionally backwash ileitis)
Skip lesions
109
Superficial ulceration
Rose thorn fissures
Pseudopoylpos
Cobblestone appearance
Fistulae and Abscesses
After 5 years she has had repeated exacerbations and presents to A&E with bloody
diarrhoea occurring fifteen times a day and lower abdominal pain, for the last 3 weeks
She is very distressed and feels she has no quality of life
Q5. Give your acute management?
ABC
IV fluids
Admit
IV steroids
NBM
AXR – ? abscess
+/- antibiotics
ASA e.g sulphasalazine
Analgesia
After 1 week she is showing signs of improvement but still does not feel that she is able
to cope at home.
Q6. Discuss her options (for each disease)
UC
Crohnes
Medically – aziothioprine
If localised – can operate
steroids
Otherwise – aziothioprine
ASA’s
steroids
Surgically – remove affected area
ASA’s
o
Panproctocolectomy
Q7. She opts for a surgical approach to discuss the issues
110
UC
Positives
Negatives
Reduced colon cancer risk
Major Surgery – anaesthetic risk
Symptom resolution
Stoma – possibly life long

Long term incontinence post reversal
Crohns
Positives
Negatives
Possible reduction in symptoms
Reduced chance of perforation

Recurrence of Disease
Adhesions
Fistula formation
Q8. When wanting to assess disease control other than GI symptoms what else might
you question her about (systemically)
Eyes – Uveitis
o
Irritis
o
Episcleritis
Skin -
Rashes – erythema nodosum, pyoderma gangrenosm
Joints- Ankylosing spondylitis
o
Liver-
Sacroilitis
PSC
Gall Stones
Q9. (Extended)
Her brother suffers from Ankylosing spondylitis. What genetic factor might they
share?
HLA B27
111
3.10
Inflammatory bowel disease
By the end of Phase II students should be able to:
• recognise the possibility of inflammatory bowel disease in patients presenting
with lower
gastrointestinal symptoms
• initiate appropriate investigations in a patient with a suspected inflammatory bowel disease
• initiate appropriate management in a patient with inflammatory bowel disease
• recognise the differences in presentation between Crohn's disease
and ulcerative colitis,
and how these relate to the underlying pathology
• recognise the possibility of systemic symptoms associated with inflammatory bowel disease
• explain to patients the nature of and the rationale for maintenance treatment for inflammatory
bowel disease
112
3.12
A 42 year old man presents to his GP with a 4 day history of bloody
diarrhoea. The blood is fresh in appearance. The patient has been
diagnosed as suffering from FAP since his early 20's. The
patient also complains of abdominal pains which subside once he goes
to the toilet. He is mildly short of breath and looks pale in
appearance.
Question 1a
What is FAP and explain why you are worried this patient has cancer
of the large bowel?
People with Familial Adenomatous Polyposis, a rate autosomal dominant
disorder, almost always develop cancer of the intestine by age 35.
It is characterised by thousands of polyps growing in the large
intestine. It is a pre-malignant condition.
Question 1b
Explain the breathlessness.
The patient is likely anemic due to chronic blood loss.
Question 2
What are the 4 layers of the large bowel and for that matter the rest
of the GI system?
Mucosa, Submucosa, Muscularis Propria/Externa, Serosa
Question 3
Name 1 other symptom you might expect the patient to complain about
if he indeed has cancer of the large bowel.
113
Unexplained weight loss
Question 4
What is the most common colon cancer cell type?
Adenocarcinoma (which accounts for 95% of cases).
Question 5
Which classification is used to stage cancer in the large bowel?
Briefly explain each stage.
Duke's A: invaded submucosa and muscle layer of the bowel but
confined to the wall.
Duke's B: breached the muscle layer and bowel wall but no involvement
of local lymph nodes.
Duke's C1: spread to immediately draining pericolic lymph nodes.
Duke's C2: Spready to higher mesenteric lymph nodes.
Duke's D: Distant visceral metastases.
Extended Question 6
Name 5 factors which increase a person's risk of developing Colo-rectal
cancer.
Any 5 of:
Smoking
Diet high in fats
Diet high in carbohydrates
Diet low in fibre
Suffering from Inflammatory bowel diseases (i.e. Crohn's, UC) as these
conditions are pre-malignant.
114
Family Hx of Bowel Cancer
Age. The risk of developing colorectal cancer increases with age.
Most cases occur in the 60s and 70s, while cases before age 50 are
uncommon unless a family history of early colon cancer is present.
HNPCC (Hereditary Non-Polyposis Colorectal Cancer)
Extended Question 7
Name 2 investigations you would like to see carried out to assess
the extent of disease in this patient's bowel?
Barium Enema
Colonoscopy +/- Biopsy
3.12
Bowel cancer
By the end of Phase II students should be able to:
• recognise the possibility of colonic cancer in patients with
disturbed bowel habit, rectal bleeding or anaemia
• initiate appropriate investigations to confirm the diagnosis and
extent of disease
• suggest a prognosis based on staging and other factors
• outline to patients the possible management strategies for bowel cancer
• outline to a patient the management of a colostomy
115
3.13
Peter, a 58 year old gentleman presents to accident and emergency with a 6 hour
history of acute abdominal pain.
As the surgical house officer you are called.
On arrival in A+E Peter has a pulse of 100bpm, is pyrexial and in obvious distress.
Q1
As you are walking to A+E what are your top 4 differential diagnoses for acute
abdominal pain.
Perforation
Bowel Obstruction
Acute Pancreatitis
Dissecting Aortic Aneurysm
Mesenteric Vascular Obstruction
Acute appendicitis
Acute Cholecystitis
Kidney Stones
Diverticulitis
Peptic Ulcer
On taking a history you find that he has a 6 hour history of severe epigastric pain,
waking him from his sleep, which is associated with nausea and vomiting.
history of gallstones and hypertension.
There is a
Peter attributes his symptoms to food at his
daughters wedding the day before.
Q2
What initial investigations would you carry out and why (be specific, not just ‘blood
test’)?
FBC: Raised WCC (infection)
U+E’s: Dehydration (N+V), General kidney function (management)
CRP, ESR: inflammatory markers
116
LFT’s: Raised AST secondary to Gallstones, Albumin for severity of ?pancreatitis
Amylase: May be raised in pancreatitis
Glucose: Severity of ?Pancreatitis
Bone (Ca): Severity of ?Pancreatitis
ABG: Severity of ?Pancreatitis
Erect CXR – Perforation? Air under the diaphragm?
AXR – Kidney stones? Sentinal Loop present? Psoas shadow?
ECG
Q3
Apart from gallstones, give 4 other common causes of pancreatitis?
If gallstones are
not the cause, what do you think is the most likely cause in this case?
Alcohol
Drugs
Trauma
ERCP
Autoimmune
GET SMASHED – causes such as scorpion sting not common in this country!
Alcohol and diuretics most likely cause – due to daughters wedding the night before,
at which he may have been drinking, and history of hypertension, which may be
treated with diuretics.
Q4
In pancreatitis you would expect an increase in amylase, name 3 other causes of an
increased amylase.
Upper GI tract perforation
Leaking AAA
Mesenteric Ischaemia
Acute cholecystitis
Renal Failure
117
Q5
What parameters would you use to assess the severity of an individual such as Peter
who is suffering from pancreatitis?
And what is the name for this scoring system?
Modified Glasgow Criteria
PaO2 < 8 kPa
(PANCREAS)
Age >55 years
Neutrophils >15x109/L
Ca < 2mmol/L
uRea >16mmol/L
Enzymes – LDH >600iu/L, AST >200iu/L
Albumin<32g/L
Sugar (glucose) > 10 mmol/L
Peter scores 4 on this scale and your consultant admits him of ITU.
Q6
In your further management what complications would you look for?
ARDS
Diabetes Mellitus
Pseudocysts
Pleural effusions
Organ failure
Pancreatic abscess
Ascites
Sepsis
Extended Q
118
What signs would you look for on abdominal examination in severe pancreatitis and
what are they called?
Cullens sign – Peri-umbilical bruising
Grey-turners sign – Bruising of the flanks
3.13
Acute abdominal pain
By the end of Phase II students should be able to:
• recognise localised and generalised peritonitis and begin to distinguish the common causes
• appreciate the causes of a ruptured viscus and how they may be begin to be distinguished
• investigate, assess and initiate management of acute pancreatitis
• appreciate the common causes of pain in the right iliac fossa and how they may be
distinguished
• initiate appropriate investigations in a patient with upper abdominal pain
• recognise the possibility of bowel obstruction
• initiate appropriate investigations and initiate intravenous fluid replacement and appropriate
pain relief
119
A 52-year-old man presents to A&E with a four-day history of severe epigastric pain
radiating to the back. The patient describes the pain as a sharp constant pain present in
the epigastric region radiating to the back in a band like fashion. The pain was made
worse on eating and drinking alcohol and relieved slightly by sitting forward. The
patient described associated symptoms of vomiting and nausea.
Question 1: What are the main differentials for the patient’s presentation?
Acute pancreatitis
Peptic ulcer
Acute cholecystitis
GI obstruction
Question 2: What investigations would you order?
Bloods – FBC, U&E, Amylase, LFT’s, and CRP
Abdominal x-ray
Ultrasound of gallbladder
Question 3: After investigation the patient’s serum amylase was found to be 3045u/mL,
LFT’s were normal, WCC was 11.5 and CRP was 105. Nothing abnormal was detected on
Ultrasound and on abdominal x-ray; there was no psoas shadow. Given these results,
what is the most probable cause of the patient’s presentation?
Acute pancreatitis
Question 4: What are the commonest causes of acute pancreatitis?
G – Gallstones
E – Ethanol
T – Trauma
S – Steroids
M – Mumps
A – Autoimmune – PAN
S – Scorpion venom
H – Hyperlipideamia/Hypercalcaemia/Hypothermia
120
E – ERCP
D – Drugs – azathioprine, asparaginase
Question 5: What common signs may be found on examination with a patient with acute
pancreatitis?
Tachycardia
Fever
Jaundice
Shock
Generalised abdominal tenderness
Periumbilical discolouration – Cullen’s sign – around the umbilicus
Grey Turners sign – at the flanks
Question 6:What are the criteria for predicting the severity of acute pancreatitis?
P – PaO2 < 8 kPa
A – Age > 55
N – Neutrophils > 15*10power9/L
C – Calcium < 2 mmol/L
R – Renal function – Urea > 10mmol/L
E – Enzymes – LDH > 600iu/L and AST > 200iu/L
A – Albumin < 32 g/L
S – Sugar – Glucose > 10 mmol/L
Question 7: What would be the management plan for this patient?
Nil by mouth
Set up IVI and give plasma expanders and 0.9% saline until vital signs are
satisfactory and urine flow > 30mL/L. Check weight daily and insert a catheter.
Analgesia: Pethidine 75-100mg/4h IM, or morphine (may cause the sphincter of oddi
to constrict more)
Hourly pulse, BP and urine flow
Daily FBC, U&E, glucose, amylase, blood gas.
121
If worsening, take to ITU.
Question 8: What are some of the complications of acute pancreatitis?
Early:
Disseminated Intravascular Coagulation
Renal failure
Respiratory failure
Haemorrhage
Thrombosis – may occur in the splenic and gastroduodenal arteries, or in the
colic branches of the superior mesenteric artery, causing
bowel necrosis.
Late: > 1 week – P
Pancreatic necrosis
Pseudocyst
Objective: acute pancreatitis
122
3.14
Lindsay, a 23 year old previously healthy woman presents to her general practitioner
with a 3 month history of generalised abdominal pain.
On looking at her previous attendance and notes she was last seen for back pain 5
months ago.
Q1
List 6 further questions you would want to ask about her presenting complaint.
Change in bowel habits
Change in consistency
PR bleeding/mucus
Diet/relationship to food
Relieving/aggravating factors
Bloating/flatus
Tenesmus
PV bleeding/LMP/cyclical/discharge
On further questioning, Lindsay reports a change in bowel habits, primarily
constipation.
Q2
Give 4 causes of constipation which may be applicable in this case?
Decreased dietary fibre
Hypothyroidism
Hyperparathyroidism
Anxiety/depression
IBS
Drugs (codeine based – may have been used to treat previous back pain)
Other common causes are less likely due to the young age and previous good health.
123
On further questioning, she reports an associated bloating feeling.
She also believes
she leads a healthy lifestyle.
On examination of Lindsay there is nothing remarkable found.
Q3
What investigations would you do, if any?
There are no real indications for any investigations, however if other symptoms were
present to indicate hypothyroidism or parathyroidism then the following would be
indicated:
Thyroid function tests
Plasma Ca
All tests come back normal.
Q4
Give steps to your management of Lindsay.
Description of IBS
Reassurance
Patient own management eg increase dietary fibre, increase exercise, decrease
stress
Medical therapy – osmotic laxative eg lactulose, anti spasmodic eg Mebeverine
Q5
Name one complication of this disorder?
Colonic diverticulae
124
3.14
Irritable bowel syndrome
By the end of Phase II students should be able to:
• recognise the presentations of irritable bowel syndrome
• appreciate other common causes of chronic abdominal pain and how they may be
distinguished
• explain to a patient the nature of irritable bowel syndrome and its relationship to precipitants
such as stress.
125
3.17
MP is a 35 year old man who was diagnosed with Type 1 diabetes mellitus at the age of
14 after presenting to his GP with polyuria, polydipsia and weight loss. Urine dipstick
demonstrated the presence of glucose and ketones, which is not diagnostic of diabetes
mellitus, but indicates the need for further investigation.
Name three blood tests used to diagnose diabetes mellitus and the values that indicate
presence of the disease.
Random blood glucose > 11.1 mmol/L
Fasting venous plasma glucose ≥ 7.0 mmol/L
2 hour oral glucose tolerance test > 11.1 mmol/L
NOTE: These values refer to venous plasma levels, rather then that of venous whole
blood.
Diabetes mellitus increases the risk of cardiovascular disease. Which other co-existent
cardiovascular risk factors can be screened / assessed in a diabetic patient? (Name 3)
Blood pressure
Cholesterol
Smoking history
Diabetic control (BMs, HbA1c)
When Mr P was initially diagnosed he was educated on his condition, advised about the
importance of diet, referred to a dietician, and started on insulin.
You are concerned that Mr P is non-compliant. You are worried that he is not following
your dietary advice with a large alcohol intake, and is not regularly taking his insulin.
Which investigation can be used to determine a patient’s glycaemic control? What is
the target value? What does this value indicate?
HbA1c < 7 %
Glycosylated haemoglobin levels relate to the mean glucose level over the previous 8
weeks (i.e. RBC half life)
Later that week Mr N was found at home drowsy and a little confused following a late
night where he consumed a lot of alcohol. Earlier that day he had complained of feeling
shaky, sweating, pallor, heart pounding, and irritability. What is the likely diagnosis?
126
Hypoglycaemia
Name two other ways that this condition may present.
Coma
Seizures
Temporary paralysis
You measure his blood glucose as 2.5. What management would you like to initiate?
Oral sugar e.g. fruit juice, sugar lump, followed by long acting starch e.g. toast
How would your management differ if he was unresponsive?
Call an ambulance
Glucose 25-50g IV or glucagon 0.5-1mg SC/IM – should see some improvement
following this.
Repeat after 20 minutes if no improvement
If prolonged hypoglycaemia, may want to give dexamethasone 4mg/4hr IV to combat
cerebral oedema.
3.17
Insulin dependent diabetes
By the end of Phase II students should be able to:
• identify patients with insulin-dependent diabetes
• screen patients for co-existent cardiovascular risk factors
• screen for diabetes-related complications
• initiate management of a patient with IDDM, including the appropriate use of long- and
short-acting insulins
• determine a patient's degree of metabolic control
• recognise diabetic ketoacidosis
• participate in the management of diabetic ketoacidosis
• recognise and manage hypoglycaemia
• outline to patients the dietary principles of the management of IDDM
127
3.19
A long standing insulin dependent diabetic Mr B, presents at your clinic having recently
been diagnosed with hypertension.
You realize that the combination of long standing
diabetes and hypertension puts Mr B at greater risk of cardiovascular complications;
hence you decide to examine his cardiovascular system.

Name 6 signs or symptoms you might expect to find on Mr B?
Intermittent claudication
angina pain
reduced exercise tolerance
diminished peripheral pulses
reduced capillary refill, cold peripheries
ischaemic skin changes
poor healing and ulcers.

How would you treat Mr B’s hypertension in order of preventing the development
of cardiovascular complications?
Treatment is dependent upon the BP;
<145/80 and no microalbuminuria check BP every 6months
>140/80 an <160/100 and no microalbuminuria, treat with antihypertensives; ACE
inhibitor, beta-blocker or a thiazide
>140/80 with microalbuminuria should be treated with either an ACE inhibitor or
angiotensin 2 antagonist, with a target BP of 135/75
In addition to examining Mr B’s cardiovascular system you decide to perform
fundoscopy to detect any retinal changes of diabetic retinopathy.

Name the 4 stages of diabetic retinopathy with the key features of each?
Background, diabetic maculopathy, pre-proliferative and proliferative
Explain why these retinal changes develop in diabetic patients?
128
These changes are the result of microvascular disease, with an increase in the
thickness of the basement membrane, and increased permeability of the retinal
capillaries.
The result is the formation of microaneurysms that haemorrhage and later
lead to new vessel development.
Diabetic retinopathy is one of the complications that can develop in a diabetic patient.

List 4 other complications of diabetes and whether they are microvascular or
macrovascular changes?
Neuropathy – microvascular
Nephropathy – microvascular
Peripheral vascular disease –macrovascular
Ischaemic heart disease –macrovascular
Stroke - macrovascular
What are the pathological differences between microvascular and macrovascular
changes?
Microvascular disease involves the small vessels in the retina, glomerulus and vasa
nervorum, whilst macrovascular involves the larger vessels.
In microvascular disease there is progressive narrowing of the vascular lumina due to
hyperglycaemia-induced microvascular hypertension, basement membrane thickening
and increased vascular permeability.
Macrovascular disease involves atherogenesis; the infiltration of lipid in the intimal layer
resulting in plaque formation and occlusion of the vessel.
Finally, you decide to manage Mr B with a multidisciplinary approach.

Name 4 health professionals/ agencies that could be involved in his care?
Endocrinologists/ GP/ diabetic nurse/ chiropodist/ophthalmologist/renal
physician/orthopaedic surgeon
3.19
Long term complications
By the end of Phase II
students should be able to:
•
recognise the long term cardiovascular complications of diabetes
•
manage hypertension appropriately in diabetics
129
•
recognise renal disease in diabetics and refer appropriately
•
recognise diabetic neuropathy
•
recognise autonomic neuropathy
•
recognise the potential importance of skin lesions in diabetics
•
recognise diabetic retinopathy and refer appropriately
•
describe the mechanisms underlying these complications and the potential for their
prevention
130
3.23
Alice is a 66-year-old housewife who complains of weakness and episodes of fainting,
which makes her spend most day in bed. She also complains of nausea, vomiting and
abdominal pain. Alice also mentions in passing that her friends have been commenting
on her tan, even though she has not been on holiday.
a) what could be the diagnosis? (name 2)
Addison’s disease
Congenital adrenal hyperplasia
a) give two common causes for this condition?
Autoimmune
Tuberculosis
HIV/AIDS
Metastatic carcinoma
Bilateral adrenalectomy
b) what is the name of the test that will confirm your diagnosis and give a brief
explanation of the procedure?
Short ACTH Stimulation test (Synacthen Test)
Do plasma cortisol, inject tetracosactide IM (exogenous ACTH), plasma cortisol
after 30 minutes.
Cortisol remains low in Addison’s disease, but if above 600nmol/l then exclude
Addison’s disease
c) name two drugs that may be given to this patient?
Glucocorticoid replacement with either hydrocortisone or prednisolone
Mineralocorticoid replacement with fludrocortisone
131
Alice was given treatment. 2 years later she complained of being overweight, easy
bruising, excess hair growth and purple marks on her stomach.
d) what is the name of this condition?
Cushing’s syndrome
EXTENDED
Why has Alice got a ‘tanned’ appearance to her skin?
Lack of adrenal hormones causes feedback on the hypothalamo-pituitary axis to
increase CRH and therefore ACTH production. ACTH contains MSH-like
sequences that cause pigmentation due to stimulation of melanocytes.
OBJECTIVES
3.23
Adrenal gland hormones
By the end of Phase II students should be able to:
• recognise the symptoms and signs of Addison's Disease
• confirm the diagnosis of Addison's Disease
• initiate the immediate management of Addisonian crisis
• recognise symptoms and signs of Cushing's Syndrome
• confirm the diagnosis of Cushing's Syndrome
• participate in the long term management of Addison's Disease and Cushing's Syndrome
132
3.24
Dr Patel thrusts another rather large set of notes and asks you to see Mr PA in the side
room. On glancing at the GP letter you read:
‘This 45yr old gentleman has been complaining of long term headaches and visual field
disturbances suggestive of a pituitary lesion. The results of his glucose tolerance tests
(GTT) are consistent with acromegaly. I would be grateful if you could review his care’
Which type of visual field loss would you expect to find in this patient?
(1 mark)
Bi-temporal hemianopia (tunnel vision)
List four other clinical features you would expect to find during a physical examination
on this patient other then those already mentioned.
(2 marks)
prominent eyebrow ridge
increased interdental space
Prognathism,
greasy skin,
large tongue,
spade like hands,
features relating to carpel
tunnel syndrome
(diff from Prognathism)
husky voice
enlarged feet
Briefly describe how you would perform a glucose tolerance test?
(2
marks)
Overnight fasting
Oral glucose 75mg taken at T 0 mins
Monitor BM and growth hormone at timed intervals, 0>30>60>90>120 mins
Absence of GH suppression suggests acromegaly.
Suggest two non-biochemical investigations you may carry out to assess the severity
of Mr PA’s main condition.
(2 marks)
MRI brain: assess size and spread of pituitary lesion
133
X-ray feet and hands: looking for osteophytes and increased joint space
( Mr PA condition is a pit adenoma causing excessive levels of GHRH and thus GH
causing acromegaly. Carpel tunnel is not his main problem thus nerve conduction
tests are not investigations you would order yet. )
Dr Patel, being impressed with your answers so far, asks how you would treat this
patient with acromegaly. (Assume he is otherwise fit and well).
(3
marks)
Surgery (hypophysectomy) is gold standard with adjunct radiotherapy to shrink the
lesion. (If unfit for surgery then somatostatin analogues (octeotide) and dopamine
agonists (bromocriptine) are effective)
Regular pituitary function tests are essential.
Extended question
Describe the clinical features you would expect to find if Mr PA’s condition was left
untreated
(E3 marks)
Acromegaly > Hypopituitarism.
Hypothyroidism features
Androgen deficiency (loss libido, hypogonadism, loss of muscle bulk)
Corticotrophin deficiency (postural HT, weight loss, low sodium)
3.24
Pituitary gland hormones
By the end of Phase II
students should be able to:
• recognise the circumstances when hypopituitarism may occur
• recognise the possibility of hypopituitarism in patients with 'non-specific' symptoms
• initiate investigations for posterior pituitary function in patients with polyuria
• initiate investigations of the cause of hypopituitarism
• recognise acromegaly
• initiate investigations for acromegaly
• outline to patients the possible treatments for acromegaly
134
1) A)-Mr A is a 49-year-old widower and has just joined your GP’s practice. The practice
nurse has referred him because during his enrolment check-up, a BP of 173/90 was
recorded. You repeat his BP and it is 176/101.
What is the difference between essential and secondary hypertension? (1 mark)
Essential hypertension has no specific underlying cause, lifestyle plays a large part,
and secondary hypertension is the result of a range of pathological processes.
B)-Give 4 possible causes of secondary hypertension: (4 marks)
(Any 4 of below)
Renal parenchymal disease eg chronic pyelonephritis, glomerulonephritis, polycystic
disease
Renal artery disease eg atherosclerosis, fibromuscular dysplasia, vasculitis
Obstructive uropathy eg hydronephrosis 2 to stone/tumour
Congenital eg coarctation of the aorta
Drugs eg combined oral contraceptive pill, NSAID’s
Endocrine eg phaeochromocytoma, hyperaldosteronism, Cushing’s syndrome,
acromegaly
C)-At this first consultation, you are struck by Mr A’s large facial features including his
jaw, nose, brow and large hands. You suspect that Mr A maybe suffering from
acromegaly. Name 4 other signs or symptoms of acromegaly other than the ones
mentioned above: (4 marks)
Interdental separation, headaches, excessive sweating, visual deterioration, tiredness,
wt gain, amenorrhoea/oligomenorrhoea, galactorrhoea, impotence, poor libido, deep
voice, hirsutism, goitre, breathlessness, tingling in hands/carpel tunnel syndrome,
polyuria/polydipsia, muscular weakness, joint pains, thick greasy skin, tight rings.
D)-Describe one of the aetiologies for acromegaly: (1 mark)
Benign pituitary tumour secreting GH (commonest)
Pituitary carcinoma
Carcinoid tumours that secrete hypothalamic GH-releasing hormone.
135
E)-In the history Mr A went on to confirm that he had been experiencing headaches and
had noticed a change in his appearance but put it down to his age. In view of his
possible diagnosis, name 3 investigations would you like to do and why? (6 marks)
(3 of any, 1 mark for investigation, 1 mark for reason)
GH levels – excessive circulating levels of GH, although this is non-diagnostic on its
own because transient raised levels are normal during stress
IGF-1 levels – almost always raised in acromegaly as a result of increased GH levels
Glucose tolerance test – GH is measured during a GTT, in healthy individuals, GH is
undetectable during the test, because hyperglycaemia inhibits GH secretion.
MRI scan of pituitary – pituitary neoplasm is commonly the cause
Prolactin levels – frequently, the adenoma secretes both GH and prolactin
Assessment of visual fields – bitemporal hemianopia caused by pressure on the optic
chiasm by the tumour.
CXR and/or ECG – left ventricular hypertrophy secondary to hypertension
F)-A diagnosis of acromegaly is eventually made. What are the risks to Mr A if his
condition goes untreated? (2 marks)
Any hypertension related illness, eg coronary heart disease, heart failure,
cerebrovascular disease, PVD
Increased risk of neoplasia, especially large bowel tumours
G)-Discuss the treatment options available? (4 marks)
(Any 2; 1 mark for treatment, 1 mark for a relevant point)
Surgery – Trans-sphenoidal route is usually used, 90% of microadenomas are cured.
Complications include hypopituitarism, meningitis, intra-operative bleeding and death
Radiotherapy- Often used if attempts at surgery do not reduce GH levels.
Hypopituitarism may occur.
Medical therapies – Most effective is Octreotide, a somatostatin analogue, which
inhibits GH secretion in 60%. Bromocriptine, dopamine agonist reduces GH release,
less effective
Extended Question
H)-Explain the mode of secretion and physiological action of GH (4 marks)
136
GH is secreted by the anterior pituitary
GH acts directly on tissues, but most effects are mediated via IGF-1 (insulin-like
growth factor), produced mainly from the liver after stimulation from GH.
Stimulates cell growth and replication by accelerating the rate of protein synthesis
Increase uptake of amino acids, especially after a meal
Virtually every tissue responds, skeletal muscle and chrondrocytes especially
Counter regulatory to insulin, i.e., increases blood glucose via stimulating hepatic
gluconeogenesis and inhibiting glucose uptake by muscle
Secretion of GH occurs in bursts, in response to dual control by GHRH and GHRIH; it
is stimulated by low blood glucose and high blood amino acids, and during REM
sleep.
Objective: pituitary gland abnormalities
137
3.25
John is 18-years-old and is walking through the park when he sees some school friends
playing football. They ask if John would like to join in and hesitantly, because he hasn’t
done any exercise for years, he agrees to make up the numbers. Towards the end of the
game John is very sweaty, his hands are shaking and collapses on the floor. At hospital
John is diagnosed as being in a hypoglycaemic coma.
a) define hypoglycaemia in relation to plasma glucose levels?
<2.5mmol/l
b) give five causes of hypoglycaemia?(any 5)
Unexpected or unusual exercise
Alcohol
Insulin/ oral diabetes drugs overdose
Malabsorption
Addison’s disease
Hypopituitarism
Congenital adrenal hyperplasia
Liver failure
Alcohol abuse
Hypothermia
Insulinoma and other metastatic disease
c) what other signs and symptoms may his friends have noticed?
Sweating
Hunger
Drowsiness
Personality change
Fits
Anxiety
Confusion
Headache
138
Nausea
Tiredness
d) how would you treat such a patient?
ABC
50-100ml 50% dextrose IV fast followed by 0.9% saline flush as dextrose harms
veins
Glucagon 1mg IM if no venous access
EXTENDED
What is Whipple’s triad and which condition does it diagnose?

Diagnoses insulinomas by
symptoms are associated with fasting or exercise
hypoglycaemia is confirmed during these episodes
glucose relieves the symptoms
3.25
Hypoglycaemia
By the end of Phase II students should be able to:
• recognise the possibility of hypoglycaemia and the circumstances under which it may occur
• initiate appropriate initial investigations
• initiate immediate management of hypoglycaemia
139
A 21 year old female presents to A&E with a GCS of 9, abdominal pain and vomiting.
She is hyperventilating. She appears quite thin. Her friend tells you she has been
drinking heavily all evening but mostly soft drinks with little alcohol. The nurse calls
you over to show you she has an insulin pen in her jacket. You ask for a blood glucose
measurement which comes back at 26mmol/L.
1. Give the most likely cause of her symptoms
[1 mark]
Diabetic ketoacidosis (DKA)
2. List 3 factors which may precipitate this condition
[3 marks]
Infection
Poor compliance
Recent diagnosis
3. You decide to investigate further. List 5 investigations that would be suitable in
the acute phase. (be specific, don’t just put ‘blood test’)
[5 marks]
FBC
HbA1c
U+E
Glucose
Blood gases
Serum amylase
CXR
Ketones in urine or blood
4. Give 3 interventions to manage this patient
[3 marks]
Insulin Sliding Scale
Fluid replacement
Potassium supplementation
Nil by mouth
Treat precipitating cause if any
140
5. Explain why the Type 1 diabetic with severe hyperglycaemia will enter a ketotic
state while the Type 2 diabetic will tend towards the hyperosmolar nonketotic
state.
[5 marks]
Low insulin prevents the uptake of glucose into tissues.
This prevents cells using glucose as an energy source. Alternatives are required.
Energy is gained by producing fatty acids from lipolysis.
These are converted into ketones by the liver. These are present in the circulation and
produce a ketotic state
In Type 2 diabetics there is plenty of insulin present but there is resistance to it’s
action.
The blood glucose produces a hyper osmolar circulation which produces its clinical
effect due to osmotic pressures leading to cellular dehydration
Ojective: HONK and DKA
141
3.26
Sidney is a 55 year old bus driver.
He presents to A+E with non-specific symptoms of
headache, confusion and drowsiness.
Bloods results: serum Na levels of 102mmol/l (130-145mmol/l)
Q1: Name 3 medical disorders causing ↓Na:
Hypothyroidism
Addisons’ disease
SIADH
↑ extracellular volume (heart failure, liver failure)
Q2: Treatment generally involves correcting the underlying cause. But how would you
manage this patient acutely?
Mild:
Fluid restriction.
Moderate: IV saline and frusemide.
Severe (emergency): IV 0.9% saline or hypertonic 1.8% saline. Gradually ↑Na. can
combine with frusemide. Caution in heart failure.
Q3: What other investigations would you consider, to establish the cause?
Hypothyroidism (TFTs)
Addisons’ disease (blood plasma, synacthen test)
SIADH (plasma osmolality, urine osmolality, U&Es)
heart failure, liver failure ( BNP level, CXR, ECG, LFTs)
142
Deepak presents to his GP with a hx of polydipsia and polyuria.
Investigations show he has a ↑ serum Na level of 158mmol/l.
Q1:
Name 4 causes of ↑Na:
Diabetes insipidus,
↓ Water intake,
↑ Fluid loss with less Na loss (vomiting, diarrhoea, burns)
Incorrect IV fluid replacement (excessive saline)
Primary aldosteronism
Renal disease (ATN)
Q2: how would you initially manage this (relating management to both hypovolaemia
and hypervolaemia):
Hypovolaemia: Give water orally, or 5% dextrose IV slowly (guided by urine output,
and plasma Na. Avoid hypotonic solutions!
Hypervolaemia: diuretics to remove Na and ↑ fluid loss.
Q3: He was later found to have Diabetes insipidus, what test confirms the diagnosis of
this condition? How is it performed, and how would you interpret the results?
Water deprivations
o
Deprivations of water for ~9hours
o
Injection/intranasal administration of ADH (Desmopressin)
o
Measure urine osmolarity after hourly intervals (also can take serial plasma
samples)
Central DI: urine osmolarity increases
Nephrogenic DI: urine osmolarity changes very little
143
Rose has recently been diagnosed with type I diabetes mellitus.
She presents to A+E
with signs and symptoms consistent with DKA.
Blood results show that her serum potassium is 6.7mmol/l.
Q1: Why do you get ↑K in DKA?
Metabolic Acidosis in DKA, causes redistribution of electrolytes to help minimise
acidosis.
K:H exchanger (↑ extracellular K, ↑ intracellular H)
Q2: List 4 other causes of ↑K levels.
Renal failure: main cause, due to oliguria
Drugs: K sparing diuretic (main), ACEi, β-blockers, excess K therapy, massive blood
transfusion.
Redistribution to ECF:
Acidosis-(K:H exchanger), cell lysis (crush injury, haemolysis,
strenuous exercise, rhabdomyolysis, burns).
Endocrine: hypoaldosteronism (Addison’s disease)
Artefact: haemolysis of RBC’s in collecting bottle, delayed analysis-leakage of K.
Q3: What ECG changes would you expect to see with increasing K levels?
Tall/tented T waves, small/flat P waves, wide QRS complex, VF.
Q4:
List 5 options in treating K levels >6.5mmol/l?
10ml calcium gluconate. This provides cardio-protection, doesn’t lower K levels!!
Insulin + glucose: moves K into cells, via Na:K:
Nebulised salbutamol: moves K into cells.
Consider Calcium resonium PO or PR to bind K in the gut, SE: constipation.
Sodium bicarbonate: corrects acidosis.
High dose frusemide
Haemodialysis:
144
Jaswinder has a past hx of hypertension, controlled by use of a diuretic.
On routine
bloods his serum potassium was 2.4mmol/l.
Q1: name some other causes of low serum K?
Drugs: diuretics (main cause), β-adrenergic stimulation (salbutamol)
Mineralcorticoid excess: Conn’s syndrome, Cushing’s syndrome, corticosteroids,
ectopic ACTH, hyperaldosteronism.
Redistribution: alkalosis (K:H exchanger), excessive insulin.
Vomiting (pyloric stenosis, bulimia nervosa), Diarrhoea
Q2: What ECG changes are associated with ↓ K?
Prolonged P-R interval, small or inverted T waves, prominent U waves (after T wave),
depressed ST segment.
Q3: How would you manage K levels <2.5mmol/l?
IV KCl not more than 20mmol/h (never give a fast bolus, peripheral vein damage)
3.26
Abnormalities of sodium and potassium
By the end of Phase II
students should be able to:
•
initiate investigation of hypo and hyper natraemia and hypo and hyper kalaemia
•
initiate management of these conditions
145
4.3
You are a doctor working in an Emergency Department, you are called to see Miss HG a
15 year old girl who has fallen from her horse. She has been triaged and sent for
investigations.
Question 1:
What is this investigation, what is it of and what does it show? [3]
X –Ray, Clavicle, Fracture of distal 1/3.
Question 2:
When presenting an X ray what are the important features? [4]
Patients name, age, penetration,type of X ray, eg PA view of shoulder
Question 3:
How may the injury in question 1 be managed? [3]
Pain relief, broad sling or collar and cuff, review in around 3 weeks. Will not usually
require any surgery.
While you are seeing Miss HG she complains of an odd feeling in her wrist, you examine
it.
Question 4:
What features are you looking for on examination of the wrist? [4]
Neurovascular compromise (wiggle your fingers, can you fell me touching this, distal
146
pulses). Any painful areas, deformity/swelling, open fractures, bruising.
Extended Question:
What are the stages of bone healing? [4]
Haematoma –Granulation Tissue – Callus Formation – Bony consolidation Remodelling -
Extra notes:
Other Fractures You May see:
Colles Fracture: Forearm fractures account for 55% of childrens fractures. May result in dinner
fork deformity.
147
Scaphoid fracture: If suspected (e.g. pain in anatomical snuff box) but not visible on initial X
ray then put in plaster and re Xray after 2 weeks
Ankle Fracture: High impact, will always require treatment, from plaster of paris backslab to
internal fixation.
148
Anterior Shoulder Dislocation: Trauma, pain, loss of movement and shoulder contour.
Objective 4.3 Investigations
By the end of Phase II students should be able to:
• recognise the radiological features of fractures in general and the specific features of
the commoner fractures seeking advice where necessary
• recognise the radiological features of subluxation and dislocation
149
4.4
1. A 25 year old man trips on a tree branch as he walks down a steep incline and falls
awkwardly. He immediately experiences severe pain in his left leg and is unable to
weight bear on standing.
a. State 4 clinical features of a fracture
Pain
Loss of function
Tenderness
Deformity
Swelling
Crepitus (grating of the broken ends)
b. You check his airway, breathing and circulation and find that he is not
shocked. You find that there are no abrasions to the skin overlying the
fracture. What would be your first 4 steps in the immediate first aid care of this
fracture?
Step 1
Immobilise the fracture
Step 2
Remove tight clothing such as boots, or anything that may
impede circulation
Step 3
Check circulation, sensation and motion beyond fracture site
Step 4
Give pain relief
c. What is the main investigation of choice?
Plain radiograph left leg
d. What 2 features are you looking for on this investigation of choice?
Alignment (displacement, angulation, rotation)
Bone pathology
( If open wound, look for trapped air and foreign body)
e. You find that there is a closed transverse fracture to the lower third of this
man’s left fibula. It is poorly aligned. What should be done to manage this
fracture?
Reduction (under anaesthesia)
Fixation
Immobilisation
150
f.
‘Fracture disease’ follows immobilization – what 3 clinical features are
associated with this condition?
Muscle atrophy
Osteoporosis
Stiff joints
g. Post-operatively this man is doing well and is about to be discharged home
when he experiences pain in his left calf. What are you concerned this may be
a symptom of?
Compartment syndrome
h. And signs would you look for to ascertain this diagnosis ? Name 3.
(3 of the following)
Redness
Skin mottling
Blisters
Swelling
Pain on passive muscle stretching
i.
What is the pathological process behind this condition? (5 points) and what is
the surgical management for this condition?
Vicious cycle of pressure (from limbs own dead weight when immobile)
 vascular occlusion  hypoxia  necrosis  increased pressure
Surgical management:
j.
Fasciotomy
He recovered and was later discharged home. This very very unlucky man
suddenly starts to experience breathlessness 10 days after his accident. What
is ur differentials? (name 2)
Fat embolism
Pulmonary embolism
k. choose one of the above differentials you have mentioned, what other signs
might you be looking for? Name 3.
For fat embolism:
Dyspnoea
Confusion
151
seizures
Coma
pyrexia
Hypoxia
For pulmonary embolism:
Dypnoea
Tachycardia
cyanosis
Raised JVP
Pleural rub
hamoptysis
l.
Thankfully, this man makes a full recovery and on follow up his check plain
radiograph shows a well healed fracture. What are the pathological steps that
lead to bone healing? 5 points
Haematoma formation (1) vascular granulation tissue  subperiosteal
osteoblast stimulation  bone matrix  endochondral ossification 
deformable woven bone (callus) (1) Replacement by lamellar bone (1) 
Remodeling (1) Fracture union (trabecular) (1)
m. What 5 factors may delay or even arrest the repair of a fracture?
Movement
Interposed soft tissues
Gross misalignment
Infection
Pre-existing bone disease
4.4
Fractures-general
152
By the end of Phase II students should be able to:
• manage the immediate first aid care of a fracture
• describe when the possibility of fracture should be suspected and request appropriate
radiological investigations
• apply a Plaster of Paris backslab to upper and lower limbs
• describe to a patient the management of the commoner fractures
• describe how the processes of healing may be promoted or retarded
• recognise the possibility of acute compartment syndrome and fat embolism
153
4.5
Mrs OL is a 70 year old lady that has been brought to the Emergency department. She
has had a fall.
She complains of pain in her right leg and right wrist.
Question 1:
What signs may be found if her neck of femur is fractured? [2]
External rotation and leg shortening.
Question 2:
Outline some tests that should be conducted prior to surgery?[3]
X-Rays (especially lateral views), bloods (FBC and U&E).
Look at this investigation.
Question 3:
What is the name and resulting deformity of this common fracture in postmenopausal
women?[2]
Colles fracture and dinner fork deformity.
Question 4:
Aside from increasing age name 4 other causes of osteoporosis? [4]
Anorexia, alcoholic, steroid use, Cushings disease, hyperparathyroidism,
thyrotoxicosis, myleoma, primary biliary cirrhosis, rheumatoid arthritis.
154
Question 5:
Name, aside for osteoporosis, two causes of pathological fractures?
Pagets disease (increased bone turnover associated with hypernucleated ostroclasts,
causing bone remodelling, enlargement and weakness) and secondary deposits
(common cancers include breast, lung, thyroid, kidney, prostate, cervix, bowel, and
from multiple myeloma).
Extended Question:
What is the blood supply to the femoral head and why is this relevant in this case?
Lateral circumflex femoral artery and medial circumflex femoral artery and the artery
to the femoeal head which is useless in older people. That is why a fracture can
disrupt the arterial supply and lead to avascular necrosis of the femoral head.
N.B: Bits I couldn’t fit in HRT and menopause, intertrochanteric and intracapsular fractures.
4.5 Fractures-the Elderly
By the end of Phase II students should be able to:
• recognise those fracture patterns particular to the elderly
• recognise the underlying aetiological factors predisposing to fracture in the elderly
• request appropriate radiographic examinations
• recognise the possibility of significant underlying disease which may influence definitive
treatment of the fracture and initiate appropriate investigation
155
4.7
27 year old male presents in A&E having sustained a compound fracture of the tibia
bone following an RTA.
1. What is his immediate management?
Airway- check airway patent. If not, try jawthrust or headtilt and chin lift. Insert geudal
airway if needed or other airways (ET, tracheostomy) if it is severely compromised.
Breathing- check RR, pulse oximeter to check O2 sats.
Circulation- check BP, pulse, capillary refill.
2x
IV cannula insertd and infuse fluid
(type of fluid depends on hospital and severity of the patient) if needed.
2. a)
Outline the treatment/ management of compound fractures.
Emergency theatre within 6 hours. The wound should be covered (and compressed)
with an antiseptic dressing and the limb immobilised using a splint (whilst waiting for
theatre). (Injury should be photographed so repeated examination is avoided). If there
is gross deformity, manipulation under sedation may be needed before splinting. The
neurovascular status must be checked before and after this is carried out. In theatre,
the wound is thoroughly washed out and any contaminated or dead tissue is debribed.
This is to avoid infection and better union for healing. The fracture is assessed and
stabilised (usually by internal or external fixation)
b) Which 2 drugs should you consider
broad spectrum antibiotics (such as cefazolin, or clindomycin if there is
pencillin allergy)
tetanus booster should be given
3. Eight hours later he is complaining of continued pain in his lower leg and his
posterior tibialis and pedis dorsalis pulses are absent. Write a note on your
diagnosis and management.
Compartment syndrome. As pressure rises within a fascial compartment in the limb
(due to e.g. haemorrhage), the capillary blood flow to the tissues decreases.
Ischaemia occurs when the capillary pressure is less than the compartment/ tissue
pressure. Management: Elevate the limb, remove all bandages and split the cast all
156
the way to the skin. If this fails to relieve the pain, remove the plaster. If symptoms
continue then a fasciotomy maybe needed to relieve the pressure.
4. What are the immediate, early and late complications of a compound fracture?
Immediate
Haemorrhage
Neurovascular and visceral damage
Early
Compartment syndrome
Infection (worse if associated with metal work)
Late
Problems with union (delayed, non- and malunion)
Thromboembolic events (prolonged immobilisation)
Avascular necrosis
Sudek’s atrophy / reflex sympathetic dystrophy
Myositis ossificans
Joint stiffness
Growth disturbance
5. Write a note on the physiological process of bone healing.
5 stages:
Bleeding into the fracture
An inflammatory reaction is set up and granulation tissue forms
The cells proliferate and early bone and cartilage form – callus
This then consolidates – as woven bone which then transforms to stronger lamellar
bone
The bone then remodels to the normal stresses its placed under
4.7
Open Fractures
157
By the end of Phase II
the student should be able to:
• recognise an open fracture
• apply the principles of immediate stabilisation and wound coverage
• prescribe tetanus toxoid and antibiotic appropriately
• have an understanding of the importance of debridement
• act as part of the team in the definitive care for these fractures
• recognise the possibility of complications such as chronic osteomylitis
158
4.7
A 32-year-old man is brought by paramedics to A&E after being involved in a
motorcycle accident; impact occurred at 65 miles per hour.
(ABCs) assessment, x-rays of the lateral C-spine, chest, and pelvis have been obtained.
No injuries are detected. The patient is hemodynamically stable.
Upper limb examination normal.
Examination of the left hip, knee, and ankle is normal, and the distal neurovascular
exam is intact.
Examination of the right lower extremity reveals a 12-cm break in the skin, with exposed
bone and periosteum on the anteromedial aspect of the leg. There is an obvious
deformity of the leg.
An anterior-posterior and lateral x-ray of the tibia and fibula is taken. The patient has a
midshaft tibia/fibula fracture with moderate comminution.
How would you dress the wound?
A sterile dressing such as dilute Betadine should be applied in A&E and should not be
removed until the patient arrives in surgery. Multiple examinations of the wound
should be avoided to prevent nosocomial contamination and infection.
Would you start him on antibiotics or wait until he goes to surgery?
Antibiotics should be started as soon as possible. At our institution, a first-generation
cephalosporin such as cefazolin 1 g every 8 hours and an aminoglycoside such as
gentamicin 80 mg every 8 hours are the antibiotics of choice. At other institutions,
cefazolin, vancomycin, and gentamicin, alone or in combination, are used.
The patient's last tetanus booster was 7 years ago and he completed the initial
159
series as a child. The current wound appears dirty.
What would you do now?
In this case, a tetanus booster is warranted (Td 0.5 cc IM).
What is the most important aspect of caring for open fractures?
Debridement of all necrotic, nonviable tissues including skin, subcutaneous tissue,
muscle, tendon, and bone are crucial to the successful management of open
fractures. If tissues are devitalized, antibiotics and nutrients, which are important in
fighting bacteria and in wound healing, will not reach them. The devitalized tissues
will become necrotic and become a nidus for infection.
What are the two forms of fixation avaliable to stabilise a fracture of this sort?
Exteranal fixation - An external fixator is indicated for bad soft tissue injury with or
without gross contamination, because dirt can enter the canal during nailing.
Internal fixing (intramedullary nail)- An intramedullary nail is ideal because it will not
cause further soft tissue damage. The downside of a nail is that you disrupt the
intramedullary blood supply of bone, which could have an effect on the healing of the
fracture.
4.7
Open Fractures
By the end of Phase II
the student should be able to:
• recognise an open fracture
• apply the principles of immediate stabilisation and wound coverage
• prescribe tetanus toxoid and antibiotic appropriately
• have an understanding of the importance of debridement
• act as part of the team in the definitive care for these fractures
• recognise the possibility of complications such as chronic osteomylitis
160
4.8
Tom is 35-year-old man who presents to A&E with an injured knee. He was tackled
whilst playing football, and fell twisting his knee. It started hurting immediately, and has
since become swollen; he is finding it difficult to weight bear.
i)
Describe the structures that help to stabilise the knee.
Anterior - the tendon of quadriceps femoris
Lateral - the ilio-tibial tract + tendon of biceps femoris
Posterior - popliteus + semimembranosus
Medial - tendons of sartorius, gracilis + semitendonosus.
Or
Static stabilisers = intrinsic static stabilisers are the anterior + posterior cruciate ligaments.
Extrinsic static stabilisers are the medial + lateral collateral ligaments.
Dynamic stabilisers = Quadriceps – rectus femoris
Vastus lateralis/medialis/intermedius
Hamstrings – Biceps femoris
Semitendonosus
Semimembranosus.
ii)
In an injury resulting from a side impact on the knee, how might the soft tissues
of the knee joint be affected?
May result in an ‘unhappy triad’ of injuries, namely a torn anterior cruciate ligament, a
torn medial/tibial collateral ligament and a torn medial meniscus.
iii)
What features of the menisci and ligaments make the above set of injuries more
likely?
The medial collateral ligament is firmly attached to the medial meniscus, which means
that frequently tearing of the ligament also results in tearing or detachment of the
meniscus.
Meniscal tears more commonly involve the medial meniscus as it is more firmly fixed to the
tibia, compared to the lateral meniscus.
161
iv)
You examine the knee; name two tests that you would use to investigate
whether an effusion was present in Tom’s knee.
Patella tap test – if there is a substantial effusion then the patella will spring back
against your fingers.
Bulge sign – useful for smaller effusions.
v)
You continue your examination with the following tests – describe in the case of
Tom, what you might expect to find on examination?
McMurray’s test? If there is a cartilage tear, its engagement between the tibia and
femur during this manoeuvre leads to severe pain, and sometimes locking.
Anterior draw test? Rupture of the anterior cruciate ligament will allow the tibia to slide
anteriorly on the femur to a greater extent than on the unaffected side.
Posterior draw test? As the posterior cruciate ligament in unlikely to be affected in this
injury, a positive posterior draw test is unlikely.
vi)
You decide that Tom has indeed ruptured is anterior cruciate ligament, and torn
his medial meniscus – which surgical procedure may he benefit from? Explain
to him what it involves and what the complications may be.
Trimming or repair of the torn meniscus is essential; this is done via an arthroscopy.
Surgical intervention reduces the recurrent pain, locking and swelling but not the risk
of secondary osteoarthritis.
An arthroscopy involves a general anaesthetic, and the examination of the inside of
the knee joint via a camera inserted through a small incision. During the procedure
repair and trimming of the cartilage can be undertaken using ‘keyhole’ surgical
techniques. The complications include bleeding, infection, knee swelling, recurrence
of symptoms, and reaction to the anaesthetic.
vii)
What is the definition of a strain, and a sprain?
Sprain – injury to a ligament, caused by sudden overstretching. As the ligament is not
severed it gradually heals but may take several months. Should be treated by cold
compress.
Strain - Excessive stretching of a muscle resulting in pain and swelling.
What are the treatment options for patients diagnosed with a sprain or strain?
162
RICE – rest, ice, compression, elevation
Analgesia/anti-inflammatories
Use of supports
vii) A patient presents with a painful swollen knee, apart from trauma what are some of
the differential diagnoses?
Septic arthritis
Osteoarthritis
Gout
Osteomyelitis
Haemathrosis
What investigation(s) could you perform to differentiate between causes of knee
swelling?
pyrexial?
FBC + Clotting screen
Blood cultures – often positive in septic arthritis
Joint x-ray
Joint aspirate – microscopy + cultures; crystals in gout, gram stain + culture
may isolate infective organism
Serum urate – usually raised in gout >600umol/L
viii)
How may a joint become infected and what factors may make an individual more
prone?
direct injury
blood-borne infection from an infected skin lesion, or other site by
haematogenous spread.
Chronically inflamed joints are more prone to infection e.g. RA, as are artificial joints
Immunosuppressed patients – drug induced/AIDS
Elderly
Alcohol + drug abuse.
163
4.8
Soft Tissue Injuries
By the end of Phase II
the student should be able to:
• distinguish clinically between strain, sprain, and rupture of ligaments and muscles
• assess traumatic knee effusions
• initiate investigations in patients with soft tissue injuries
• initiate management of soft tissue injuries
• recognise the common underlying aetiological factors predisposing to infection in soft
tissue injuries and wounds
• identify infection in soft tissue injuries
• request appropriate radiographic examinations and other investigations as required to
assess the extent of soft tissue infections
• recognise the possibility of a gas-forming infective organism being present, the
significance of this to the patient's health and the principles of investigation and
management of this infection
• assess skin lacerations, including the recognition of foreign bodies, perform simple
suturing under local anaesthesia and dress wounds
• initiate surgical management of soft tissue injuries
164
4.9
Mr Willow, 25 years old presented in A&E having sustained a chain saw injury to his left
hand.
a) What further questions would you ask?
When, where, how?
Dominant hand?
Occupation?
Hobbies?
Previous hand injuries and disability?
b) On examination list 3 features you would look for on;
i.
Inspection
Hand posture, where the lacerations are, what is underneath it (tendons, vessels,
nerves), is hand/finger perfused? Is there soft tissue loss?
ii.
Palpation
ulnar and radial pulses present,
is hand/fingers cold,
is sensation normal distal to injury.
range of movement
From the examination you suspect he has injured a flexor tendon in his hand.
c) Name the two tendons you are concerned about?
flexor digitorum superficialis
flexor digitorum profundus
d) How could you test the integrity of these tendons?
FDS inserts into middle phalanx, FDP inserts into distal phalanx.
Failure to flex the DIP joint against resistance indicates divided FDP. If this is intact
165
but flexion at PIP joint is affected then division of FDS.
You have also noticed he has a laceration at the wrist.
e) Which nerves are you concerned he may have damaged?
Median
ulnar
f)
Give the clinical findings for each nerve stated if injured.
median – unable to oppose thumb (paralysis of thenar muscles), fine control of 2 nd
and 3rd is impaired (first two lumbricals impaired). APE HAND on trying to make a fist.
Sensory lost over thumb and adjacent two and half digits.
ulnar – difficulty making a fist because of paralysis of most intrinsic hand muscles
(these muscles put fingers into a position of flexion at MCP and extension at
interphalangeal joints) and unopposed long flexors. CLAW HAND . Sensory loss over
medial one and half digits.
g) What clinical signs would suggest a vascular injury?
Pulselessness, temperature, colour, capillary refill, venous return, swelling. Note
absent distal pulsation or pulsating haematomas.
Consider the management of this patient
h) What are the indications for surgical repair? Name 3
tendon injury (open). Nb. Closed rupture of for example extensor tendon (mallet finger)
can often simply be splintered.
nerve injury
i)
Outline the need for hand rehabilitation to the patient,
166
Rehabilitation involves protected mobilization allowing early movement without
excessive stress on the tendon repair. Wear a splint. It restores gliding function,
minimising adhesions and increases tensile strength. Greatly aiding restoration of
function.
4.9
Limb Injuries
By the end of Phase II
the student should be able to:
•
diagnose nerve and tendon injuries in the hand
•
identify injuries needing operative repair
•
discuss
•
identify vascular injury in the upper and lower limbs
•
identify division of tendons in the upper limb
rehabilitation in broad outline with the patient with the injured hand
167
4.10
You are a member of an air ambulance team attending to a 28 year old man with multiple
injuries at the site of a road traffic accident.
1. State, in order of priority your measures to ensure patient survival until transfer to a
trauma centre
Ensure airway patency and adequate ventilation by administering supplementary
oxygen.
Intubate and ventilate manually if necessary
Immobilise cervical spine and place patient on a long spine board.
Restore circulatory blood volume by crystalloids, plasma expanders and colloids as
necessary with pulse, BP, JVP monitoring
Commence head injury observations.
Alleviate pain with narcotic analgesics
(5 marks)
2. Whilst you are attempting to stabilize the patient, he becomes increasingly
breathless.
i) State the two probable causes for his progressive respiratory failure
Tension Pneumothorax
Haemothorax
(2 marks)
ii) List the clinical signs you would elicit to confirm your diagnoses.
Cyanosis
Tracheal shift
Hyper-resonance or dull to percussion, with diminished or absent breath sounds in
the chest
Evidence of injury to chest wall on gently springing the rib cage.
(4 marks)
iii) What would be your immediate management of a tension pneumothroax?
168
Wide bore cannula incision in the mid clavicular line, 2 nd intercostals space on the
affected side.
(2 marks)
iii) State one blood investigation that would be particularly useful to monitor his
condition (if you had available facilities).
Arterial Blood Gases
(1 mark)
3. The patient is trapped in the wreckage of his vehicle and his transfer is delayed by
three hours.
i) Discuss your supportive measures.
Prevent hypothermia, administer oxygen and support ventilation.
Set up iv infusion and maintain normal blood pressure
Maintain constant verbal and tactile communication and reassure the conscious
patient.
Treat life and limb threatening emergencies as facilities permit.
(4 marks)
ii) What is meant by “the golden hour”
The golden hour is the 60 minutes after the initial trauma when fairly severe injuries
can still be treated and lives saved.
(2 marks)
Extended question:
Draw and label a chest drain.
(5 marks)
169
4.10
Multiple Trauma
By the end of Phase II
•
the students should be able to:
identify the basic principles of resuscitation in the multiply traumatised patient according
to ATLS guidelines
•
be aware of the injury severity grading systems
•
describe the importance of the 'golden hour'
•
identify the range of investigative procedures in a patient with multiple trauma such as
radiographs, peritoneal lavage and urethography and be aware of the indications for
these
•
identify the indications for immediate life support procedures such as endotracheal
intubation, needle thoracocentesis or insertion of chest drain and drainage of cardiac
tamponade
170
4.11
Mr Segaren, a 25 year old jack the lad, is brought to the A&E department having
sustained a closed head injury from a blow to the back of the head outside Evolve in
Leamington.
1. i) What is meant by “coup” and “contrecoup” injury with reference to closed head
injuries?
Coup injury is damage to the brain at the site of the blow.
Contrecoup injury represents damage to the brain on the opposite side to the blow
due to brain movement within the rigid skull.
(2 marks)
ii) What is the difference between primary and secondary brain injury?
Primary brain injury is damage that occurs to the brain immediately as the result of
trauma and is directly related to the amount of force applied to the head.
Secondary brain injury results from complications associated with the initial trauma.
(2 marks)
iii) List four complications that can contribute to secondary brain injury.
Respiratory complications eg hypoxia, hypercapnia, airway obstruction)
Hypovolaemic shock
Intracranial bleeding
Cerebral oedema
Epilepsy
Infection
(4 marks)
2. i) you asses his level of consciousness.
He opens his eyes only in response to pain,
he is making random words but no conversational exchange and he flexes his limbs on
painful stimulus. What does he score on the Glasgow coma scale?
3 points for motor response – flexor response to pain
3 for inappropriate speech
171
2 for eye opening in response to pain
total of GCS= 8.
ii) Assuming he is unconscious, list the physical signs you would elicit to establish the
extent of the intracranial injury
Blood pressure, pulse and respiration
Pupillary sizes and reflexes
Reflexes – Babinski
Fundoscopy for papilloedema
(4 marks)
iii) List the types of intracranial bleeding that may be present and state one
investigation that would demonstrate the lesion.
Extradural / Subdural / Subarachnoid / intracerebral
CT/MRI head scan.
(2 marks)
3. Mr Segaren is found to have an extra-dural haematoma.
Write short notes on your
management of him. (5 marks)
Protect the airway
Exclude cervical spine injury
Quarter hourly obs of BP and pulse
Intracranial pressure monitoring
Daily ABG/FBC/U&E
Mannitol infusion preoperatively
Surgical evacuation of haematoma with placement of intra-cranial pressure
monitoring device.
Postoperative antibiotic therapy.
(5 marks)
4.11
Head injury
By the end of Phase II
students should be able to:
172
•
make an initial assessment of the patient taking into account the mechanism of injury
•
calculate the Glasgow coma scale
•
participate in the management of head injury and recognise the importance of
prevention of secondary brain ischaemia
173
4.12
Whilst working in the Paediatric Assessment Unit, a 5 year old boy attends with
extensive scalding on his lower limbs and buttock area.
How would you estimate the extent of the burn?
Chart (Lund & Browder) as appropriate for age. (1)
Briefly describe the appetence of 1st, 2nd and 3rd degree burns. (3)
1st – minor, painful erythema
2nd – painful, blistering, partial thickness, non-blanching
3rd – White, brown or black. Absent sensation. No blisters.
They are extensive 3rd degree burns in this child.
Name 2 lab investigations you wish to carry out (2)
FBC
174
U&E
Glucose
Group and Save/ X-match
Urinalysis
Coagulation
Name 3 things to be included in your initial management (3)
Analgesia e.g. morphine
Anti- emetic e.g. Cyclizine
Ensure airway patent/protected
Oxygen
IV fluids
Antibiotics
Tetanus prophylaxis
Urinary catheter
Mention 3 complications of serious burns in any patient (3)
Hypovolaemic
Renal failure
Sepsis/toxic shock
Stress Ulcers
Pulmonary dysfunction
Cardiac arrest
Post-traumatic stress
Scarring/Keloid formation
3 Days after admission, the patient become pyrexial, hypotensive and complains of
nausea.
What is the likely reason for this? (1)
Toxic shock syndrome/Sepsis
What is the organism most commonly responsible? (1)
Staphylococcus aureus
Extended Question
What may make you suspect non-accidental injury in a child in terms of burns? Mention
one (1)
175
Explanation unlikely
Late presentation
Other injuries also present
Scalds on the buttocks, lower limbs and hands
Small, circular (cigarette burns)
4.12
Burns
By the end of Phase II
students should be able to:
•
assess the extent and depth of a burn
•
plan appropriate investigations for a patient with severe burns
•
initiate immediate management of a patient with burns, recognising the possible
complications of burns, including airways obstruction and the occurrence of toxic shock
syndrome in children
•
be aware of burns as a form of non-accidental injury
•
discuss with patients the physical and mental effects of burns
•
discuss management of burns with physiotherapists and occupational therapists
176
4.13
A 78 year old lady presents with hip pain which she has had for many years and has
worsened over the past few months.
She has pain in the hip, which is typically worse
at the end of the day and after doing her housework and other activities.
She also
finds that she has stiffness in the morning but this is better by the end of the day.
These problems also severely affect her mobility and gait and have made her prone to
several falls in the past.
1) The following radiological pictures show the classical features of a particular
condition.
Name the condition and describe the major radiological changes that occur
with this condition (5 marks).
Osteoarthritis
Reduced joint space
Marginal osteophytes
Sclerotic bone changes
Cyst formation
2) What signs are found on examination?
Joint tenderness
Crepitus on movement
177
Limitation of range of movement
Joint instability
Joint effusion and variable levels of inflammation
Bony swelling
Wasting of muscles
3) Briefly outline the pathphysiological changes associated with the condition (8
marks).
The essential feature is degeneration of the cartilage:
Fibrillation (cracks in the surface cartilage)
Loss of cartilage matrix
Chondrocyte proliferation and death
Fibrous repair tissue fills the cracks in the cartilage
Subchondral bone cysts form from exposure to synovial fluid pressure
Loss of cartilage with eburnation (ivory like thickening and polishing) of bone surface
Osteophyte formation (Heberden’s nodes – osteophytes at the DIP)
Synovitis caused by fragments of cartilage in joint space.
4) What typical investigations would you do for this disease?
Blood tests.
There is no specific test; the ESR and CRP are normal.
Rheumatoid
factor only when the damage is advanced.
X-rays. As above.
These are only abnormal when the damage is advanced.
MRI. This demonstrates early cartilage and subchondral bone changes.
Arthroscopy. This reveals early fissuring and surface erosion of the cartilage.
5) Describe the major treatment options available?
Physical measures
Weight loss and exercise for strength and stability.
Walking aid held in contralateral hand in hip arthritis and in ipsilateral hand if it’s the
knee.
Regular exercises (e.g. quadriceps exercises in knee OA) and keep active.
178
Medication
Should only be used with severe disease.
Simple analgesia first e.g. paracetamol
and if no good NSAIDs but prescribe with adequate explanation of side effects and
can be left off on good days.
Surgery
Total hip replacement arthroplasty for severe OA, reduces pain and stiffness and
greatly increases function.
179
Q1)
The clinical and radiological pictures shown in picture 1a and 1b) and picture 2a
and 2b) of hands show two different conditions.
Picture 1a
Picture 1b
180
Name the condition in picture 1a) and 1b) above and describe some of the salient
features.
Osteoarthritis of the hands
Hand joint involvement showing bony thickening of the DIP (Heberden nodes) and
PIP joints (Bouchard nodes).
Picture 2a
Picture 2b
The above hands (2a and 2b) are those of a 65 year old lady who has had this problem
with her hands for many years and describes how her hands are often very swollen,
painful and stiff, especially in the morning.
This eventually involved her larger joints
i.e. ankles, knees, hips, elbows, shoulders, cervical spine and TMJ.
debilitated with this condition.
She is thus very
She also suffers with breathing difficulties and has
problems with her eyes.
Q2) Name the condition in picture 2a) and 2b) above and describe some salient features
that you find in the hands with this condition?
Rheumatoid arthritis of the hands
181
Sausage shaped fingers and MCP joint swelling.
Later on, ulnar deviation and volar
subluxation (partial dislocation) at MCP joints Boutonniere and swan-neck deformity
of the fingers or z-defromity of the thumbs. The wrist subluxes and the radial head
becomes prominent (piano-key).
Extensor tendons in the hand may rupture and
adjacent muscles waste.
Findings on xrays:
 Soft tissue, juxta-articular osteoporosis; ↓joint space.
Later on:
bony erosions (+/- subluxation) and (+/- complete carpal destruction).A
Q3) Briefly outline the pathophysiology of this condition.
The essential feature in joints is chronic nonsuppurative inflammation of the synovial
membrane that can destroy the articular cartilage, bone, and adjacent tissues.
Chronic inflammatory synovitis
Synovial membrane is thickened with villous outgrowth (hypertrophic villous synovitis)
Proliferating synovial tissue extends over the surface of the articular cartilage as
“pannus”.
Pannus covers the cartilage, eroding into cartilage and bone
Neutorphils are prominent in the joint fluid
B and T lymphocytes (especially CD4 helper cells) are found in the synovial stroma
Synovial antigen presenting cells (dendritic cells and macrophages) are activated
Pannus comes to resemble a piece of highly vascular, active, cellular lymphoid tissue
Osteoclastic activity in underlying bone results in osteoporosis and bone erosions
Inflammation of tendon sheaths and other synovium-lined tissues.
Q4)
Outline the major investigations for this condition.
No one specific laboratory test is diagnostic of RA.
The diagnosis is made primarily
182
on the clinical features (an arthritis involving small peripheral joints) but blood test can
be done with the following features:along with:
↓ HB (Normochromic or hypochromic anaemia, and other indicators of a chronic
systemic acute phase response e.g. elevated globulins, decreased albumin).
 ESR
 CRP
Rheumatoid factor (often negative in the start and becoming positive in 80%).
NB
also raised with other conditions.
↓WCC
ANA (+ve 30%)
Radiology: Plain radiographs, occasionally CT and/or MRI
Biopsy:
Q5)
Synovium, skin, nodules, vessels, lymph nodes – rarely necessary.
Outline the treatment options associated with this condition
The aim is to minimize symptoms and improve prognosis
Multidisciplinary approach
Regular exercise
Physiotherapy
Occupational therapy
Household aids and personal aids (orthoses) e.g. wrist splints.
Intra-lesional steroids (joint injection)
Surgery – to improve function, not for cosmetic effect.
Simple analgesics or NSAIDs
( no affect on long term outcomes)
2nd Line DMARDs (disease modifying anti-rheumatic drugs)
Start these if there is persisting synovitis for more than 6 weeks.
Sulfasalazine is the
first choice in the UK
183
Sulfasalazine: s/e nausea, headaches, ↓marrow, sperm count and hepatitis.
Methotrexate: s/e nausea, diarhoea, cough, SOB, cirrhosis.
Ciclosporin:
s/e nausea, tremor, gingival hypertrophy, hypertension.
s/e ↓marrow, proteinuria.
Gold:
Azathioprine:
Penicillamine:
s/e ↓marrow, nausea,  LFTs, oncogenic.
s/e ↓marrow, proteinuria.
Hydroxychloroquine: s/e permanent retinopathy with high doses, tinnitus, headache.
Adjunctive therapy with corticosteroids
For severe systemic illness, intermittent and troublesome mono/pauciarticular
inflammation +/- vasculitis.
e.g.methylprednisolone
inflamed joints.
Steroids may be given as intermittent pulses
80-120mg im or 500-1000mg iv or can injected into
Or can be used until the DMARDs take effect.
Have numerous s/e
especially osteoporosis and potential for opportunistic sepsis.
Biological agents
Target pro-inflammatory cytokines e.g. TNF and IL1
Chimeric monoclonal antibodies (infliximab)
Fc fusion proteins with soluble TNF receptor (etanercept)
Expensive but highly effective
More recently recombinant IL-1 receptor antagonists are also an effective agent.
184
4.13
Investigations
By the end of Phase II
•
students should be able to:
interpret the major radiological changes of osteoarthritis of the hip and of rheumatoid
arthritis of the hand
•
request and interpret routine investigations for auto-immune disease
•
relate these changes to the underlying pathology
185
4.14
A 75 year old gentleman presents in A&E complaining of an excruciatingly painful left
knee. The pain came on suddenly the previous night and he has never suffered from
similar symptoms before. There is no recent history of trauma to the knee.
1. What is your differential diagnosis?
Crystal arthropathy (e.g. gout, pseudogout)
Septic arthritis
Monoarthritic presentation of a polyarticular disease (e.g. OA, RA)
2. On examination he has a red, hot, swollen knee with significantly reduced range
of movement due to pain. You decide to aspirate joint fluid. What 3 other
investigations would you perform on him?
Take blood for: FBC (looking for ↑WCC), ESR/CRP (inflammatory markers), U&E
(renal status important in gout), serum urate level
Blood cultures (if sepsis is possible)
Knee X-ray – look for erosions, calcification, widening/loss of joint space, changes
in underlying bone (e.g. sclerotic areas, osteophytes)
3.
Joint aspiration reveals turbid, yellow synovial fluid, rich in neutrophils. No
organisms have been identified in culture. Microscopy reveals negatively
birefringent needle-shaped crystals. What is the diagnosis and how would you
manage this condition in the acute setting?
Goutut.
NSAIDs (e.g. ibuprofen, naproxen) – analgesic/anti-inflammatory effect [If NSAIDs
are contra-indicated, e.g. due to peptic ulcer, give colchicine PO]
Joint aspiration (reduces pressure)
NB: NSAIDs & colchicine are problematic in renal failure – steroids can be very effective
but not without side-effects!
186
4. What are the predisposing factors for the above condition?
chronic renal disease
drugs (e.g. thiazide diuretics, low-dose aspirin)
hypertension
lead toxicity
alcohol
genetic (e.g. glucose-6-phosphate deficiency)
idiopathic gout
increased purine turnover (myeloproliferative disorders, e.g. polycythaemia vera –
lymphoproliferative disorders, e.g. leukaemia – others, e.g. carcinoma)
genetic (e.g. Lesch-Nyhan syndrome)
5.
How would you prevent similar attacks in the future?
Avoid purine-rich foods (e.g. offal, oily fish), obesity & alcohol excess
Consider long-term allopurinol (↓ serum urate) but not until 3 wks after an attack
know allopurinol’s mode of action, dose, side-effects!
Control hypertension, avoid diuretics and aspirin
E. If microscopy of the synovial fluid had revealed positively birefringent
rhomboid-shaped crystals what would the diagnosis be? What are the risk
factors for this condition?
Pseudogout.
Risk factors:
Old age
Dehydration
Hyperparathyroidism
DM
Acromegaly
Haemochromatosis
Wilson’s disease
↓ PO4/ ↓ Mg
187
A 35-year old gentleman presents in A&E with an acutely painful knee. The pain came
on over the last 24 hours and he is also feeling feverish and generally unwell. His
doctors suspect that he might be suffering from septic arthritis.
1. What are the possible micro-organisms responsible for this condition?
(in order of decreasing prevalence)
Staph aureus
Streptococci (mainly – Strep pneumoniae, Group A strep)
Gram –ve bacilli (mainly – H. influenzae, E. coli)
Neisseria gonorrhoea
Anaerobes
2. What are the 3 possible routes by which the joint becomes colonised?
Direct inoculation (e.g. during joint surgery/aspiration)
Spread from infected peri-articular tissue (e.g. spread from osteomyelitic focus)
Haematogenous spread (due to transient/persistent bacteraemia, e.g. skin/lung/bladder
infection)
3. Name 3 host factors that can predispose to non-gonococcal septic arthritis
Increasing patient age
Decreased immunocompetence (e.g. diabetes)
Pre-existing joint disease (e.g. RA patients)
4. You decide to aspirate synovial fluid from the patient’s knee and send it for
culture. How would you treat his condition while awaiting culture results?
Ask for advice from orthopaedic surgeons and microbiologists!
Start empirical antibiotic treatment depending on suspected organism (e.g. for
staphs – flucloxacillin & benzylpenicillin IV until sensitivities are known)
Rest affected joint (no bearing of weight)
188
Encourage early joint mobilisation as symptoms subside – vigorous physio!
4.14
Acute onset joint pain
By the end of Phase II
the students should be able to:
• recognise the symptoms of infection of bone or soft tissue
• initiate diagnostic procedures and treatment of the condition
• recognise the possibility of septic arthritis
• recognise the indications for surgery
• discuss the prognosis and rehabilitation with the patient or parents
• recognise acute and chronic gout and initiate management
189
4.15
Mr L is a 65 year-old man who presented to his GP with unremitting chest pain that is
particularly worse at night.
His worried wife, who accompanied him, has also noticed
that he has lost weight recently.
His significant past medical history comprises of
successful management of prostatic carcinoma by brachytherapy 3 years ago.
a.
What is brachytherapy?
Radiotherapy administered by implanting radioactive wires or grains into or close to
a tumour
b.
Name 3 differential diagnoses for his presenting complaint
rib tumour, primary or metastatic
muscular strain
rib fracture
The GP was concerned that Mr L may have bone metastases from his prostatic
carcinoma.
c.
Which investigations would you carry out to confirm or refute the presence of
bone cancer?
Serum alkaline phosphatase
Calcium
Prostate specific antigen
d.
Name 2 types of primary malignant neoplasms of bone
Osteosarcoma
Fibrosarcoma
Chondromas
e.
Name three treatment options for Mr L.
Analgesics
190
Anti-inflammatory drugs
Local radiotherapy
Hormonal therapy
Internal fixation of pathological fracture
4.15
Bone and Soft Tissue Tumour
By the end of Phase II
•
the student should be able to:
identify the broad pathological types of bone and soft tissue tumours which affect the
musculo-skeletal system
•
recognise the possibility of underlying bone or soft tissue tumour in patients presenting with
musculoskeletal symptoms
•
detect major abnormalities on examination suggestive of bone and soft tissue tumour
•
recognise the importance of staging and how this may be performed
•
be aware of the concepts of treatment of bone and soft tissue tumours
191
Mrs JR, a 76 year old female, presents to your GP clinic with a 2 day history of a painful
left knee.
1. List four possible causes of a painful solitary large joint
trauma
Septic arthritis
Gout/pseudogout
Gonococcal arthritis
Haemophilia
Rheumatoid arthritis
The following is found on examination
Left knee examination shows:

overlying skin red and warm

swollen, painful left knee joint

pain on all left knee movement – with diminished movements due to pain and
spasm
2. What additional features would you expect on general examination that would
suggest a infective aetiology
Swinging pyrexia
Tachycardia
3. From the clinical findings what is the most likely diagnosis
Septic arthritis
4. What predisposing factors need to be considered in any patient with this
condition?
debilitating disease
infection elsewhere such as an infected leg ulcer
rheumatoid arthritis
192
immunosuppression/corticosteroids
diabetes mellitus
recent joint surgery
joint prostheses
penetrating injury
5a) What is your immediate management?
Admit to hospital as risk of rapid joint damage
5b) What investigations should be ordered?
blood cultures
full blood count for leucocytosis
erythrocyte sedimentation rate
C-reactive protein
aspiration of synovial fluid - usually purulent with a neutrophil count above 50,000
per mm3, and low glucose concentration
anti-streptolysin O titre
Plain X-ray of left knee
What radiological findings would expect?
None at this stage, first radiological changes appear after 2-3 weeks.
5. What radiological features may you find in 2-3 weeks?
Narrowed joint space, and patchy rarefaction
6. What aetiological agents are likely to be isolated from your investigations name
2.
Staphylococcus aureus is the most common agent.
Others may include:
Streptococcus pyogenes
pneumococcus
Haemophilus influenzae, although most likely in those under three years
gonococcus
gram negative rods, such as E. coli, Pseudomonas
193
All the above are primary causes, but may be secondary to osteomyelitis of an
adjacent bone.
7. What immediate antibiotic agents would you expect to be administered?
Should check local protocol, and most recent BNF, but likely agents include
clindamycin alone or flucloxacillin and fusidic acid
if Haemophilus influenzae then give amoxicillin or cefuroxime
8. What complications could arise from this condition?
chondrolysis
septic dislocation
avascular necrosis
shortening
late degenerative change
Objective: painful solitary large joint
194
4.16
A 29 year old woman presents to medical outpatients with a 3 month history of painful,
swollen proximal interphalangeal joints associated with early morning stiffness and
malaise and lethargy.
What is the likely cause of her arthopathy? (1 mark)
Rheumatoid arthritis
Name 3 investigations that you may wish to conduct and the likely finding if it is the
diagnosis suggested in question 1? (6 marks)
Blood count –
low Hb,
high platlets,
Rheumatological marker- high rheumatoid factor,
Inflammatory markers – high ESR, high CRP
X-ray – periarticular erosion, soft tissue swelling, angular deformatity, osteoperosis
In the categories below list an associated feature which may occur in this disorder 1)
eyes, 2)skin 3) respiratory 4) cardiovascular 5)renal:
Sjogrens syndrome (dry eyes and mouth), scleromalacia
Rheumatoid nodules , vasculitic rash
Pleural effusion, rheumatoid nodules,
pulmonary fibrosis
Pericarditis, pericardial effusion
Chronic renal failure, secondary to amyloidosis or drugs
Outline the pharmaceutical medical management of this condition
Analgesia – paracetamol, NSAIDS
Anti-inflammatory – NSAIDS
Corticosteroids- prednisolone
DMARDS (Disease Modifying Agents of Rheumatological Diease) D-penacillamine, gold
salts, sulphazalazine
Immune modulatory – anti TNF (Infliximab)
Extended
195
Breifly outline the role of 2 members of the MDT in the care of this patient
Physician -patient education of disease, medical management, referral management
Physiotherapist – maintain joint mobility through exercise
Occupational therapist – evaluate impact of disease on life, advise easier ways to
perform daily tasks and design and prescribe assistive devices
Orthotist- provide and fit splints and other aids
Surgeon- perform joint replacement
Others you can mention:
Social worker,
Specialist nurse,
GP
196
An obese 67y old woman presents to her GP with a 4 month history of
increased swelling , pain and stiffness in her right knee. She suffered a
comminuted fracture of her distal right femur in a road traffic accident
15 years ago?
What is the likely diagnosis?
osteoarthritis.
List 3 risk factors for the development of this condition
Family history of OA
Herediatry disorders (Ehler Danlos)
Metabolic conditions – haemachromatosis, agromegaly
Concominant bone disorders (Pagets, Perthes)
Trauma – previous fractures
Obesity
Repeatative trauma/ occupation (footballers, atheletes, ballet
dancers)
For each of the following features place either RA (for rheumatoid
arthritis) or OA (for osteoarthritis) – for which ever condition it is
commonly associated with.
Number 10 is done for you.
1. Loss of joint space on xray
6. Bouchards node
2. Feltys syndrome
7. Initial synovial inflammation
3. HLA DR1
8. Swan neck deformity
4. Z deformity of the thumb
9. Subchondral cysts
5. Osteophyte formation
10. Ulnar deviation
RA
Answers below:
OA-1,5,6,9
RA- 2,3,4,7,8,10
Briefly outline the management of this condition
197
Conservative/supportive:
Patient education (explanation of disease and its management).
Lifestyle changes (weight loss, exercise)
Physiotherapy (maintain joint moment, increase muscle strength),
Occupational therapy to supply aids and appliances to aid mobility
Medical:
Pain relief and NSAIDS. Steroid injections
Surgical:
Debridement (removal of dead tissue within the joint)
Artherodesis (fusing a joint)
Osteotomy
Arthroplasty
4.16
Chronic Inflammatory and Degenerative Joint Disease
By the end of Phase II
the student should be able to:
• recognise rheumatoid arthritis and distinguish active from inactive disease
• describe the immunological basis of rheumatoid arthritis
• recognise the multi-system manifestations of many auto-immune processes
• diagnose and assess the severity of degenerative disease of the hip and knee
• initiate appropriate investigations
• initiate appropriate management, especially the relief of pain
• discuss with patients the range of aids to daily living that are available
• describe the referral pathways for patients to physiotherapy and occupational therapy
• recognise the need for surgical assessment
• be able to discuss with patients the treatment options available, the role of rehabilitation and
possible complications of surgical treatment
4.18
A 23 year old man who had previously fractured his leg is now on crutches.
He has
noticed that since the use of his crutches, he has experienced loss of sensation and
198
weakness in his hands.
On examination a diagnosis of radial nerve palsy is made.
Q) which actions of the upper limb would he find difficult to perform?
extension of fingers, wrist and elbow
Q) What area of sensation would be lost?
Dorsum of hand – lateral 3 ½ digits with sparing of finger tips
Q)
How did the radial nerve become affected?

Compression of nerve in the axilla by incorrect use of crutches
A 42 Year old female is seen by her GP complaining of pain and numbness in her hands.
She notices that her symptoms are classically worse at night and during the day, during
prolonged flexion of the wrist e.g typing or knitting.
Q)
What is the diagnosis ?
Carpal tunnel syndrome
Q) What are features on examination?
Wasting of thenar eminence, weakness of thumb movements
Q)
What test can be done to confirm the diagnosis?
Tinnels Test (tapping over median nerve at the wrist to reproduce the
symptoms)
Q) What are the causes of this condition?
Median nerve compression as it passes under flexor retinaculum (in carpal tunnel).
Much more common in females than in males and associations are pregnancy,
rheumatoid arthritis, hypothyroidism, acromegaly, trauma and idiopathic in post
menopausal women ?
199
Q)
What is the treatment for this condition?
Conservative – using splints
Medical – local steroid injections
Surgical – decompression of the flexor retinaculum
4.18
Nerve Compression
By the end of Phase II
the student should be able to:
• recognise nerve compression in the upper limb
• initiate investigations for a patient with nerve compression
• be aware of the role of surgery and radiotherapy in the relief of nerve compression
200
4.19
During a neonatal health check, performed the first day after birth, a baby is diagnosed
with developmental dysplasia of the hip
(DDH)
Q) Which clinical tests would be carried out to diagnose this?
Ortolani and Barlow Manoeuvre
Q) Name 3 risk factors for DDH?
Breech birth,
+ve family history,
increase in birth weight,
post maturity,
Oligohydramnios
Q)
What imaging technique can be used as a screening tool for DDH?
Ultrasound
Q)
Briefly outline the treatment of this condition?
Use double nappies – reassess in 3 wks with ultrasound
If still problem – splint hips in abduction
6 – 18 mths closed reduction – frame /gallows
before 6 yrs – open reduction (surgery)
4.19
Congenital Problems
By the end of Phase II
the students should be able to:
• detect scoliosis, fixed flexion, kyphosis, varus and valgus deformities
• detect deformity present in the knee,
hip, shoulder
and the small joints of the hand
• consider the possibility of congenital dislocation of the hip and talipes equinovarus
• perform the diagnostic procedures available at birth and early life to detect these
conditions
201
• outline the diagnosis, management and prognosis to the family
202
4.21
Mrs S is a 47 year old woman that presents to her GP. Her mother has suffered
from osteoporosis and Mrs S wants advice on how she can help prevent
osteoporosis.
Q1.
What steps can Mrs S take to help prevent herself getting osteoporosis?
Dietary – Calcium supplementation, Vitamin D supplementation
Regular Weight bearing exercise (30 mins 3 times per week)
Smoking cessation
Avoid excess alcohol
Q2.
Give four risk factors for Osteoporosis.
Hip protectors to reduce fall risk
Female sex, increasing age, family history of osteoporosis, early menopause,
smoking, excessive alcohol, immobility, lack of exercise, malabsorption, 4
prolonged steroid therapy, Cushing’s disease, Hyperparathyroidism,marks
Myeloma,
Hypogonadism in men.
Q3.
Give four treatment options for Osteoporosis.
4
Oestrogen therapy via HRT in postmenopausal women
marks
Biphosphonates
Vitamin D (Calcitriol)
Calcium supplementation
Calcitonin
Androgens for Hypogonadal men
4marks
Q4.
Osteomalacia and Osteoporosis may both present with subclinical
4
fractures. What blood tests would help differentiate osteomalacia from
marks
osteoporosis?
4
In osteoporosis, Serum alk phos, plasma calcium and serum phosphate are all
marks
normal.
In osteomalacia, serum alk phos is high, plasma calcium is low or low normal
and serum phosphate is low.
marks
4
203
4.21
Metabolic bone disease
By the end of Phase II
students should be able to:
•
recognise the circumstances where osteoporosis is likely to occur
•
be able to give advice to patients about preventing osteoporosis and outline treatment
strategies
•
recognise the circumstances when osteomalacia may occur
•
initiate investigation of osteomalacia
•
recognise the circumstances when hypercalcaemia may occur
•
initiate the management of hypercalcaemia
204
4.22
A 62 year old lady presents to her GP. She has been feeling non-specifically
unwell for nearly 3 months. She has been suffering pain and stiffness in her
shoulders, neck, hips and lumbar spine (limb girdle pattern) which is worse first
thing in the morning upon waking and makes getting out of bed difficult. For
the past 2 days she has suffered a severe constant headache affecting her right
temporal scalp. She has also noticed tenderness in this same area.
Q1.
What is the likely diagnosis?
Polymyalgia Rheumatica with concurrent Temporal Arteritis (Giant cell
arteritis/Cranial arteritis).
Q2.
Which investigations should be carried out and what is the definitive
marks
diagnostic test for this condition?
FBC may demonstrate mild anaemia
ESR characteristically very elevated
CRP raised
LFT’s – Serum alk phos and γ-glutamyltransferase may be raised
2
Definitive diagnostic test is Temporal Artery Biopsy
Q3.
How should this condition be initially managed?
4
marks
High dose Corticosteroids PO
2
Q3.
Name 3 other conditions associated with vasculitis.
marks
Rheumatoid Arthritis
SLE
Scleroderma
Polymyositis/Dermatomyositis
Goodpastures Syndrome
Inflammatory Bowel Disease
Paraneoplastic
Subacute Infective Endocarditis
3
marks
4.22
Other conditions
By the end of Phase II
students should be able to:
205
•
recognise, investigate and initiate management of cranial arteritis
•
recognise and refer cauda equina compression
•
recognise progressive cervical myelopathy
•
recognise common dermatological manifestations of rheumatic disease
206
3.1 & 5.2
Mr Bogg is an unfortunate 65 year old man who has attended the ward prior to a right
hemicolectomy for a longstanding colonic tumour.
history.
He has a longstanding medical
He has had diabetes mellitus for the past 48 years for which he is currently
using insulin and notes that he has good control and has no known complications.
He
also suffers from hypertension that he diligently checks at home and at his GP’s and
notes is within the ‘normal range’.
He is otherwise well on the day of admission.
(a) Given his medical history what ASA grade would you assign him and state what
features in his history suggest this particular grade.
What does the ASA grading
system predict? (3 marks)
Class II: “a patient with mild to moderate systemic disease that
does not limit the patient’s activities in any way”.
Known hypertensive and diabetic both of which are well controlled
and have no significant complications.
Predicts perioperative mortality.
(b) Name 3 pre-operative investigations that would be appropriate in this patient (3
marks)
Bloods (FBC, U&E, Cr, BM, HbA1c)
PA CXR
12-Lead ECG
(c) What steps would take to optimise his pre-operative glucose levels.
account (exact numerical details are not required).
Give a detailed
(7 marks)
Liaise with surgeon and anaesthetist
Put patient first on list
Refer to local policy
Stop LA insulin the night before
Setup a sliding scale (50U of SA insulin with 50mL N/saline)
Check BM every hour and adjust sliding scale
Aim for a BM of between 7 – 11mmol/L during surgery
Continue sliding scale until px eats second meal – check BM every 2
hours
Switch back to normal regimen
On the day of his operation, Mr Bogg was transferred from the ward to the anaesthetic
207
room.
Following pre-oxygenation and initial iv induction with propofol the medical
trouble
maintaining a seal and therefore the anaesthetist asked him to put in an oral
airway.
student was asked to ventilate him by bag and mask.
He was having
(d) How do you size a guedel airway in an unconscious patient? (1 mark)
An estimate of the airway size required is given by comparing the airway length with
the vertical distance between the patient’s incisor teeth and the angle of the jaw
(e) Describe the process of inserting an oral airway (2 marks)
Start with airway upside down
Place into oropharynx in this position, advance to the hard palate
Rotate the airway 180o
Fully insert until opening lies in front of teeth
(f) What is the main problem that one must warn the patient of during propofol
administration – especially if they are of an anxious disposition? (1 mark)
Often administered with local anaesthetic due to pain on injection
He was then paralysed prior to intubation.
The anaesthetist used rocuronium to
facilitate this.
(e) What is the mechanism of action of rocuronium and how could its effects be
reversed if the surgeon broke a fingernail and decided to cancel the procedure. (2
marks)
Rocuronium is a non-depolarizing competitive neuromuscular blocking agent that
competes with acetylcholine for post-synaptic binding sites.
It is commonly reversed
using an anticholinesterase such as neostigmine to overcome the block.
(f) Name 2 methods to confirm that the ETT was in the correct place.
Give 4
complications of intubation. (6 marks)
To confirm correct placement either:
ventilate the chest and watch it rise symmetrically
auscultate over the stomach during ventilation and hope not to hear bubbles with
ventilation.
208
Complications:
(Any from below)
Oesophageal intubation
Right main-stem bronchus intubation
Gastric aspiration
Airway trauma
Dental trauma
Cervical cord injury
(g) How does one prevent gastric aspiration during induction of anaesthesia?
How
does it work? What is the complication that may arise from forgetting to do this
(especially in a trauma situation)? Where would such a patient go in the post-operative
period?
(4 marks)
Use cricoid pressure
Cricoid is the only complete ring of cartilage in the larynx therefore occludes
oesophagus with application of direct anterior pressure
Aspiration pneumonia
Send to ITU for monitoring and management
5.1
Pre-operative assessment for anaesthesia
By the end of Phase II
students should be able:
• to order appropriately pre-operative blood tests, chest radiographs and ECGs.
• to recognise patients who have an increased risk of peri-operative morbidity and mortality
• to participate in the pre-operative work-up regimes for high risk patients
• to liaise with the anaesthetist in the management of a diabetic patient having surgery
5.2
General anaesthesia
By the end of Phase II
students should be able to:
• recognise the properties of some of the commoner drugs used in anaesthesia and their side
effects
• perform venous cannulation competently
• recognise some of the complications which may occur during an anaesthetic including,
hypertension, bradycardia and tachycardia, myocardial infarction, aspiration of stomach
contents
• care for an unconscious patient
209
5.3
Question 1
A post-operative patient is in recovery.
State 3 risk-factors that increase the chance of post-operative nausea and vomiting.
(3 marks)
Female gender
Obesity
Young age
Positive history
Surgery to middle ear, posterior cranial fossa, intra-abdominally, hysterectomy
Severe pain
Administration of opioid analgesics,
nitrous oxide (ketamine, isoflurane)
Hypoglycaemia, hypoxia, uraemia, hypotension.
Gastric dilatation e.g. from bag and mask ventilation
State 2 different anti-emetic drug groups and the generic name for 1 drug from each.
(2 marks)
Anti-histamine e.g. chlorphenamine, loratidine, cyclizine.
Dopamine antagonist e.g. metoclopramide, domperidone.
5HT3 antagonist e.g. granisetron, ondansetron.
Anti-cholinergic e.g. hyoscine, atropine
Where in the body can the receptors for these drugs be found?
(2 marks)
Histamine (H1) receptors, found mainly in the vestibular labyrinth.
Dopamine receptors are found in chemoreceptor trigger zone
5HT3 receptors are predominantly found in the GI tract.
Cholinergic receptors are found in the vestibular labyrinth and chemoreceptor trigger
zone
Question 2
Post-surgery a patient is sent to the ward on patient-controlled analgesia (PCA).
Name 2 advantages of PCA over infusion or i.m. injection.
(2 marks)
210
Increased patient satisfaction.
Increased ability to match the dose with the patient’s perceived pain level.
Reduced work-load for nursing staff.
Elimination of painful repeated i.m. injections.
Greater certainty of plasma opiate levels as given i.v.
Name 2 disadvantages of PCA over infusion or i.m. injection.
(2 marks)
Initial cost of pump
Not suitable for children under 5-7 years, confused, frail or elderly patients.
Potential for overdose remains.
What instructions would you give to the nursing staff regarding safe monitoring of
PCA?
(3 marks)
Regular observations, especially sedation score (neuro) and respiratory rate.
Pain score.
Written-up prescription for naloxone.
Check syringe volume with actual consumption – stop PCA if discrepancy.
Nurse on open ward rather than side room.
Prescription for breakthrough anti-emetic.
Later, you check the patient, but they are unresponsive, have a respiratory rate of 7
breaths per minute and pin-point pupils.
What do you do?
(4 marks)
Call for help
Stop the PCA
ABCs giving O2 via bag-and-mask ventilation
Give naloxone
State the generic name and dose of the reversal agent given in this situation.
(2 marks)
Naloxone, 0.8-2mg i.v. every 2-3 mins until respiratory rate is adequate, to a
maximum of 10mg.
5.3
Postoperative problems
By the end of Phase II
students should be able to:
211
•
treat post-operative nausea and vomiting
•
appreciate the likely causes and initiate management of post-operative confusion
•
recognise and initiate management of post-operative respiratory failure
•
understand the importance of effective post-operative analgesia
•
manage i.v. patient controlled analgesia and epidural analgesia and their side effects
•
prescribe strong, intermediate and minor analgesics in the post-operative period
having regard to the operation and to appropriate doses, routes of administration,
side effects and contra-indications
•
perform intravenous, intramuscular and subcutaneous injections competently
212
5.4
Question 1
Pulse oximetry is commonly used to monitor hospital patients.
Describe 2 advantages and 3 limitations of this technique.
(5 marks)
Advantages:
Small and portable
Continuous monitoring
Non-invasive
Simple to use
Fast results
Limitations:
Does not measure C02 levels
Carboxyhaemoglobin gives falsely high readings
Methaemoglobin gives falsely low readings
In respiratory failure a high FiO2 may give misleadingly high SpO2%
Inadequate pulse e.g.
hypoperfusion, hypovolaemia, hypothermia, peripheral
vasoconstriction.
Excessive patient movement can disturb
Nail varnish, nicotine and henna can prevent reading
Methylene blue (thyroid surgery) gives false low readings
In anaemia the SpO2 may be high, but there may be inadequate tissue delivery of
oxygen
Below 10kPa, a small drop in PaO2 produces a large drop in saturation reading due
to the haemoglobin dissociation curve
Readings <70% are inaccurate (machines are calibrated against healthy human
volunteers – they don’t let them get this hypoxic apparently!)
Question 2
Name 2 vessels appropriate for introducing a central venous catheter.
(2 marks)
Internal jugular vein
Subclavian vein
Basilic vein at antecubital fossa (only 60% chance of success)
213
State 4 complications that can potentially arise from central venous catheterisation.
(4 marks)
Air embolus
Pneumothorax
Scepticaemia
Local infection at catheterisation site
Arterial puncture and haemorrhage
Venous thrombosis
Thoracic duct injury (left side)
Injury to brachial plexus, phrenic or recurrent laryngeal nerve
5.4
Monitoring
By the end of Phase II
students should be able to:
•
recognise the need for central venous cannulation and the associated complications
•
measure a patients CVP using a water manometer
•
recognise the need for pulmonary artery catheterisation and its uses
•
use and interpret pulse-oximetry
•
recognise the need for cardiac monitoring
214
5.5
While working as a house officer on the cardiology ward, Mr S a 75-year-old man with a
history of ischaemic heart disease, mistakenly arrives at the ward for his outpatient
appointment. While you are showing him to the outpatient department he collapses in
the corridor with central crushing chest pain and quickly becomes unresponsive. Using
simple manoeuvres you open his airway but find he is not breathing and has no pulse.
After calling for the resuscitation team you return to him to begin basic life support until
help arrives.
1) According to the 2005 resuscitation council guidelines, at what ratio should chest
compressions to rescue breaths be given to an adult? (1)
30:2
Help quickly arrives and a defibrillator is attached to assess his heart rhythm and IV access is
gained.
A)
B)
C)
D)
2) Assuming they are all pulseless, name the ECG rhythms A-D (4)
A)
Pulseless electrical activity (PEA) / electromechanical dissociation (EMD)
B)
Asystole
C)
Ventricular fibrillation (VF)
D)
Ventricular tachycardia (VT)
3) Mr S is found to be in the same rhythm shown in C. According to the advance life
support algorithm 2005 how should this be treated? (4)
1 x defibrillator shock
215
2mins of CPR (30:2)
Adrenaline 1mg IV each cycle
Reassess rhythm
4) What other rhythm is treated this way (use the letter A-D as above)? (1)
D
5) Name one drug other than adrenaline that can be used in order to correct rhythm C
and your answer to question 4 (1)
Amiodarone
6) Name four reversible causes of cardiac arrest (4)
Hypoxia
Hypovolaemia
Hypothermia
Hyper/hypokalaemia (metabolic)
Tension pneumothorax
Thromboembolism
Toxic
Tamponade, cardiac
Eventually the resuscitation attempts are successful and Mr S returns to sinus rhythm
but does not regain consciousness. He is taken to the intensive care unit due to his
poor condition. His condition continues to deteriorate and a decision is needed on
whether to attempt to resuscitate Mr S, should he experience anther cardiac arrest.
7) Under what circumstances is it appropriate to issue a do not attempt resuscitation
order (DNAR) in an incompetent / unconscious patient? (3)
When CPR will to restart the patient’s heart and breathing - Futile
When there will be no benefit gained by restarting the patient’s heart and breathing –
Patient will die now or next time they arrest
When the expected benefits of CPR are outweighed by the burdens – co-morbidity
216
will cause poor or no quality of life
8) Who is ultimately responsible for decision-making regarding resuscitation orders for
incompetent / unconscious patients? (1)
The consultant / GP in charge of the patients care (Where care is shared consensus
is needed)
The resuscitation council 2005 guidelines can be accessed here.
5.5
Cardio-pulmonary resuscitation
By the end of Phase II students should be able to:
•
Diagnose and initiate management of asystole, ventricular fibrillation,
pulseless ventricular tachycardia and electro-mechanical
dissociation according to established guidelines
•
Operate a defibrillator competently
•
Discuss the ethical dilemmas surrounding resuscitation and how
they may be resolved
217
5.8
Miss S, a 20 year old women attends A&E and admits to taking an overdose of pills.
1. What essential questions must you ask at this point?
What have you taken? Including drugs and alcohol.
How much have you taken?
When did they take them?
Why did they take them?
2. She tells you that she has taken 16 paracetamol tablets, no alcohol, roughly an hour
and a half ago.
What is your immediate management plan?
IV access
Bloods – FBC, U&E, LFT, Clotting/INR
Paracetamol levels taken at 4 hours post OD
ABG
Activated charcoal – according to local protocol
Senior advise
3. What is the specific antidote/treatment for paracetamol overdose?
N-acetyl cysteine (Trade name is Parvolex)
4. When would you decide to use it?
Refer to Paracetamol OD chart found in all A&E departments
Table shows two treatment lines for plasma-paracetamol concentration against time
(in hours) since the OD.
Most patients will receive N-aceyl cysteine if their plasma-paracetamol level is above
the normal treatment line.
5. There are two treatment lines on the paracetamol overdose chart, one for
normal/low-risk patients and one for high-risk patients.
When would you use the
high-risk treatment line?
Malnourished patients e.g. anorexia, alcoholism, HIV positive
Patients taking enzyme-inducing drugs e.g. carbemazepine, phenobarbitone,
phenytoin, rifampicin, alcohol.
218
6. What would you like to ask Miss S to determine her suicide risk?
What happened today to make her take the OD?
Was the self-harm/attempted suicide premeditated or opportunistic?
Did she leave a note?
Did she intend to kill herself?
Does she regret her actions or would she do this again?
Any history of psychiatric disorders? E.g. depression, schizophrenia.
Previous suicide attempts?
Assess support network.
7. What are the long term consequences of paracetamol overdose?
Liver failure & encephalopathy
Renal failure
5.8
Acute poisoning
By the end of Phase II
•
students should be able to:
recognise the commoner inhalants and ingestants responsible for accidental and
deliberate poisoning
•
initiate immediate resuscitation if appropriate
•
participate in the management of the commoner poisonings
•
make an assessment of the underlying social and psychiatric factors in deliberate
self-harm and the likelihood of a repetition
219
5.9
You are the Surgical HO on call.
You are asked
as she has an increasing Mews Score.
by the nursing staff to assess Mrs B,
Mrs B had a right hemicolectomy two days ago
for a colonic carcinoma.
1. Define shock?
Circulatory failure resulting inadequate organ perfusion.
2. List four types of shock and a specific example, not including neurogenic shock.
Cardiogenic shock – post-MI
Hypovolaemic shock – bleeding from trauma, ruptured AAA, ruptured ectopic
pregnancy

fluid loss from vomiting, diarrhoea, burns, third
space
losses
Anaphylatic shock – Type I hypersensitivity reaction
Septic shock – Endotoxins released from Gram negative bacteria e.g. E.coli
Endocrine shock – Addisonian crisis, hypothyroidism
Iatrogenic shock – from anaesthetics, antihypertensives
3. What signs would suggest shock in a patient?
List six.
Systolic BP <90 mm Hg
 heart rate/tachycardia
 Capillary refill
 respiratory rate
 urine output/oliguria
Pallor
Faint/collapse
Confusion
Mrs B has the following observations:
Temp: 38.4C
Heart rate: 108bpm
BP: 92/74 mm Hg
O2 Sats: 97%
220
On examination she has warm peripheries and capillary refill is <2 seconds. The pulse
is rapid but bounding.
4. What type of shock is Mrs B likely to be suffering from?
Septic shock
5. What other observation would be useful to aid management?
Urine output
Fluid balance
6. What would be your immediate management plan?
IV access, if not already got it
IV fluid, colloid or crystalloid
Blood, urine and wound swab cultures
Antibiotics: Cefuroxine 1.5g TDS IV, Metronidazole 500mg QDS IV, Gentamicin
5mg/kg OD IV (blind treatment)
Senior review
5.9
Shock
By the end of Phase II
•
students should be able to:
recognise and initiate immediate investigation and management of
hypovolaemic
shock
•
recognise and initiate immediate investigation and management of septic shock and its
complications
•
recognise the pathophysiological disturbances which accompany shock
•
monitor and assess the response to treatment
221
5.11
Mrs D, an 80 year-old lady is brought to the Emergency department by her daughter
who is very concerned about her mother’s sudden onset of acute confusion. Her
mother lives alone independently but recently complained of a fever and has spent the
last few days in bed.
List 3 possible differential diagnoses for her acute confusional state (3)
Infection - LRTI

UTI

Septicaemia
Metabolic - Hypoglycaemia of hyperglycaemia

Hyponatraemia

Alcohol

Uraemia

Hypothyroidism or hyperthyroidism
CNS - Meningitis

CVA

Post-ictal
Drugs - Tranquillizers

Lithium, Digoxin

Benzodiazepines
You are unable to obtain a history from Mrs D. On examination Mrs D is very confused,
pyrexial 38ºC with a respiratory rate of 32 breaths/min, SATS 85% on air, BP 150/60
mmHg. Bronchial breathing is noted at the right lower lobe. Abdomen examination is
unremarkable.
What is the most likely diagnosis? (1)
LRTI
List 3 immediate investigations you wish to do. (3)
CXR
ABG
Blood cultures
Blood tests (urea, WCC, CRP)
Bm
222
An ABG comes back for you to interpret it
paO2 7 KPa
paCO2 4KPa
pH 7.47
HCO3- 20
BE 1
What does it show? (2)
Type I RF
What is your immediate management? (5)
High flow (60%) oxygen
IV access
IV fluids
IV antibiotics
Analgesia
Monitor ABGs regularly over the next few hours
Inform anaesthetist
Mrs D subsequently improves over the next few hours and is moved to a ward. What
potential complications could result from pneumonia? (2)
Lung abscess
Empyema
End of Question
Total 16 Marks
5.11
Acute confusional state
By the end of Phase II
students should be able to:
•
recognise and distinguish the common causes
•
initiate investigation and management of the commoner causes
223
5.13:
Whilst on your respiratory rotation, you are called by the nurses to see a 26 year old
man, with a known history of asthma, because he is complaining of shortness of breath!
When you reach his bed he is agitated and confused. He is tachpnoeic (RR = 24). His
most recent blood gas results are shown below.
ABG's:
pH = 7.36 kPa
pO2 = 7.8 kPa
pCO2 = 3.2 kPa
HCO3 = 24 mmol/l
BE = -2.0 mmol/l
SaO2 = 92% on air
Temperature = 38.1oC
1. What is your assessment of this patient?
ABG's show hypoxia. All other values are within normal parameters.
? Exaccerbation of asthma. Patient is suffering from Type 1 respiratory failure.
Causes include:

Pneumonia

Pulmonary oedema

Pulmonary embolism

Asthma

Emphysema

Fibrosing alveolitis

ARDS
2. What is your management?
A,B,C etc.....
Auscultate - wheezing?
Start highflow oxygen (35 - 60%) by face-mask to correct hypoxia
224
Salbutamol nebuliser
Position patient comfortably
Check hydration, pulse, BP, temperature etc.
Look for underlying cause of respiratory failure:
Order CXR
FBC; U and E's; CRP
Repeat ABG's.
Sputum and blood cultures (esp. if febrile)
Check medication and time of last dose.
Look up the BTS guidelines on step-wise approach to asthma management in adults (Beta 2
agonist, Inhaled steroid, Lk receptor agonist, Aminophylline etc).
Don't forget to get Senior help early, particularly if there is a chance his condition could
deteriorate! i.e. anaesthetists/ITU
Having done the above the patient improves so you go back to bed. However, 3 hours
later you are called back to the same ward because another patient has "gone off". You
stroll merrily back!
On arrival, the patient is a 56 year old man with an 80 pack-year smoking history and a
known COPD sufferer. He is cyanosed, sleepy and has a bounding pulse. His most
recent ABG results are as follows:
pH = 7.32
pO2 = 7.1 kPa
pCO2 = 7.0 kPa
HCO3 = 26 mmol/l
BE = 4
SaO2 = 89% on 2l O2 via nasal cannulae
3. What is you assessment?
ABG's show Respiratory acidosis with Type 2 respiratory failure.
Type 2 respiratory failure is defined as Hypoxia (PaO2 < 8kPa) with Hypercapnia
(PCO2 > 6.5kPa). This is caused by alveolar hypoventilation with or without V/Q
mismatch. Causes include:
225
Pulmonary disease (COPD, asthma, pneumonia, pulm. fibrosis, obstructive sleep
apnoea)
Reduced respiratory drive (sedative drugs, CNS tumour, trauma)
Neuromuscular disease (cervical cord lesion, diaphragm paralysis, polio,
Guillain-Barre syndrome)
Thoracic wall disease (kyphoscoliosis, flail segment).
4. What is your management?
Some debate on this one!
- Most consultants say......
A,B,C's as normal.
Start high-flow oxygen as hypoxia will kill you quicker than hypercapnia.
This is because high-flow oxygen will "knock-off" the patient's respiratory drive which
is driven by hypoxia instead of hypercapnia. (The reason for this is becasue the
patient has been suffering from a chronic resp. disease, resulting in
higher-than-normal blood CO2 levels. The central respiratory centre is therefore
relatively insensitive to CO2 levels, "resets" to a higher CO2 level and as a result
hypoxia drives respiration. By giving lots of O2, pO2 levels rise and respiration
reduces. This raises CO2 levels to dangerous levels and causes acidosis. In addition,
hypoxia will also occur due to the lack of respiration).
CALL ANAESTHETISTS/ITU CONSULTANT!!!
- The OHCM says......
Treat underlying cause
Controlled O2 therapy: start at 24%
re-check ABG's after 20 mins. If PCO2 is steady or lower, increase O2 to 28%. If
PCO2 has risen > 1kPa and the patient is still hypoxic, consider a respiratory
stimulant e.g. Doxapram 1.5 - 4 mg/min IV or assisted ventilation e.g NIPPV.
If this fails consider intubation and ventilation i.e. CALL ANAESTHETISTS/ITU
CONSULTANT!!!
Guide to colours of Venturi valves:
Venturi valve Flow rate Oxygen delivered colour (l/min) (%)
Blue
White
2
4
24
28
226
Yellow
6
35
Red
8
40
Green
12
60
Which mask for which patient?
Nasal cannulas:
Patients with normal vital signs—for example, postoperative, slightly low oxygen
saturations, long term treatment with oxygen at home
Simple face masks and masks with a reservoir bag:
Higher concentrations of oxygen are required and controlled oxygen is not
necessary—for example, severe asthma, acute left ventricular failure, pneumonia,
trauma, or severe sepsis. (Masks should always be set to a minimum of 5 l/min
because significant rebreathing of CO2 can occur when exhaled air is not adequately
flushed from the mask)
Venturi masks:
Controlled treatment with oxygen required in people with chronic respiratory
failure—for example, COPD
5.13
Respiratory failure
By the end of Phase II
•
students should be able to:
make a rapid assessment and initiate management in the patient
with acute respiratory
distress
•
recognise a patient in respiratory failure
•
assess the cause, severity and type of respiratory failure and initiate management
•
interpret arterial blood gas results
•
prescribe oxygen and oxygen delivery systems appropriately
227
6.1 + 6.2
Miss P.V Bleed presented to her G.P with long standing menorrhagia. She went on to
explain that recently she had been feeling tired all the time and on occasions becomes
quite breathless. On examination the G.P noted she was pale, tachycardic and had
spoon shaped nails. He decided to undertake some routine blood tests, one of these
being a full blood count.
Which 2 components of the full blood count are we most interested in when classifying
Miss P.V Bleeds anaemia? (1)
Haemaglobin
MCV
Miss P.V Bleeds full blood count results were as follows.
Haemoglobin 9.8g/dL
Mean Corpuscular volume (MCV) 72fl
Mean Corpuscular Hb (MCH) 23 pg
From these results, how would you classify Miss P.V Bleeds anaemia? (1)
Microcytic anaemia
Given her history and above blood results, Miss P.V Bleed is most likely to be suffering
from Iron deficiency anaemia as a result of her chronic blood loss.
List three other causes of Iron deficiency anaemia (3).
malabsorption
low dietary Fe intake
Pregnancy (increased requirement)
Hypothyroidism
chronic gastrointestinal bleed
What two following blood tests could you request the lab to run on Miss P.V Bleeds
blood related to her condition of Iron deficiency anaemia to confirm your diagnosis? (2)
228
Serum ferritin
Serum Iron
What is the appropriate treatment for Miss P.V Bleed given her Hb level is 9.8 g/dL (1)
Oral Fe, ferrous sulphate, I.M Fe if unable to tolerate oral.
How would you initially manage her if the Hb level was 4.8 g/dL? (1)
Blood transfusion
List 3 hazards for this initial treatment of correcting a low Hb level (3).
Transfusion reactions (cross reaction between donor and recipient)
transmission of infection, Hep B, C, HIV
Circulatory overload leading to heart failure, electrolyte disturbances from large
volumes in transfusion.
Following her Gynae outpatient appointment Miss P.V Bleed’s menorrhagia was treated
and her iron deficiency anaemia corrected itself with the above treatments.
Unfortunately 6 years later she developed a gastric carcinoma and required a
gastrectomy.
After her operation she returned to her G.P with similar symptoms as before, pallor,
exhaustion and lethargy.
Her Hb was 9g/dL.
How would the red cells appear on a blood film taken from Miss P.V Bleeds blood? (1)
Macrocytic
Based on her history, what form of anaemia is Miss P.V Bleed most likely suffering from
now? (1)
Pernicious anaemia.
229
What is required for Vitamin B12 to be adequately absorbed by the terminal ileum? (1)
Intrinsic factor
The Schillings test is often used to confirm a diagnosis of Vitamin B12 deficiency
anaemia, briefly describe this test (4)
Patient given a loading dose of 1000mg of Vit B12 intramuscularly, followed by
small doses of radioactive vit B12 and excretion is then measured in the urine.
Vitamin B12 malabsorption is corrected by giving intrinsic factor with labelled
vitamin in pernicious anaemia, but persists despite the use of intrinsic factor in
intestinal disease.
How would you wish to manage Miss P.V Bleed if the schillings test was positive for Vit
B12 deficency anaemia and her Hb was around 9 g/dL? (2)
IM hydroxycobalamin 3 times weekly for 2 weeks and then once every 3 months for
life.
Name 4 other causes of macrocytic anaemia.
Alcohol
Folate deficiency
Liver disease
Drugs , e.g. azathioprine
Reticulocytosis
hypothyroidism
230
6.1
Investigations
By the end of Phase II
students should be able to:
• obtain a blood sample by venepuncture
• interpret a full blood count report
• explain to patients the nature and value of a bone marrow aspiration and biopsy
6.2
Anaemia
By the end of Phase II
•
students should be able to:
recognise the significance of a low haemoglobin taking into account the age and sex of
the patient
•
distinguish the type of anaemia from the blood count and together with the clinical
information from the patient determine the likely cause
•
evaluate the cause of iron-deficiency anaemia
•
treat iron-deficiency anaemia
•
distinguish between iron-deficiency anaemia, the anaemia of chronic disease and
thalassaemia trait and either treat or make appropriate referral
•
distinguish between macrocytic and megaloblastic anaemia
•
determine the cause of macrocytic anaemia clinically and with appropriate laboratory
investigation
•
use laboratory investigation to identify the cause of megaloblastic anaemia and to
institute treatment with appropriate urgency
•
initiate investigation of normochromic, normocytic anaemia and make appropriate
referrals
•
recognise the possibility of and likely causes of haemolytic anaemia from laboratory
investigation and clinical evidence and make appropriate referrals
•
recognise and institute management of acute complications of sickle cell anaemia
231
6.3
Miss F is an 18 year old female who presented with acute shortness of breath. On
further questioning she has a 3- 4 month history of malaise and night sweats which has
worsened over the last two weeks. She also complains of frequent nose bleeds, heavy
periods and easy bruising. On examination the positive findings are pallor, a
temperature of 380c, pulse of 105, BP: 135/79, respiratory rate of 18 per minute. You
suspect a Leukaemia.
A) Name four features you would expect to see on a blood test.
Any four from:
Anaemia
Leucopenia
Thrombocytopenia
Hyperuricaemia (due to break down of cells)
Reduced clotting times
B) Apart from a blood test, name four other investigations you would carry out.
Any four from:
Peripheral blood film
Bone marrow aspirate/trephine
Immunophenotyping (identify cell type by looking at cell surface)
Cytogenetics (to look for Philadelphia Chromosome which is found in Acute
Lymphoblastic and Chronic Myeloid Leukaemia).
Cytochemical studies (to identify cell types by looking at enzymes in cells).
Chest X-ray (to rule out lymphoblastic lymphoma which would cause a mediastinal
mass which would be identified on the chest x- ray).
C) As part of her treatment you give her a blood transfusion. Name two complications of
a blood transfusion.
Immunological complications:
Haemolytic transfusion reaction.
Non- haemolytic transfusion reaction (febrile reaction)
Alloimmunisation
232
Incompatibility
Transfusion related acute lung injury (TRALI)
Anaphylaxis (plasma protein incompatibility)
Non immunological complication:
Transmission of infection
Circulatory failure because of volume overload
Iron overload, with repeated transfusions
Electrolyte change (K+ particularly) with massive transfusion
Thrombophlebitis
Air embolism
D) The studies show she has acute lymphoblastic leukaemia and you begin
chemotherapy to induce remission. During chemotherapy there is a risk of Tumour
Lysis Syndrome. What is this and how can it be prevented?
It is the rapid necrosis of tumour cells due to the chemotherapy which results in
hyperuricaemia and raised potassium and phosphate levels. It can cause renal failure
due to the deposition of uric acid and calcium phosphate crystals in the renal tubules.
It is often exacerbated by a low intravascular volume.
It can be prevented by ensuring the patient has normal uric acid levels by prescribing
Allopurinol and by ensuring diuresis is established with I.V. fluids.
E) Apart from Tumor Lysis Syndrome name three short term and three long term effects
of chemotherapy:
Short term: (any three)
Myelosuppression
Nausea and Vomiting
Alopecia
Oral ulceration
Long term: (any three)
Infertility
Second malignancy
Cardiac Arrhythmias and cardiomyopathy
Pulmonary fibrosis
Cataracts
233
F) Leukaemic blast cells can infiltrate the brain and lungs resulting in coma and
respiratory failure respectively. If the blast cell count in the peripheral blood is very high
(> 100x 109/L) that patient may need leukophoresis. What is leukophoresis?
Blood is collected from a vein and centrifuged to remove the leukaemic cells. The red
blood cells and plasma are then returned to the patient via another vein. It prevents
sludging of blast cells in the capillary beds.
6.3
Haematological malignancies
By the end of Phase II
•
students should be able to:
recognise the possibility of a malignant disorder of the lympho-haemopoietic system on
clinical grounds and from the blood count
•
make an appropriate estimate of the urgency of referral
•
undertake investigations to confirm myeloma as a cause of a raised plasma viscosity
•
describe to patients in outline the potential benefits and possible side effects of
radiotherapy, chemotherapy and hormonal therapy in lymphoma
234
6.4
(answers not provided for this question)
Clinical Scenario:
You receive the following lab data at the end of clinic and turn up these values:
Haematocrit: 49%
RBC; 6.4 x 106 microlitres
MCV: 76fL
WBC 14 000 per microlitre
Platelet count 480 000 per microlitre
The white cell differential is normal.
The patient is a middle aged woman who has come in for a yearly check up, she does
not smoke or drink. She appeared well and physical examination was normal.
1. Are these results normal?
2. Are these separate abnormalities or part of a patter
3. What separate abnormalities would cause similar results?
4. If you consider the data together, what disease comes to mind?
5. What would be your plan at this stage?
The patient returned in 2 months and re examination revealed a spleen tip.
6. What is your thought now?
This same patient returns with a swollen left leg. Doppler studies have revealed she has
a femoral vein thrombosis. Her laboratory results return as follows:
HCT: 50%
WBC: 18 000 per microlitre
Platelet count: 850 000 per microlitre
MCV: 72 fL
7. There are many causes of venous thrombosis. Does any possibility seem obvious
from this data?
8. What treatment would you offer her?
6.4
Polycythaemia
235
By the end of Phase II
students should be able to:
• recognise an abnormally high haemoglobin
• use clinical examination and a full blood count to determine the cause
• investigate and refer appropriately
• participate at an appropriate level in venesection treatment
236
6.5
(answers not provided for this question)
A 12-year-old girl visits the GP following concerns about her first menstrual period. The
flow was very heavy and persisted for two weeks. On further discussion she reported
having always bruised easily and suffered with nose bleeds as a young child.
What other information should be sought if a bleeding disorder is suspected?
You suspect this is a congenital bleeding disorder. What blood tests will you need to
carry out for the haematologist?
If she is diagnosed with a bleeding disorder what 3 interventions should her GP avoid?
Bleeding disorders are rarely congenital and are more commonly acquired, complete
the following table regarding common disease states and the coagulation deficit they
cause:
Disease state
Pathophysiology
Malabsorption
Vitamin K deficiency
Decreased synthesis of coagulation factors
Bacterial septicaemia
Thrombotic disorders:
Mr RB is a 58-year-old businessman who has come to see his GP with some pain and
swelling in his left calf. He has recently been on a long haul flight to New York and is
worried about deep vein thrombosis (DVT)
What else do you need to ask about in the history?
If you suspect a DVT what would you look for on examination?
After a full history and examination are carried out Mr RB is sent for investigations that
confirm a DVT and anticoagulation therapy is commenced.
237
As the F1 on call you are asked to start Mr RB on warfarin, what dose will Mr RB be
started on, and how will the treatment be monitored?
What else does Mr RB need to know about starting anticoagulation therapy?
What is the target international normalised ratio for treatment of a one off DVT or
pulmonary embolism?
Mr RB returns to the GP after a minor episode of bleeding following a tooth extraction
during the treatment period, the results of a blood test show an INR of 5, he is also
complaining of bleeding gums. What 3 measures are recommended management
strategy for this type of patient?
6.5
Disorders of bleeding and thrombosis
By the end of Phase II
students should be able to:
•
recognise a bleeding tendency on history and examination
•
manage at an appropriate level an acute haemorrhagic state
•
interact appropriately with the haematologist in the management of patients with chronic
problems of haemostasis
•
recognise the possibility of a pro-thrombotic state on clinical grounds and make an
appropriate referral
•
initiate and monitor anticoagulant therapy with warfarin and heparin according to
published guidelines
•
counsel patients on anticoagulant therapy
•
recognise over-anticoagulation on clinical and laboratory grounds and be able to initiate
management after appropriate discussion
238
6.8
Jane Smith is a 52 year old lady receiving chemotherapy for cancer. She has undergone
2 cycles with an interval of 6 weeks between them.
She finished the latest chemotherapy 2 weeks ago.
Reviewing her blood results it is clear that she has the following abnormalities:
Anaemia
Thrombocytopaenia
Leucopenia
Q1) what has happened and why?
EXPERIENCING BONE MARROW FAILURE ?SECONDARY TO
CHEMOTHERAPY.
Q2) what are the risks associated with this picture?
INCREASED BLEEDING, INCREASED SUSCEPTIBILITY TO INFECTION
Q3) how do chemotherapy drugs work?
AFFECT DNA SYNTHESIS OR REPAIR
PROMOTES CELLULAR APOPTOSIS
The rest time between the cycles of chemotherapy allow the normal cells such
as bone marrow and the GI tract to recover from the toxicity.
Q4) what precautions should be taken (haematological) before commencing
chemotherapy?
REVIEW PERIPHERAL BLOOD FILM
Q5) what can be given to accelerate the recovery of the white cell count after
chemotherapy
GROWTH FACTOR G-CSF
239
Q6) A cytotoxic drug (such as Alkylating drug) when used in combination with
extensive irradiation, can result in what possible haematological disorder?
Increased incidence of ACUTE NON-LYMPHOCYTIC LEUKAEMIA
Q7)Give two factors that can increase the incidence of blood dyscrasias in a patient?
INREASING AGE
DOSE OF DRUG
DURATION OF DRUG USE
6.8
Drug interactions
By the end of Phase II
•
students should be able to:
recognise a haematological problem as being a possible consequence of concomitant
drug therapy especially in patients on chemotherapy
•
recognise patients who are at risk of severe sequelae to drugs because of underlying
haematological diseases
240
7.2 & 7.3
Barry, a previously well sixty two
year old butcher from Barton on
the Beans presents with a 3 day
history of swollen ankles and
shortness of breath.
He is a
poorly controlled diabetic and has
been so for at least 18 years, and
has poorly controlled
hypertension. During his work up
he has the above PA x-ray taken
and an ECG.
1. Give 3 pathological findings you can see on this x-ray?
Cardiomegaly: The cardio thoracic ratio is greater than 50%.
Pulmonary Oedema: Bat wing appearance (Caused by the initial narrowing of the
lower lobe vessels and widening of the upper lobe vessels).
Pulmonary Effusion (blunting of the costophrenic angles)
*** Also to look for in real life are KERLEY B LINES: 1-2 cm long horizontal
lines which meet the pleura at right angles. They are usually seen up the
side of the lungs beginning at the costophrenic angle. They are due to
interlobular lymphatics which have been distended by fluid or tissue
classically secondary to left ventricular failure or mitral stenosis. ***
2. Why is it important to know if this is a PA or AP film and which is most useful in this
case?
241
When x-rays enter the chest anteriorly and are detected posteriorly, the heart, which is
anterior in the chest, casts a bigger shadow than in PA films. PA films are thus more
accurate in the assessment of cardiomegaly.
When a large heart shadow isn’t necessarily cardiomegaly:
In an AP x-ray
In expiration:
A pregnant patient.
One with ascites.
An obese patient
A disobedient patient
In these cases patients may not be able to take a full inspiration which might make the
heart look bigger than it actually is. Eight to nine posterior ribs showing is a good
inspiration.
3.
Considering the history, what are the likely causes each of the 3 pathological
findings?
Cardiomegaly:
Caused by hypertrophy of the myocardium as a result of years of
uncontrolled hypertension
Pulmonary Oedema:
Caused by fluid leaking into the interstitium
due to ↑
hydrostatic pressures as a result of left ventricular failure and pulmonary venous
congestion.
Pleural Effusion:
A Transudative pleural effusion also caused by raised hydrostatic
pressure as above.
242
One of WMS’s medical students suggested to the Consultant cardiologist that an ECG
might be useful. Below is the patient’s ECG
[Sorry the ECG does not fit the case perfectly;
I couldn’t find one that did]
4.
Where and what are the abnormalities, and hence where and what is the cause?
Bradycardia of 50 bpm (Often associated with MI)
S-T segment changes: ST elevation in leads II, III, and aVF suggesting inferior wall MI.
Reciprocal changes (ST segment depression) in leads V1 and V2.
In addition there is ST elevation in lead V6 suggesting lateral wall involvement. Lateral
infarction produces changes in leads I, avL and V5/6.
T Waves:
Tall, (>75% of R waves) especially in Lead III and AVF which can be
caused by myocardial

ischemia.
Inverted T waves in V1, V2 V3
These changes suggest an infarction affecting the inferior aspect of heart.
5.
What is the likely diagnosis? Be specific about where in the heart the problem is.
Inferior lateral MI (Silent)
243
6.
From your basic understanding of ECG’s, explain why is there S-T depression in
leads V1 and V2, and S-T elevation in leads II, III and aVF?
The ST depression is basically S-T elevation when viewed from the other side of the
heart. S-T Elevation represents MI in the inferior aspect of the heart. When viewed
from the V1 and V2 leads, the opposite picture is seen.
E
7.
Why might our patient not have any chest pain despite the likely diagnosis?
A long term Diabetic, especially one who is poorly controlled may have diabetic
neuropathy.
The damage to nerves caused by hyperglycaemia may mask the usual
symptoms of infarction, causing a silent MI.
Florence, a 72 year old women with a 15 year history of COPD presents to you with an
acute exacerbation of shortness of breath.
She also has fevers and rigors and pain
associated with breathing and coughing. Her cough is productive of rusty coloured
sputum.
She has a chest x-ray shown below:
8.
What can you see?
How does this support
the likely diagnosis?
The right upper
lobe (RUL) is
showing an
increased
density.
244
There are linear lucencies in the right upper lobe (air bronchogram).
This is due to consolidation consistent with a lobar pneumonia.
Explanation: On a normal CXR, we can visualize the air-filled trachea, mainstem bronchi
and initial portions of the lobar bronchi. Further branches should only be seen with the
injection of contrast during a bronchogram. When air can be visualized in the more
peripheral intrapulmonary bronchi, this is known as the ‘air-bronchogram sign’. This
abnormality is usually caused by an infiltrate/consolidation that surrounds the bronchi.
Dave Is a 66 year old man who has been getting progressively more breathless over a
period of years.
He is a life long non smoker, is phobic of pigeons and only tried slate
mining once for a few minutes as a teenager (and he did not inhale).
He has no
medical history except for long standing and severe rheumatoid arthritis.
9.
Describe what you can see
in this x-ray that may be related
to Dave’s breathlessness. State
in general terms the diagnosis:
Fine nodular and streaky
linear shadowing
(reticularnodular pattern)
which is worse in the lung
basis. Poor definition of
cardiac border (Due to
adjacent lung fibrosis)
Fibrosis
Blunted costophrenic angle indicative of pleural effususion.
245
A spirometry is requested to investigate Dave’s lung function.
A Leicester medical student who is writing up Dave for a portfolio intercepts the results
and finds that Dave’s FEV1:FVC ratio is 0.88. Having a vague recollection of the
importance of the ratio being over 0.70 He asks you if he can tell Dave the good news,
that all is well.
10.
What do you say? Explain your answer, especially the relevance of a ratio being >
than 0.75.
No you can’t. A ratio of >0.75 indicates that there is no obstructive airway disease, so
we can only rule out diseases such as COPD and Asthma. To interpret the results fully
you will need the rest of the results.
11.
When you get the results you find that Dave’s Vital capacity is only 55% of the
predicted for his age and height. What can you conclude about the pattern of lung
disease now?
This is a restrictive pattern of disease. The vital capacity is reduced by fibrosis, and the
FEV1: FEV ratio is normal because there is no obstruction.
E
12.
Years later Dave presents with worsening breathlessness and swollen ankles.
What is the likely cause?
Dave’s rheumatoid related lung fibrosis has caused right heart failure (cor-pulmonale).
Pulmonary vasoconstriction due to alveolar hypoxia or blood acidemia and/or
anatomic compromise of the pulmonary vascular bed in lung disease eventually
causes pulmonary hypertension, which eventually causes right heart failure.
A women presents to A+E acutely unwell:
Her arterial blood gases are measured, and
the results are shown below:
Normal Values
pH
7.25
7.35-7.45
pCO2 kPa
7.12
4.7-6
246
pO2 kPa
HCO3
13.
9.1
mmol/l
>10.6
25
What do they indicate?
22-28
(State respiratory/metabolic alkalosis/acidosis)
Respiratory acidosis
14.
Suggest a Cause:
Severe asthma
COPD
Decreased GCS (CVA)
19 year old Sarah presents to you, her GP, with shortness of breath. She has had an
irritating cough at night for a month.
15.
You decide to get her to do a peak flow.
Explain how you would explain to her what to do:
Make sure the peak flow dial is at zero. Stand up if possible and take in as deep a
breath as you can. Then place your mouth around the mouthpiece of the meter and
form a tight seal with your lips. Blow out through your mouth as hard and fast as you
can until you can not blow out any longer. This will move the main flow indicator up the
scale and the number listed beside the indicator is your peak flow.
While chatting to Sarah she tells you that she recently spent 6 months working and
travelling in the Indian sub continent.
She stayed with a family for three months in a
remote village. She left in a rush and burnt all her belongings when she found out that 2
of the adults had TB, and she is worried she has caught it.
Sarah has had all her due
vaccinations.
16.
Would a tuberculin test such as the mantoux test be useful in ruling out TB
infection? Explain.
No.
If she has had a BCG vaccination she should react to the tuberculin protein
regardless of whether she has contracted TB.
[Incidentally, the vaccination only
247
provides 80% protection, and only for the first 15 years after the vaccination, so she
could have TB despite having a BCG]
E.
17.
What would be the most useful investigation to rule out pulmonary TB, especially
in someone with a non productive cough?
Chest X-ray. (In TB Typically shows patchy or nodular shadows in the upper lobes,
loss of volume and fibrosis with or without cavitation) An abnormal chest x-ray is often
found in asymptomatic patients, but the reverse is very rare.
Alan, a 56 year old Social Science lecturer presents with central crushing chest pain on
exertion. The pain only comes when he is exercising and has been getting slightly
worse over a period of 2 years.
He smokes 20/day and has a 5 year history of
hypertension, which he attributes to a particularly stressful module he teaches at his
university’s medical school.
18
Which two important investigations would you do at this early stage?
ECG
Cardiac Enzymes
Both these tests were normal. His blood tests are normal except cholesterol which
comes back as 7.9 mmol/l.
19.
What other non-invasive test would you like to do?
Stress electrocardiography.
20.
How would you explain this test to Alan?
Stress electrocardiography is an measurement of your hearts electrical activity taken
while you are exercising. The purpose is to analyse the changes in your hearts
248
electrical pattern as it does more work. Some patterns which indicate a lack of
oxygen reaching your heart are not visible until the heart is stressed.
Below is Alan’s stress ECG
21.
Is the
exercise test
positive or
negative?
Explain.
There
is S-T
segment depression, indicating a positive exercise test and probable IHD.
not actually have IHD).
(5% may
In particular the segment of S-T depression is flat or even
slightly down sloping which is much more likely to be due to IHD than up sloping S-T
depression (see Oxford Hand book).
22.
What are Alan’s risk factors for IHD?
Smoking,
Hypertension,
Hyperlipidaemia,
sex,
age,
stress.
23.
Alan is diagnosed with Angina.
What would you prescribe for him to take when
he experiences the same chest pain on exertion again.
249
Sublingual glyceryl trinitrate tablet (0.5 mg) or GTN spray.
24.
In the longer term how would you treat him?
Give life style and medical
management.
Weight loss
Stop Smoking
Take gentle exercise
Diet (low fat, low salt)
Relaxation
Decrease alcohol (if appropriate)
Medical:
Asprin
(75-150mg /24hrs PO unless contraindications)
Beta blockers
Nitrates
(atenalol 50-100mg/24hr PO)
(GTN spray or sublingual tablets up to every ½ hour)
(as prohylaxis 10-30mg oral nitrates BD such as isosorbide mononitrate)
Calcium antagonists
Statin
25.
(amlodipine 5mg/24hr, diltiazem-MR 90-180mg/12 hr PO)
(Aim to reduce cholesterol to <5mmol/l)
Alan is scheduled for coronary angiography. Explain to him what this entails and
how it may benefit him.
This involves insertion of a catheter into the heart via a blood vessel in your leg.
Contrast can be introduced into the coronary allowing visualisation of the anatomy and thus patency - of the coronary arteries.
Obstructed blood vessels can be unblocked in various ways in the same procedure if
appropriate.
Objectives:
250
7.2
Investigations
By the end of Phase II
•
students should be able to:
recognise the basic radiological features of cardiac enlargement and
pulmonary oedema and relate them to the underlying
pathophysiology
•
perform an ECG
•
understand the basic principles of electrocardiography and interpret
major abnormalities.
•
be able to explain stress electrocardiography, echocardiography,
and coronary and femoral angiography to a patient.
•
recognise the basic radiological features of consolidation, pleural
fluid, lung collapse, carcinoma and lung fibrosis, seeking advice
where necessary and relate them to the underlying pathology
•
obtain and interpret a spirograph and recognise the changes in
obstructive and restrictive lung disease
•
interpret blood gas abnormalities
•
use and explain the use of a peak flow meter
•
interpret tuberculin tests
251
7.3
Angina pectoris
By the end of Phase II students should be able to:
 recognise angina
 initiate investigations to confirm the diagnosis and assess the severity of
underlying coronary artery disease
 recognise the risk factors in individual patients
 initiate immediate management
252
7.3
Daisy, an 80 year old lady presents to her GP following a few episodes of chest pain she
has experienced whilst out walking. This is particularly noted on colder days and the
pain is relieved by sitting on a nearby park bench.
The GP informs her that she may have a condition known as ‘Angina Pectoris’.
Question> What is the definition of Angina pectoris (also known as stable angina)?
Central chest pain (caused by myocardial ischaemia) which is brought on by exertion
and relieved by rest.
Question> The GP mentions that a cold environment, exercise, eating a large meal and
emotional stress can play a role in initiating an angina pectoris attack. As well as these
triggers, list 6 risk factors associated with coronary artery disease?
Hypertension
diabetes mellitus
smoking
hyperlipidaemia
high BMI
family history
exercise (lack of)
age, sex of the individual
ethnicity
socio-economic status
alcohol.
Question> The GP after taking a detailed history, begins an examination (with a
chaperone). In particular, he looks for signs of anaemia, thyrotoxicosis and
hyperlipidaemia. He then auscultates the chest for heart sounds. In particular, he listens
for murmurs associated with aortic stenosis and aortic regurgitation.
What role would aortic stenosis play in someone with angina pectoris?
If the individual exerts themselves, the cardiac output would increase. However, if
there is an obstruction to left ventricle emptying, then the coronary arteries will not
receive the perfusion they are now requiring with exertion, thus exacerbating angina
253
pectoris.
Can you draw the features of an aortic stenosis murmur on this diagram?
HS
HS
I----------II------------I
I111111111II------------I

(ejection pan-systolic murmur)
The GP refers Daisy for an ECG.
Question> Name 2 positive findings on an ECG that would indicate Daisy was suffering
with an acute attack of angina pectoris?
ST depression, T wave inversion and T wave flattening.
Question> Name 2 positive findings that would indicate previous ischaemic heart
disease (IHD)?
Left bundle branch block
pathological Q waves
left ventricular hypertrophy (i.e. left axis deviation, left ventricular strain, large R
waves in V5 and large S waves in V2).
Question> The GP refers Daisy for an Exercise Tolerance Test (ETT). The cardiologist
reviews the ETT-ECG for any of the ECG changes aforementioned. If there are no
positive ECG changes and no symptoms experienced whilst on the treadmill, does this
mean Daisy does not have angina pectoris?
No. Angina pectoris is a clinical diagnosis and the ETT is only 70% sensitive and 80%
specific. The diagnosis of angina pectoris is a clinical one, derived after a combination
of history, examination and investigations.
With a normal ETT, the GP wants to refer Daisy for a diagnostic Coronary angiography,
where therapeutic treatment may also occur; PTCA (Percutaneous transluminal
coronary angioplasty i.e. ballooning) and/or a stent may be inserted. Depending on the
results, a distant option is a coronary artery bypass graft (CABG).
Question> In the meantime, the GP puts Daisy on a β-antagonist. What is the
254
mechanism of action in angina pectoris?
A β antagonist acts on the β1 receptors in the blood vessels and heart to reduce heart
rate (negative chronotropic effect) thus allowing greater filling of the coronary arteries
during diastole. β antagonists also reduce contractility of the ventricles (negative
inotropic
effect)
and
thus
reduce
the
oxygen
demand
on
the
heart.
Non-cardioselective β antagonists will also act on β 2 receptors in the lungs and cause
broncho-constriction.
Question> Name two other classes of medication that are used to relieve symptomatic
angina pectoris?
Calcium receptor antagonists
nitrates
Potassium receptor agonist (e.g. Nicorandil).
Extended Question> Name 2 additional types of angina (other than angina pectoris) and
state their definition?
Crescendo angina. The amount of effort to invoke symptoms decreases with time,
although symptoms do not occur at rest.
Unstable angina. Symptoms appear unpredictably either with minimal exertion or at
rest.
Nocturnal angina. Symptoms appear at night and can awake the patient.
Decubitus angina. Onset of symptoms occurs when the patient is lying down.
The GP also notes that treatment will be a holistic one with an aim of reducing all
cardiovascular risk factors.
7.3
Angina pectoris
By the end of Phase II
•
students should be able to:
recognise angina
255
•
initiate investigations to confirm the diagnosis and assess the severity of underlying
coronary artery disease
•
recognise the risk factors in individual patients
•
initiate immediate management
256
7.4
1)
A)
65 years old Mr. E presented with acute onset of central crushing chest pain 2
hours ago.
He said that the pain came on suddenly while he was sitting and
reading a book and since then the pain persisted.
otherwise was not breathless.
He looked generally unwell but
Apart from acute coronary syndrome (ACS), list 4
other possible causes for his symptoms:
(any 4)
Pericarditis
Dissecting aortic aneurysm
Pulmonary embolism
Gastroesophageal reflux
Musculoskeletal pain
Pancreatitis
Perforated peptic ulcer
B)
Hopefully you know you will need to do a 12 lead ECG.
What ECG patterns would
indicate ST elevated myocardial infarction (STEMI) that fits the criteria for
thrombolysis?
More than 1 mm elevated ST segment in 2 or more contiguous limb leads, or
More than 2 mm ST elevated in two or more contiguous chest leads.
C) You have diagnosed him as having ST elevated myocardial infarction (STEMI) and
decided to prescribe thrombolytic drugs. List five contraindications to the prescription
of thrombolytic drugs.
(any 5 from below)
History of haemorhagic stroke at anytime in the past
Active or recent gastric ulcer or any active internal haemorrhage
Suspected aortic dissection
257
Recent major surgery with the last 6 weeks e.g laparotomy
Head injury
Previous allergic reaction to fibrinolytic agents
Non haemorrhagic stroke within the past 1 year
Previous streptokinase exposure
Pregnancy or post partum
C) what other immediate investigations will you do for Mr. E ?
Bloods- FBC, U&Es, LFTs, D- dimer, glucose, lipid profile, cardiac enzymes (CK,
Troponin T 12 hours after the onset of chest pain)
Chest xray
ABG
Later management( arrange Echo)
D) At the mean time, you will need to control his pain. what analgesia, apart from GTN,
will you use, what dose and what route?
Diamorphine IV
2.5-10mg
C)
C) what are the three factors that might contribute to thrombus formation in Mr. E’s
coronary artery?
Virchow’s triad:
Damage in the intimal surface of the vessel
changes in the pattern of blood flow
hypercoagubility
D) Few hours later Mr. E had a cardiac arrest and you were called in by the nurses.
The
cardiac nurses were doing the chest compression and a Guedal airway was in place.
Another nurse now showed you the following ECG pattern.
What is your diagnosis?
What do you do next?
258
Diagnosis:
ventricular fibrillation
Attach defibrillator and defibrillate (joules depending on the machines) and check
pulses in between.
Give adrenaline every 3 mins accordingly, IV 1 mg.
7.4
Acute myocardial infarction
By the end of Phase II
•
students should be able to:
recognise acute myocardial infarction and use appropriate investigations to confirm the
diagnosis
•
act appropriately to ensure that those patients likely to benefit receive thrombolysis as
quickly as possible
•
control the pain of myocardial infarction
•
recognise ventricular fibrillation and carry out immediate management
•
recognise the need for further active management in the medium to long term
•
describe the features of the underlying pathophysiological changes which can be used
to develop treatment and prevention strategies
259
A fifty-five year old man, Mr. Askew, presents in A&E with sudden onset of severe chest
pain persisting for one hour. He describes his chest pain as “crushing”, with no
radiation. He admits to having similar chest pains lasting for around 15 minutes over
the last 5 months. These pains usually occur when he is exercising and are relieved by
rest. Rest does not relieve his chest pain today. He feels sweaty, breathless and
nauseous. He does not smoke.
1 What features of the history suggest a cardiac origin for his chest pain?
Central, crushing chest pain with sweating, breathlessness and nausea.
2 What chest pain features would be suggestive of aortic dissection?
Sudden tearing chest pain, may radiate to the back.
3 Give two likely cardiac causes of Mr. Askew’s chest pain.
Unstable angina.
Acute myocardial infarction.
4 Which pathological process commonly underlies these two diagnostic possibilities?
Thrombus formation on an atherosclerotic plaque.
5 What biochemical markers would you assay to distinguish between your two
diagnostic possibilities given in question 3?
Cardiac enzymes – creatine kinase (MB), aspartate transferase (AST), lactate
dehydrogenase (LDH).
Cardiac troponins (troponin T and/or I).
6 Mr. Askew’s ECG shows ST elevation and development of pathological Q waves in
leads V1-6, I and aVL. What diagnosis do these ECG findings suggest?
Acute anterior myocardial infarction.
7 What would you expect to find at cardiac catheterisation in this patient?
Lesion causing blockage in/narrowing of left anterior descending coronary artery.
8 Mr. Askew is given 300mg oral aspirin. What is the mode of action of aspirin?
Irreversible acetylation of platelet cyclo-oxygenase enzyme, preventing thromboxane
A2 formation, leading to inhibition of long-term platelet activity/aggregation.
260
9 Give the four phases of clot formation.
Vasoconstriction
Platelet activation
Clotting
Clot lysis and scarring
10 Name three other substances you would prescribe, in addition to aspirin, as part of
Mr. Askew’s immediate management.
Oxygen
Diamorphine/morphine
Glyceryl trinitrate
11 What non-modifiable risk factors for coronary heart disease does Mr. Askew have?
Male sex
Age
12 List five modifiable risk factors for coronary heart disease.
Smoking
Hypertension
Hypercholesterolaemia
Diabetes mellitus
Obesity
13 Mr. Askew has a body mass index of 26.9kg/m2. What does this indicate?
Patient is overweight.
14 Name two drugs that may be used in obesity treatment to achieve weight reduction.
Orlistat.
Sibutramine.
15 Mr. Askew receives thrombolysis. List five contraindications to thrombolysis.
Coagulation defects, including anticoagulant treatment.
Stroke within last six months.
Possible aortic dissection.
Acute/recent gastrointestinal bleeding.
Surgery/major trauma within last two weeks.
16 One UK National Service Framework states patients suitable for thrombolysis should
261
receive this within 60 minutes of symptoms onset. Name the process that could be used
to evaluate whether current practice is meeting this standard.
Clinical audit.
A few hours later, Mr. Askew suddenly feels unwell. His ECG is shown below.
(Source: http://www.ecglibrary.com)
17 What does this ECG show?
Ventricular fibrillation.
18 Name two drugs that you might need to use as part of your immediate management
of this condition.
Adrenaline
Amiodarone
19 List five early complications of myocardial infarction.
Arrhythmias
Sudden death
Cardiogenic shock
Ruptured papillary muscle or chordae tendinae
Pericarditis
20 Mr. Askew has a total blood cholesterol of 8.1mmol/l. Which type of drug would you
consider prescribing?
Statin.
262
21 Give two side effects of this type of drug.
Myalgia
Myopathy
22 What lifestyle advice would you give Mr. Askew as part of secondary prevention?
Diet – decrease saturated fat intake, decrease salt intake, increase fibre intake (fruit and
vegetable intake).
Exercise – advise programme of cardiac rehabilitation leading to at least 30 minutes
of aerobic exercise three times per week.
Smoking – remain abstinent.
23 Another patient dies 3 days after having a myocardial infarction. What histological
features would you expect to find in the myocardium at post mortem?
Marked necrosis and inflammation is seen 2-4 days post-MI.
24 The relatives of the deceased patient are grieving. Name and outline one
psychological model which attempts to explain this process.
Worden’s four-stage model of grieving:
Stage 1- Accept the reality of the loss
Stage 2- Experience the pain of grief
Stage 3- Adjust to an environment without the deceased person
Stage 4- Withdraw emotional energy from the deceased person and reinvest this in
other relationships
Topic “ACUTE CORONARY SYNDROME”
7.3 Angina pectoris
By the end of Phase II students should be able to:
•recognise angina
•initiate investigations to confirm the diagnosis and assess the severity of underlying coronary
artery disease
•recognise the risk factors in individual patients
•initiate immediate management
7.4 Acute myocardial infarction
By the end of Phase II students should be able to:
263
•recognise acute myocardial infarction and use appropriate investigations to confirm the
diagnosis
•act appropriately to ensure that those patients likely to benefit receive thrombolysis as quickly
as possible
•control the pain of myocardial infarction
•recognise ventricular fibrillation and carry out immediate management
•recognise the need for further active management in the medium to long term
•describe the features of the underlying pathophysiological changes which can be used to
develop treatment and prevention strategies.
264
7.5
Rose, is an 83 year old lady, who presents to the same GP, requesting a ‘medical’. Rose
needs a medical certificate to satisfy the requirements for a local charity bungee-jump.
One of the first actions performed by the GP was to take Rose’s blood pressure.
Question> What are the diastolic and systolic values of a blood pressure reading, above
which, constitute hypertension?
Controversial numbers but essentially 140(systole) / 90 (diastole).
The GP notes that Rose has a large arm but the GP only has a small blood pressure cuff,
as the room had previously been used for a paediatric clinic.
Question> If the GP were to progress with the blood pressure recording using this
equipment, what effect would it have on the blood pressure reading, and why?
Blood pressure reading would be inaccurate and the reading (both systolic and
diastolic) would be higher than the actual blood pressure. The small cuff would not
occlude the brachial artery even at a high cuff pressure, thus giving a false high
reading.
In search of the correct equipment, the GP comes across an automated oscillometric
blood pressure device (e.g. ‘Dinamap’).
Question> State one limitation of this device?
Does not measure diastolic pressure (it calculates it using systolic pressure and mean
arterial pressure).
The machine may not be automated.
Need electrical/battery supply.
Requires user education.
Requires blood pressure recording to be done twice consecutively for a more
accurate measurement.
Is less accurate with arrhythmias.
The GP thinks Rose may have hypertension, so advises her to return to the surgery on
2 more occasions.
265
If Rose expresses anxiety associated with attending the doctor’s surgery, the GP may
want to investigate her blood pressure using a 24 hour blood pressure monitor.
Question> With a diagnosis of hypertension, the GP wants to assess end-organ
damage.
He looks into her eyes with a fundoscope. List the 4 stages of hypertensive retinopathy
outlining a characteristic feature of each.
Grade 1. Silver wiring; tortuous retinal arteries with increased reflectiveness.
Grade 2. Grade 1 plus arterio-venous (A-V) nipping; thickened retinal arteries passing
over retinal veins.
Grade 3. Grade 2 plus haemorrhages and soft exudates (i.e. cotton wool spots); small
infarcts.
Grade 4. Grade 3 plus papilloedema (blurring of optic disc margins).
Question> Apart from the eyes, list 3 other organs potentially at risk from uncontrolled
hypertension?
Brain; six times more likely to suffer with a Stroke.
Heart; 3 times more likely to have coronary artery disease.
Kidneys; renal failure.
Peripheral blood vessels; twice as likely to develop PVD. At an increase risk of aortic
dissection.
Question> Although in over 90% of cases, the cause of hypertension is unknown
(‘essential hypertension’), the GP would like to exclude any secondary causes of Rose’s
hypertension. Can you name 4 causes of secondary hypertension?
Renal causes: diabetic nephropathy, chronic glomerulonephritis, polycystic disease,
tubulointerstitial nephritis and renovascular disease (e.g. renal artery stenosis).
Endocrine causes: conn’s, adrenal hyperplasia, phaeochromocytoma, cushing’s and
acromegaly.
Congenital causes: coarctation of the aorta.
Medication: oral contraceptive, NSAID’s, steroids, vasopressin, carbenoxolone,
liquorice derivatives, and sympathomimetics.
Pregnancy.
266
Question> The GP initiates Rose’s hypertension management by giving her some
lifestyle advice. List 4 such interventions that will aid Rose’s hypertension and reduce
her risk of complications.
Weight reduction (BMI<25 ideally).
Low fat (especially saturated fat) diet.
Low sodium intake (<6g/day). Smoking cessation.
Limit alcohol consumption to advisable units (<14 (women) and <21 (men)/week).
Partake in exercise.
Eat fruit, vegetables and fish regularly.
Question> Despite this good advice, it is clear Rose may still need medication to reduce
her blood pressure. List 4 categories of first line medication that can be used to achieve
a reduction in blood pressure?
ACE Inhibitor
β antagonist
Calcium channel blocker
Diuretic.
Angiotensin II receptor antagonist.
α antagonists.
Extended Question> What is Malignant Hypertension and name 2 symptoms that may
occur in someone with malignant hypertension.
A recent and severe rise in blood pressure that, if left untreated, could be life
threatening (>220/120mmHg). It carries an 80%+ mortality rate at 1 year if left
untreated.
Symptoms include: severe headache, visual disturbances, seizures, transient loss of
consciousness, reduced urine output, blood in the urine, symptoms of heart failure (i.e.
frothy white sputum cough, dyspnoea, swollen ankles, orthopnoea, paroxysmal
nocturnal dyspnoea, anorexia, fatigue and nausea).
Rose wasn’t too bothered when the GP advised against performing the bungee jump, she had
done 10 jumps already.
267
7.5
Hypertension
By the end of Phase II
•
students should be able to:
appreciate the importance of recognising and treating hypertension to reduce the risk of
morbidity and mortality from its complications
•
diagnose hypertension having regard to the variability in blood pressure and possible
errors in its measurement
•
look for possible remediable causes
•
recognise end organ damage
•
manage a patient with hypertension having regard to modification of other factors, the
place of non-drug methods, the risks and benefits of pharmacological intervention and
problems of compliance
268